You are on page 1of 65

Aznar vs. Garcia, G.R. No. L-16749, Jan.

3, 1963
FACTS:
Edward Christensen, who at his death was a US
citizen but domiciled in the Philippines, left a
will, devising unto Maria Helen a certain
amount of money and giving the rest of his
estate to Maria Lucy. Helen opposed the
partition on the ground that she is deprived of
her legitime. Her contention is that the law of
California directs that the law of the domicile
(Philippines) should govern the will.
ISSUE: Whether or not the national law or the
domiciliary law should apply
HELD:
The intrinsic validity of wills is governed by the
national law of the decedent. In the present
case, the national law of Edward is the laws of
California. However, there were two conflicting
California laws regarding succession. One is
enunciated in In Re Kaufman (which does
not provide for legitimes) and another is
Art. 946 of the California Civil Code
(which provides that the law of the
domicile applies). SC held that the
national law is Art. 946, which is the
conflict of laws rule of California. The
reason is that In Re Kaufman applies only to
residents while Art. 946 is specific to nonresidents. Thus, since Art. 946 contains a referback to Philippine laws (the law of the
domicile), then Maria Helen is entitled to her
legitime.
Bellis vs. Bellis, G.R. No. L-23678, June 6, 1967
FACTS:
Amos Bellis, a US citizen, died a resident of
Texas. He left two wills -- one devising a
certain amount of money to his first wife and
three illegitimate children and another, leaving
the rest of his estate to his seven legitimate
children. Before partition, the illegitimate
children who are Filipinos opposed on the
ground that they are deprived of their
legitimes.
ISSUE: Whether the applicable law is Texas law
or Philippine laws
HELD:
Applying the nationality rule, the law of Texas
should govern the intrinsic validity of the will
and therefore answer the question on
entitlement to legitimes. But since the law of
Texas was never proven, the doctrine of
processual presumption was applied.
Hence, SC assumed that Texas law is the
same as Philippine laws, which upholds
the nationality rule.

Private International Law

Renvoi doctrine is not applicable because


there is no conflict as to the nationality
and domicile of Bellis. He is both a citizen
and a resident of Texas. So even if
assuming the law of Texas applies the
domiciliary rule, it is still Texas law that
governs because his domicile is Texas.
Gibbs v. Government of Philippine Islands
Facts:
Gibbs and his wife were American nationals,
domiciled in California. They acquired lands in
the Philippines. The wife died in California.
Gibbs was appointed administrator of the
intestate proceedings instituted in Manila.
Gibbs asked the court to adjudicate to him
lands acquired in the Philippines not under our
laws on succession but because in accordance
with the law of California, the community
property of spouses who are citizens of
California, upon the death of the wife previous
to that of the husband, belongs absolutely to
the surviving husband without administration.
CFI granted such to Gibbs upon proof of
California law. The register of deeds refused to
transfer such properties on the ground of nonpayment of inheritance tax. Gibbs argued that
the conjugal right of a California wife in a
community property is a personal right and
even if this was a case of succession, California
law would still apply.
Issue: is Gibbs exempt from inheritance tax?
Held:
The appellee contends that the law of
California should determine the nature
and extent of the title, if any, that was
vested in Eva Johnson Gibbs citing article
9 of the Civil Code. But that, even if the
nature and extent of her title under said
certificates be governed by the law of the
Philippine Islands, the laws of California
govern the succession to such title, citing
the second paragraph of article 10 of the
Civil Code. It is argued that the conjugal right
of the California wife in community real estate
in the Philippine Islands is a personal right and
must, therefore, be settled by the law
governing her personal status, that is, the law
of California. But our attention has not been
called to any law of California that
incapacitates a married woman from acquiring
or holding land in a foreign jurisdiction in
accordance with the lex rei sitae.
The trial court found that under the law of
California, upon the death of the wife, the
entire
community
property
without
administration belongs to the surviving
husband; that he is the absolute owner of
all the community property from the
moment of the death of his wife, not by

Page 1

virtue of succession or by virtue of her


death, but by virtue of the fact that when
the death of the wife precedes that of the
husband he acquires the community
property, not as an heir or as the
beneficiary of his deceased wife, but
because she never had more than an
inchoate interest or expentancy which is
extinguished upon her death. However, the
argument of the appellee apparently leads to
this dilemma: If he takes nothing by succession
from his deceased wife, how can the second
paragraph of article 10 be invoked? Can the
appellee be heard to say that there is a
legal succession under the law of the
Philippine Islands and no legal succession
under the law of California? It seems clear
that the second paragraph of article 10 applies
only when a legal or testamentary succession
has taken place in the Philippines and in
accordance with the law of the Philippine
Islands; and the foreign law is consulted only in
regard to the order of succession or the extent
of the successional rights; in other words, the
second paragraph of article 10 can be invoked
only when the deceased was vested with a
descendible interest in property within the
jurisdiction of the Philippine Islands.
However, the court held that it is
principle firmly established that to the
law of the state in which the land is
situated we must look for the rules which
govern its descent, alienation, and
transfer,
and
for
the
effect
and
construction
of
wills
and
other
conveyances. This fundamental principle is
stated in the first paragraph of article 10 of our
Civil Code as follows: "Personal property is
subject to the laws of the nation of the
owner thereof; real property to the laws
of the country in which it is situated.
Under the provisions of the Civil Code and
the jurisprudence prevailing here, the
wife, upon the acquisition of any conjugal
property, becomes immediately vested
with an interest and title therein equal to
that of her husband, subject to the power
of management and disposition which the
law vests in the husband. It results that the
wife of the appellee was, by the law of the
Philippine Islands, vested of a descendible
interest, equal to that of her husband and the
descendible interest of Eva Johnson Gibbs
in the lands aforesaid was transmitted to
her heirs by virtue of inheritance and this
transmission plainly falls within the
language of section 1536 of Article XI of
Chapter 40 of the Administrative Code
which levies a tax on inheritances.
Fleumer vs. Hix
54 Phil 610
Facts:

Private International Law

The petitioner is a special administrator of


the estate of Edward Hix. He alleged that
the latters will was executed in Elkins,
West Virginia on November 3, 1925 by Hix
who had his residence in that jurisdiction,
and that the laws of that state govern.
To this end, the petitioner submitted a
copy of Section 3868 of Acts 1882, c.84
as found in West Virginia Code, annotated
by Hogg, Charles E., vol.2 1914, p. 1690
and as certified to by the Director of
National Library.
The Judge of the First Instance however
denied the probate of the will on the
grounds that the will did not show the
following:

acknowledgment
by Hix in
the
presence of 2 competent witnesses

Witnesses subscribed to will in


presence of the testator, and of each
other
Hence, this appeal.
Issue:
Is it necessary to prove in this jurisdiction
the existence of such law in West Virginia
as a prerequisite to the allowance and
recording of said will?
Held:
Yes. The laws of the foreign jurisdiction do
not prove themselves in our courts. The
courts of the Philippine Islands are not
authorized to take judicial notice of the laws
of the various states of the American
Union. Such laws must be proved as
facts.
Here the requirements of the law were not
met.

There was no showing that the book


from which an extract was taken was
printed
or
published
under
the
authority of the state of West Virginia,
as provided in Sec 30 of the Code of
Civil Procedure.
Nor was the extract from the law
attested by the certificate of the officer
having charge of the original, under the
seal of the State of West Virginia as
provided in Sec 301.
No evidence was introduced showing
that the extract from the laws of West
Virginia was in force at the time the
alleged will was executed.
Due execution of the will was not
established: only showed testimony of the
petitioner

The court therefore did not err in denying


the probate of the will. The existence of
such law in West
Virginia must be proved.
Miciano v. Brimo

Page 2

Decedent is a Turkish citizen. He left a will


stating that he wishes to dispose of his
properties in accordance with Philippine Laws.
His brother, Andre Brimo opposed the partition.
The appellant's opposition is based on the fact
that the partition in question puts into effect
the provisions of Joseph G. Brimo's will which
are not in accordance with the laws of his
Turkish nationality, for which reason they are
void as being in violation or article 10 of the
Civil Code which states that testamentary
successions shall be governed by the national
law of the deceased. He was therefore
excluded from the inheritance pursuant to a
clause in the decedents will that anyone who
would oppose the decedents wish to dispose
his estate under Philippine law shall have his
share annul or cancelled.

Impossible conditions and those


contrary to law or good morals
shall be considered as not imposed
and shall not prejudice the heir or
legatee in any manner whatsoever,
even should the testator otherwise
provide.
And said condition is contrary to law
because
it
expressly
ignores
the
testator's national law when, according to
article 10 of the civil Code above quoted,
such national law of the testator is the
one
to
govern
his
testamentary
dispositions.
PCIB VS. ESCOLIN
56 SCRA 266
FACTS:

Issue:
Whether or not the declaration that Turkish
laws are impertinent to this case;
Whether or not the appellants exclusion from
the will is valid?
Held:
1st issue
The oppositor did not prove that said
testamentary
dispositions
are
not
in
accordance with the Turkish laws, inasmuch as
he did not present any evidence showing
what the Turkish laws are on the matter,
and in the absence of evidence on such
laws, they are presumed to be the same
as those of the Philippines.
The refusal to give the oppositor another
opportunity to prove such laws does not
constitute an error. It is discretionary with the
trial court, and, taking into consideration that
the oppositor was granted ample opportunity
to introduce competent evidence, the Court
finds no abuse of discretion on the part of the
lower court in this particular. There is,
therefore, no evidence in the record that
the national law of the testator Joseph G.
Brimo was violated in the testamentary
dispositions in question which, not being
contrary to our laws in force, must be
complied with and executed.
2nd issue
The institution of legatees in this will is
conditional, and the condition is that the
instituted legatees must respect the testator's
will to distribute his property, not in
accordance with the laws of his nationality, but
in accordance with the laws of the Philippines.
The fact is, however, that the said condition is
void, being contrary to law, for article 792 of
the civil Code provides the following:

Private International Law

Linnie
Jane
Hodges
died
giving
her
testamentary provisions to her husband. At the
time
of
her
death, she was citizen of Texas but, was,
however domiciled in the Philippines. To see
whether
the testamentary provisions are valid, it is
apparent and necessary to know what law
should
be
applied.
ISSUE:
Whether or not laws of Texas is applicable.
RULING:
Prior evidence already presented to prove the
existence of Texas Law.
It is necessary that the Texas law be
ascertained. Here it must be proven whether a
renvoi
will
happen or whether Texas law makes the
testamentary provisions valid. In line with
Texas
law,
that which should be proven is the law
enforced during the death of Hodges and not in
any
other
time.
The Supreme Court held that for what the
Texas law is on the matter, is a question
of fact to be resolved by the evidence
that would be presented in the probate
court. Texas law at the time of her death
(and not said law at any other time).
Article 16 of the Civil Code provides that
the national law of the person whose
succession
is
under
consideration,
whatever may be the nature of the
property and regardless of the country
wherein said property may be found,

Page 3

shall prevail. However, the Conflict of Law of


Texas, which is the national law of the
testatrix, Linnie Jane Hodges, provide that the
domiciliary law (Philippine law see
paragraph 2, supra) should govern the
testamentary dispositions and successional
rights over movables (personal properties), and
the law of the situs of the property (also
Philippine law as to properties located in the
Philippines) with regards immovable (real
properties). Thus applying the Renvoi
Doctrine, as approved and applied by our
Supreme Court in the case of In The Matter Of
The Testate Estate of Eduard E. Christensen,
G.R. No. L-16749, promulgated January 31,
1963, Philippine law should apply to the
Will of Linnie Jane Hodges and to the
successional rights to her estate insofar
as her movable and immovable assets in
the Philippines are concerned. We shall
not, at this stage, discuss what law should
govern the assets of Linnie Jane Hodges
located in Oklahoma and Texas, because the
only assets in issue in this motion are those
within the jurisdiction of this motion Court in
the two above-captioned Special Proceedings.
Nota bene: When can foreign law be given
judicial notice
1.
2.

If the court of the forum is familiar of


the law
Or it is within the actual knowledge of
the court

Suntay v. Suntay
Jose B. Suntay died intestate leaving properties
in the Philippines and a house in China. He is
survived by children from the 1st marriage and
a child and his widow from the 2nd. Intestate
proceedings were instituted. Thereafter the
widow filed a petition for a probate of a will but
was later denied when the will was lost after
the filing of said petition. On appeal, the
petition was granted since there was
sufficiency to prove the loss of the will. In spite
of the fact that a commission from the probate
court was issued on 24 April 1937 for the
taking of the deposition of Go Toh, an attesting
witness to the will, on 7 February 1938 the
probate court denied a motion for continuance
of the hearing sent by cablegram from China
by the surviving widow and dismissed the
petition. In the meantime the Pacific War
supervened. After liberation, Silvino claimed to
have found a will by his father which was filed,
recorded and probated in the Amoy district
court, Province of Fookien, China and thus filed
a petition in the intestate proceedings praying
for the probate of the will.
Issue:
May a will filed, recorded, and probated in
China be reprobated in the Philippines?
Held:
As to the will claimed to have been executed
on 4 January 1931 in Amoy, China, the law on
the point in Rule 78.

Private International Law

Section 1 of the rule provides:


Wills proved and allowed in a
foreign country, according to the
laws of such country, may be
allowed, filed, and recorded by the
proper Court of First Instance in
the Philippines.
Section 2 provides:
When a copy of such will and the
allowance
thereof,
duly
authenticated, is filed with a petition
for allowance in the Philippines, by the
executor or other person interested, in
the court having jurisdiction, such court
shall fix a time and place for the
hearing, and cause notice thereof to be
given as in case of an original will
presented for allowance.
Section 3 provides:
If it appears at the hearing that
the will should be allowed in the
Philippines, the court shall so
allow it, and a certificate of its
allowance, signed by the Judge,
and attested by the seal of the
courts, to which shall be attached a
copy of the will, shall be filed and
recorded by the clerk, and the will shall
have the same effect as if originally
proved and allowed in such court.
The fact that the municipal district court
of Amoy, China, is a probate court must
be proved. The law of China on procedure in
the probate or allowance of wills must also be
proved. The legal requirements for the
execution of a valid will in China in 1931 should
also be established by competent evidence.
There is no proof on these points.
The unverified answers to the questions
propounded by counsel for the appellant
to the Consul General of the Republic of
China objected to by counsel for the
appellee, are inadmissible, because apart
from the fact that the office of Consul
General does not qualify and make the
person who holds it an expert on the
Chinese law on procedure in probate
matters, if the same be admitted, the
adverse party would be deprived of his
right to confront and cross-examine the
witness. Consuls are appointed to attend to
trade matters.
The order of the municipal district court
of Amoy, China, does not purport to
probate or allow the will which was the
subject of the proceedings. In view thereof,
the will and the alleged probate thereof cannot
be said to have been done in accordance with
the accepted basic and fundamental concepts
and principles followed in the probate and
allowance
of
wills.
Consequently,
the
authenticated transcript of proceedings held in
the municipal district court of Amoy, China,
cannot
be
deemed
and
accepted
as

Page 4

proceedings leading to the probate or


allowance of a will and, therefore, the will
referred to therein cannot be allowed, filed and
recorded by a competent court of this country.

State of Illinois and the will was not in


conformity with the laws of that State.

Likewise, the proceedings had in the


municipal district court of Amoy were for
the purpose of taking the testimony of
two attesting witnesses to the will and
that the order of the municipal district
court of Amoy does not purport to
probate the will. In the absence of proof that
the municipal district court of Amoy is a
probate court and on the Chinese law of
procedure in probate matters, it may be
presumed that the proceedings in the matter of
probating or allowing a will in the Chinese
courts are the a deposition or to a perpetuation
of testimony, and even if it were so it does not
measure same as those provided for in our
laws on the subject. It is a proceedings in
rem and for the validity of such
proceedings
personal
notice
or
by
publication or both to all interested
parties must be made. The interested
parties in the case were known to reside
in the Philippines. The evidence shows
that no such notice was received by the
interested
parties
residing
in
the
Philippines. The proceedings had in the
municipal district court of Amoy, China, may be
likened to or come up to the standard of such
proceedings in the Philippines for lack of notice
to all interested parties and the proceedings
were held at the back of such interested
parties.

Emil Johnson was a citizen of the State of


Illinois. In the absence of clear proof to the
contrary it should be presumed that a person
naturalized in a court of a certain State thereby
becomes a citizen of that State as well as of
the United States.

The decree appealed from is affirmed, without


pronouncement as to costs.
In re: Johnson
Emil H. Johnson, a native of Sweden and a
naturalized citizen of the United States, died in
the city of Manila, leaving a holographic will
and is signed by himself and two witnesses
only, instead of three witnesses required by
section 618 of the Code of Civil Procedure.
However, a petition was presented in the Court
of First Instance of the city of Manila for the
probate of this will, on the ground that Johnson
was at the time of his death a citizen of the
State of Illinois, United States of America; that
the will was duly executed in accordance with
the laws of that State; and hence could
properly be probated here. Petitioner contends
that the decedent is not a citizen of Illinois and
prays to annul the decree of probate and put
the estate into intestate administration, thus
preparing the way for the establishment of the
claim of the petitioner as the sole legitimate
heir of her father.
Issue:
Whether or not judgment from which the
petitioner seeks relief should be set aside
because the testator was not a resident of the

Private International Law

Held:

In Section 625 of the Code of Civil


Procedure it is declared that "the
allowance by the court of a will of real or
personal property shall be conclusive as
to its due execution."
The due execution of a will involves conditions
relating to a number of matters, such as the
age and mental capacity of the testator, the
signing of the document by the testator, or by
someone
in
his
behalf,
and
the
acknowledgment of the instrument by him in
the presence of the required number of
witnesses who affix their signatures to the will
to attest the act. The proof of all these
requisites is involved in the probate; and
as to each and all of them the probate is
conclusive. Our reported cases do not contain
the slightest intimation that a will which has
been probated according to law, and without
fraud, can be annulled, in any other proceeding
whatever, on account of any supposed
irregularity or defect in the execution of the will
or on account of any error in the action of the
court upon the proof adduced before it.
We are not unmindful of the fact that when a
citizen of one State leaves it and takes up his
abode in another State with no intention of
returning, he immediately acquires citizenship
in the State of his new domicile. This is in
accordance with that provision of the
Fourteenth Amendment to the Constitution of
the United States which says that every citizen
of the United States is a citizen of the State
where in he resides. The effect of this
provision necessarily is that a person
transferring his domicile from one State
to another loses his citizenship in the
State of his original above upon acquiring
citizenship in the State of his new abode.
The acquisition of the new State citizenship
extinguishes the old. That situation, in our
opinion, has no analogy to that which
arises when a citizen of an American
State comes to reside in the Philippine
Islands. Here he cannot acquire a new
citizenship; nor by the mere change of
domicile does he lose that which he
brought with him.
Proper rule in taking judicial notice: The
proper rule is to require proof of the statutes of
the States of the American Union whenever
their provisions are determinative of the issues
in any action litigated in the Philippine courts.
Collector vs. Fisher

Page 5

Facts:
This case relates to the determination and
settlement of the hereditary estate left by the
deceased Walter G. Stevenson, and the laws
applicable thereto. Walter G. Stevenson (born
in the Philippines on August 9, 1874 of British
parents and married in the City of Manila on
January 23, 1909 to Beatrice Mauricia
Stevenson another British subject) died on
February 22, 1951 in San Francisco,
California, U.S.A. whereto he and his wife
moved and established their permanent
residence since May 10, 1945. In his will
executed in San Francisco on May 22, 1947,
and which was duly probated in the Superior
Court of California on April 11, 1951,
Stevenson instituted his wife Beatrice as
his sole heiress to the following real and
personal properties acquired by the
spouses while residing in the Philippines.
Ancillary administration proceedings were
instituted in the Court of First Instance of
Manila for the settlement of the estate in the
Philippines. In due time Stevenson's will was
duly admitted to probate by our court and Ian
Murray
Statt
was
appointed
ancillary
administrator of the estate, filed a preliminary
estate and inheritance tax return with the
reservation of having the properties declared
therein finally appraised at their values six
months after the death of Stevenson.
Preliminary return was made by the ancillary
administrator in order to secure the waiver of
the Collector of Internal Revenue on the
inheritance tax due on the 210,000 shares of
stock in the Mindanao Mother Lode Mines Inc.
which the estate then desired to dispose in the
United States. Acting upon said return, the
Collector of Internal Revenue accepted the
valuation of the personal properties declared
therein, but increased the appraisal of the two
parcels of land located in Baguio City by fixing
their fair market value. After allowing the
deductions
claimed
by
the
ancillary
administrator for funeral expenses in the
amount of P2,000.00 and for judicial and
administration expenses in the sum of
P5,500.00, the Collector assessed the state the
amount of P5,147.98 for estate tax and
P10,875,26 or inheritance tax, or a total of
P16,023.23. Both of these assessments were
paid by the estate.
The ancillary administrator filed in
amended estate and inheritance tax
return in pursuance of his reservation
made at the time of filing of the
preliminary return and for the purpose of
availing of the right granted by section 91
of the National Internal Revenue Code.
Beatrice Mauricia Stevenson assigned all her
rights and interests in the estate to the
spouses,
Douglas
and
Bettina
Fisher,
respondents herein.
The ancillary administrator filed a second
amended estate and inheritance tax return.
This return declared the same assets of the
estate stated in the amended return of
September 22, 1952, except that it contained
new claims for additional exemption and
deduction to wit: (1) deduction in the
amount of P4,000.00 from the gross
estate of the decedent as provided for in
Section 861 (4) of the U.S. Federal

Private International Law

Internal Revenue Code which the ancillary


administrator averred was allowable by
way of the reciprocity granted by Section
122 of the National Internal Revenue
Code, as then held by the Board of Tax
Appeals in case No. 71 entitled "Housman vs.
Collector,"
August
14,
1952;
and
(2)
exemption from the imposition of estate
and inheritance taxes on the 210,000
shares of stock in the Mindanao Mother
Lode Mines, Inc. also pursuant to the
reciprocity proviso of Section 122 of the
National Internal Revenue Code. In this last
return, the estate claimed that it was liable
only for the amount of P525.34 for estate tax
and P238.06 for inheritance tax and that, as a
consequence,
it
had
overpaid
the
government. The refund of the amount of
P15,259.83,
allegedly
overpaid,
was
accordingly requested by the estate. The
Collector denied the claim. For this reason,
action was commenced in the Court of First
Instance of Manila by respondents, as
assignees of Beatrice Mauricia Stevenson, for
the recovery of said amount. Pursuant to
Republic Act No. 1125, the case was forwarded
to the Court of Tax Appeals which court, after
hearing, rendered decision :
that: (a) the one-half () share of the
surviving spouse in the conjugal partnership
property as diminished by the obligations
properly chargeable to such property should
be deducted from the net estate of the
deceased Walter G. Stevenson, pursuant to
Section 89-C of the National Internal Revenue
Code; (b) the intangible personal property
belonging to the estate of said Stevenson
is exempt from inheritance tax, pursuant to
the provision of section 122 of the National
Internal Revenue Code in relation to the
California Inheritance Tax Law but decedent's
estate is not entitled to an exemption of
P4,000.00 in the computation of the estate tax;
(c) for purposes of estate and inheritance
taxation the Baguio real estate of the spouses
should be valued at P52,200.00, and 210,000
shares of stock in the Mindanao Mother Lode
Mines, Inc. should be appraised at P0.38 per
share; and (d) the estate shall be entitled to a
deduction of P2,000.00 for funeral expenses
and judicial expenses of P8,604.39.
Issue: whether or not foreign law needs to be
proved in our jurisdiction?
Ruling:
It is well-settled that foreign laws do not
prove themselves in our jurisdiction and
our courts are not authorized to take
judicial notice of them. Like any other fact,
they must be alleged and proved. Section 41,
Rule 123 of our Rules of Court prescribes the
manner of proving foreign laws before our
tribunals. However, although we believe it
desirable that these laws be proved in
accordance with said rule, we held in the case
of Willamette Iron and Steel Works v.
Muzzal, 61 Phil. 471, that "a reading of sections
300 and 301 of our Code of Civil Procedure
(now section 41, Rule 123) will convince one
that these sections do not exclude the
presentation of other competent evidence to
prove the existence of a foreign law." In that

Page 6

case, we considered the testimony of an


attorney-at-law
of
San
Francisco,
California who quoted verbatim a section
of California Civil Code and who stated
that the same was in force at the time the
obligations were contracted, as sufficient
evidence to establish the existence of
said law. In line with this view, we find no
error, therefore, on the part of the Tax Court in
considering the pertinent California law as
proved by respondents' witness.

the death of the testator. Since no right to


share in the inheritance in favor of a
divorced wife exists in the State of
Nevada and since the court below had already
found that there was no conjugal property
between the testator and Magdalena C.
Bohanan, the latter can now have no longer
claim to pay portion of the estate left by the
testator.

Phil Trust Company vs. Bohanan

whether
the
estementary
dispositions,
especially those for the children which are
short of the legitime given them by the Civil
Code of the Philippines, are valid?

Facts:
Appeal against an order of the Court of First
Instance of Manila, Hon. Ramon San Jose,
presiding, dismissing the objections filed by
Magdalena C. Bohanan, Mary Bohanan and
Edward Bohanan to the project of partition
submitted by the executor and approving the
said project.
The Court of First Instance of Manila, Hon.
Rafael Amparo, presiding, admitted to probate
a last will and testament of C. O. Bohanan,
executed by him in Manila. In the said order,
the court made the following findings:
According to the evidence of the opponents the
testator was born in Nebraska and
therefore a citizen of that state, or at
least a citizen of California where some of
his properties are located. This contention
is untenable. Notwithstanding the long
residence of the decedent in the Philippines,
his stay here was merely temporary, and he
continued and remained to be a citizen of the
United States and of the state of his pertinent
residence to spend the rest of his days in that
state. His permanent residence or domicile in
the United States depended upon his personal
intent or desire, and he selected Nevada as his
homicide and therefore at the time of his
death, he was a citizen of that state. Nobody
can choose his domicile or permanent
residence for him. That is his exclusive
personal right.
Wherefore, the court finds that the testator C.
O. Bohanan was at the time of his death a
citizen of the United States and of the State of
Nevada and declares that his will and
testament, is fully in accordance with the laws
of the state of Nevada and admits the same to
probate. Accordingly, the Philippine Trust
Company, named as the executor of the will, is
hereby appointed to such executor and upon
the filing of a bond in the sum of P10,000.00.
The executor filed a project of partition dated
January 24, 1956, making adjudications, in
accordance with the provisions of the will.
The wife Magadalena C. Bohanan and her
two children question the validity of the
testamentary provisions disposing of the
estate in the manner above indicated,
claiming that they have been deprived of
the legitime that the laws of the forum
concede to them.
Moreover, the court below had found that the
testator and Magdalena C. Bohanan were
married on January 30, 1909, and that
divorce was granted to him on May 20, 1922;
that sometime in 1925, Magdalena C.
Bohanan married Carl Aaron and this
marriage was subsisting at the time of

Private International Law

Issue:

Ruling:
The old Civil Code, which is applicable to
this case because the testator died in
1944,
expressly
provides
that
successional rights to personal property
are to be earned by the national law of
the person whose succession is in question.
Says the law on this point:
Nevertheless,
legal
and
testamentary
successions, in respect to the order of
succession as well as to the extent of the
successional rights and the intrinsic validity of
their provisions, shall be regulated by the
national law of the person whose succession is
in question, whatever may be the nature of the
property and the country in which it is found.
(par. 2, Art. 10, old Civil Code, which is the
same as par. 2 Art. 16, new Civil Code.)
In the proceedings for the probate of the will, it
was found out and it was decided that the
testator was a citizen of the State of Nevada
because he had selected this as his domicile
and his permanent residence. It is not disputed
that the laws of Nevada allow a testator to
dispose of all his properties by will. It does not
appear that at time of the hearing of the
project of partition, the above-quoted provision
was introduced in evidence, as it was the
executor's duly to do. The law of Nevada,
being a foreign law can only be proved in
our courts in the form and manner
provided for by our Rules, which are as
follows:
SEC. 41. Proof of public or official record.
An official record or an entry therein,
when admissible for any purpose, may be
evidenced by an official publication
thereof or by a copy attested by the
officer having the legal custody of the
record, or by his deputy, and accompanied, if
the record is not kept in the Philippines, with a
certificate
that
such
officer
has
the
custody. . . . (Rule 123).
We have, however, consulted the records of
the case in the court below and we have found
that during the hearing on October 4, 1954 of
the motion of Magdalena C. Bohanan for
withdrawal of P20,000 as her share, the foreign
law, especially Section 9905, Compiled
Nevada Laws was introduced in evidence
by appellant's counsel. Again said laws
presented by the counsel for the executor
and admitted by the Court during the
hearing of the case on before Judge
Rafael Amparo.

Page 7

In addition, the other appellants, children of


the testator, do not dispute the above-quoted
provision of the laws of the State of Nevada.
Under all the above circumstances, we are
constrained to hold that the pertinent law of
Nevada, especially Section 9905 of the
Compiled Nevada Laws of 1925, can be
taken judicial notice of by us, without
proof of such law having been offered at
the hearing of the project of partition.
As in accordance with Article 10 of the old Civil
Code, the validity of testamentary dispositions
are to be governed by the national law of the
testator, and as it has been decided and it is
not disputed that the national law of the
testator is that of the State of Nevada, already
indicated above, which allows a testator to
dispose of all his property according to his will,
as in the case at bar, the order of the court
approving the project of partition made in
accordance with the testamentary provisions,
must be, as it is hereby affirmed, with costs
against appellants.
Zalamea vs. CA
Facts:
Petitioners-spouses Cesar C. Zalamea and
Suthira Zalamea, and their daughter, Liana
Zalamea, purchased three (3) airline tickets
from the Manila agent of respondent
TransWorld Airlines, Inc. for a flight to New
York to Los Angeles.
The tickets of petitioners-spouses were
purchased at a discount of 75% while that of
their daughter was a full fare ticket. All three
tickets represented confirmed reservations.
Petitioners
received
notice
of
the
reconfirmation of their reservations for said
flight. On the appointed date, however,
petitioners checked in at 10:00 a.m., an hour
earlier than the scheduled flight at 11:00 a.m.
but were placed on the wait-list because the
number of passengers who had checked in
before them had already taken all the seats
available on the flight. Liana Zalamea
appeared as the No. 13 on the wait-list while
the two other Zalameas were listed as "No. 34,
showing a party of two." Out of the 42 names
on the wait list, the first 22 names were
eventually allowed to board the flight to Los
Angeles, including petitioner Cesar Zalamea.
The two others, on the other hand, at No. 34,
being ranked lower than 22, were not able to
fly. As it were, those holding full-fare tickets
were given first priority among the wait-listed
passengers. Mr. Zalamea, who was holding the
full-fare ticket of his daughter, was allowed to
board the plane; while his wife and daughter,
who presented the discounted tickets were
denied boarding.
Even in the next TWA flight to Los Angeles Mrs.
Zalamea and her daughter, could not be
accommodated because it was also fully
booked. Thus, they were constrained to book in
another flight and purchased two tickets from
American Airlines at a cost of Nine Hundred
Eighteen ($918.00) Dollars.
Upon their arrival in the Philippines,
petitioners filed an action for damages
based on breach of contract of air
carriage before the Regional Trial Court of

Private International Law

Makati. As aforesaid, the lower court


ruled in favor of petitioners.
On appeal, the respondent Court of Appeals
held that moral damages are recoverable in a
damage suit predicated upon a breach of
contract of carriage only where there is fraud
or bad faith. Since it is a matter of record
that overbooking of flights is a common
and accepted practice of airlines in the
United States and is specifically allowed
under the Code of Federal Regulations by
the Civil Aeronautics Board, no fraud nor
bad faith could be imputed on respondent
TransWorld Airlines.
Ruling:
That there was fraud or bad faith on the
part of respondent airline when it did not
allow petitioners to board their flight for
Los Angeles in spite of confirmed tickets
cannot be disputed. The U.S. law or
regulation allegedly authorizing overbooking
has never been proved. Foreign laws do not
prove themselves nor can the courts take
judicial notice of them. Like any other fact,
they must be alleged and proved. Written
law may be evidenced by an official
publication thereof or by a copy attested
by the officer having the legal custody of
the record, or by his deputy, and
accompanied with a certificate that such officer
has custody. The certificate may be made
by a secretary of an embassy or legation,
consul general, consul, vice-consul, or
consular agent or by any officer in the
foreign
service
of
the
Philippines
stationed in the foreign country in which
the record is kept, and authenticated by
the seal of his office. 7
Respondent TWA relied solely on the statement
of Ms. Gwendolyn Lather, its customer service
agent, in her deposition dated January 27,
1986 that the Code of Federal Regulations of
the Civil Aeronautics Board allows overbooking.
Aside from said statement, no official
publication of said code was presented as
evidence. Thus, respondent court's finding that
overbooking is specifically allowed by the US
Code of Federal Regulations has no basis in
fact.
Even if the claimed U.S. Code of Federal
Regulations does exist, the same is not
applicable to the case at bar in
accordance with the principle of lex loci
contractus which require that the law of
the place where the airline ticket was
issued should be applied by the court
where the passengers are residents and
nationals of the forum and the ticket is
issued in such State by the defendant
airline. 8 Since the tickets were sold and
issued in the Philippines, the applicable law in
this case would be Philippine law.
Existing jurisprudence explicitly states that
overbooking amounts to bad faith, entitling the
passengers concerned to an award of moral
damages. In Alitalia Airways v. Court of
Appeals, 9 where passengers with confirmed
bookings were refused carriage on the last
minute, this Court held that when an airline
issues a ticket to a passenger confirmed on a
particular flight, on a certain date, a contract of
carriage arises, and the passenger has every

Page 8

right to expect that he would fly on that flight


and on that date. If he does not, then the
carrier opens itself to a suit for breach of
contract of carriage. Where an airline had
deliberately overbooked, it took the risk of
having to deprive some passengers of their
seats in case all of them would show up for the
check in. For the indignity and inconvenience
of being refused a confirmed seat on the last
minute, said passenger is entitled to an award
of moral damages.
Wild Valley Shipping Co. Vs. CA
Facts:
The Philippine Roxas, a vessel owned by
Philippine President Lines, Inc., private
respondent herein, arrived in Puerto Ordaz,
Venezuela, to load iron ore.
Upon the completion of the loading and when
the vessel was ready to leave port, an official
pilot of Venezuela, was designated by the
harbour authorities in Puerto Ordaz to navigate
the Philippine Roxas through the Orinoco River.
The Philippine Roxas experienced some
vibrations when it entered the San Roque
Channel. The vessel proceeded on its way, with
the pilot assuring the watch officer that the
vibration was a result of the shallowness of the
channel.
The master (captain) checked the position of
the vessel and verified that it was in the centre
of the channel.
The Philippine Roxas ran around in the Orinoco
River, thus obstructing the ingress and egress
of vessels.
As a result of the blockage, the Malandrinon, a
vessel owned by herein petitioner Wildvalley
Shipping Company, Ltd., was unable to sail out
of Puerto Ordaz on that day.
Subsequently, Wildvalley Shipping Company,
Ltd. filed a suit with the Regional Trial Court of
Manila, Branch III against Philippine President
Lines, Inc. and Pioneer Insurance Company (the
underwriter/insurer of Philippine Roxas) for
damages in the form of unearned profits, and
interest thereon amounting to US $400,000.00
plus attorney's fees, costs, and expenses of
litigation.
Issue: whether or not Venezuelan
applicable to the case at bar?

law is

Ruling:
It is well-settled that foreign laws do not
prove themselves in our jurisdiction and
our courts are not authorized to take
judicial notice of them. Like any other fact,
they must be alleged and proved.
For a copy of a foreign public document to be
admissible, the following requisites are
mandatory: (1) It must be attested by the
officer having legal custody of the records
or by his deputy; and (2) It must be
accompanied by a certificate by a
secretary of the embassy or legation,
consul general, consul, vice consular or
consular agent or foreign service officer,
and with the seal of his office. The latter
requirement is not a mere technicality but is
intended to justify the giving of full faith and

credit to the genuineness of a document in a


foreign country.
With respect to proof of written laws, parol
proof is objectionable, for the written law itself
is the best evidence. According to the weight of
authority, when a foreign statute is involved,
the best evidence rule requires that it be
proved by a duly authenticated copy of the
statute.
At this juncture, we have to point out that the
Venezuelan law was not pleaded before the
lower court.
A foreign law is considered to be pleaded
if there is an allegation in the pleading
about the existence of the foreign law, its
import and legal consequence on the
event or transaction in issue.
A review of the Complaint revealed that it was
never alleged or invoked despite the fact that
the grounding of the M/V Philippine Roxas
occurred within the territorial jurisdiction of
Venezuela.
We reiterate that under the rules of private
international law, a foreign law must be
properly pleaded and proved as a fact. In the
absence of pleading and proof, the laws of a
foreign country, or state, will be
presumed to be the same as our own local
or domestic law and this is known as
processual presumption.
Board of Commissioners v. Dela Rosa
Facts:
On July 12, 1960, Santiago Gatchalian,
grandfather of William Gatchalian, was
recognized by the Bureau of Immigration as a
native born Filipino citizen following the
citizenship
of
natural
mother
Mariana
Gatchalian. On June 27, 1961, Willian, then
twelve years old, arrives in Manila from
Hongkong together with a daughter and a son
of Santiago. They had with them certificate of
registration and identity issued by the
Philippine consulate in Hongkong based on a
cablegram bearing the signature of the
secretary of foreign affairs, Felixberto Serrano,
and sought admission as Filipino citizens.
On July 6, 1961, the board of special inquiry
admitted the Gatchalians as Filipino citizens
and issued an identification certificate to
William. The board of commissioners was
directed by the Secretary of Justice to Review
all cases where entry was granted on the
ground that the entrant was a Filipino citizen
such included the case of William. As a result of
the decision of the board of special inquiry
which recommended for the reversal of the
decision of the Board of Commissioners. Acting
commissioner issued an order affirming the
decision of the Board of Special Inquiry.
On August 15, 1990, the Commission on
Immigration and Deportatiion ordered the
arrest of William and was released upon
posting P 200,000 cash bond. Thus on the 29th
of the same month, he filed a petition for
certiorari and prohibition before the RTC of
Manila. A motion to dismiss was filed but
denied.
Issue:

Private International Law

Page 9

the law of the country where it


is stipulated.

Whether or not William Gatchalian is to be


declared as a Filipino citizen

However,
intestate
and
testamentary successions, both
with respect to the order of
succession and to the amount
of successional rights and to
the
intrinsic
validity
of
testamentary provisions, shall
be regulated by the national
law of the person whose
succession
is
under
consideration, whatever may
be the nature of the property
and regardless of the country
wherein said property may be
found. (10a)

Held:
William Gatchalian is declared as a Filipino
Citizen. Having declared the assailed marriage
as valid, respondent William Gatchalian follows
the citizenship of his father, a Filipino as
legitimate child. Respondent belongs to a class
of Filipinos who are citizens of the Philippines
at the time of the adoption of the constitution.
In Moy Ya Lim vs. Commissioner of Immigration
(41 SCRA 292 [1971]) and in Lee vs.
Commissioner of Immigration (supra), this
Court declared that:
(e)verytime the citizenship of a person is
material or indispensable in a judicial or
administrative
case,
whatever
the
corresponding court or administrative authority
decides therein as to such citizenship is
generally not considered as res adjudicata,
hence it has to be threshed out again and
again as the occasion may demand.

Art. 17.

When the acts referred to are


executed before the diplomatic
or consular officials of the
Republic of the Philippines in a
foreign country, the solemnities
established by Philippine laws
shall be observed in their
execution.

An exception to the above rule was laid by this


Court in Burca vs. Republic (51 SCRA 248
[1973]), viz:
We declare it to be a sound rule that where the
citizenship of a party in a case is definitely
resolved by a court or by an administrative
agency, as a material issue in the controversy,
after a full-blown hearing with the active
participation of the Solicitor General or his
authorized representative, and this finding or
the citizenship of the party is affirmed by this
Court, the decision on the matter shall
constitute conclusive proof of such party's
citizenship in any other case or proceeding. But
it is made clear that in no instance will a
decision on the question of citizenship in such
cases be considered conclusive or binding in
any other case or proceeding, unless obtained
in accordance with the procedure herein
stated.

Prohibitive laws concerning


persons, their acts or property,
and those which have, for their
object, public order, public
policy and good customs shall
not be rendered ineffective by
laws
or
judgments
promulgated,
or
by
determinations or conventions
agreed upon in a foreign
country. (11a)
Art. 71.

All
marriages
performed
outside the Philippines in
accordance with the laws in
force in the country where they
were performed, and valid
there as such, shall also be
valid in this country, except
bigamous,
polygamous,
or
incestuous
marriages
as
determined by Philippine law.
(19a)

Art. 124.

If the marriage is between a


citizen of the Philippines and a
foreigner, whether celebrated
in the Philippines or abroad,
the following rules shall prevail:

Thus, in order that the doctrine of res judicata


may be applied in cases of citizenship, the
following must be present: 1) a person's
citizenship must be raised as a material issue
in a controversy where said person is a party;
2) the Solicitor General or his authorized
representative took active part in the
resolution thereof, and 3) the finding or
citizenship is affirmed by this Court.
Doctrine of processual presumption
Civil Code
Art. 15.

Art. 16.

Laws relating to family rights


and duties, or to the status,
condition and legal capacity of
persons are binding upon
citizens of the Philippines, even
though living abroad. (9a)
Real property as well as
personal property is subject to

Private International Law

The forms and solemnities of


contracts, wills, and other
public instruments shall be
governed by the laws of the
country in which they are
executed.

1. If the husband is a citizen

of the Philippines while the


wife is a foreigner, the
provisions of this Code
shall govern their relations;

Page 10

2.

If the husband
is a
foreigner and the wife is a
citizen of the Philippines,
the laws of the husband's
country shall be followed,
without prejudice to the
provisions of this Code with
regard
to
immovable
property. (1325a)

Art. 815.

When a Filipino is in a foreign


country, he is authorized to
make a will in any of the forms
established by the law of the
country in which he may be.
Such will may be probated in
the Philippines. (n)

Art. 816.

The will of an alien who is


abroad produces effect in the
Philippines if made with the
formalities prescribed by the
law of the place in which he
resides, or according to the
formalities observed in his
country, or in conformity with
those
which
this
Code
prescribes. (n)

Art. 818.

Art. 819.

Art. 829.

Art. 1039.

Two or more persons cannot


make a will jointly, or in the
same instrument, either for
their reciprocal benefit or for
the benefit of a third person.
(669)
Wills,
prohibited
by
the
preceding article, executed by
Filipinos in a foreign country
shall not be valid in the
Philippines,
even
though
authorized by the laws of the
country where they may have
been executed. (733a)
A revocation done outside the
Philippines, by a person who
does not have his domicile in
this country, is valid when it is
done according to the law of
the place where the will was
made, or according to the law
of the place in which the
testator had his domicile at the
time; and if the revocation
takes place in this country,
when it is in accordance with
the provisions of this Code. (n)
Capacity
to
succeed
is
governed by the law of the
nation of the decedent. (n)

Rule 132 Sec. 25


What attestation of copy must state.
Whenever a copy of a document or record is
attested for the purpose of evidence, the
attestation must state, in substance, that the
copy is a correct copy of the original, or a
specific part thereof, as the case may be. The

Private International Law

attestation must be under the official seal of


the attesting officer, if there be any, or if he be
the clerk of a court having a seal, under the
seal of such court. (26a)
Rule 130
Section 45. Commercial lists and the like.
Evidence of statements of matters of interest
to persons engaged in an occupation contained
in a list, register, periodical, or other published
compilation is admissible as tending to prove
the truth of any relevant matter so stated if
that compilation is published for use by
persons engaged in that occupation and is
generally used and relied upon by them
therein. (39)
Section 46. Learned treatises. A published
treatise, periodical or pamphlet on a subject of
history, law, science, or art is admissible as
tending to prove the truth of a matter stated
therein if the court takes judicial notice, or a
witness expert in the subject testifies, that the
writer of the statement in the treatise,
periodical or pamphlet is recognized in his
profession or calling as expert in the subject.
(40a)
How to acquire nationality
1. by birth
2. by repatriation
3. by naturalization
4. by subrogation and cession
how to lose a nationality
1. release
2. deprivation
3. expiration
4. renunciation
domicile
1. intent to stay
2. physical presence
3. conduct indicative of such intention
Rep. Act No. 9225
signed into law by President Gloria M. Arroyo
on August 29, 2003
SECTION 1. Short Title.-This Act shall be known
as the "Citizenship Retention and Reacquisition
Act of 2003."
SEC. 2. Declaration of Policy.-It is hereby
declared the policy of the State that all
Philippine citizens who become citizens of
another country shall be deemed not to
have lost their Philippine citizenship
under the conditions of this Act.
SEC. 3. Retention of Philippine Citizenship.-Any
provision
of
law
to
the
contrary
notwithstanding, natural-born citizens of the
Philippines who have lost their Philippine
citizenship
by
reason
of
their
naturalization as citizens of a foreign
country are hereby deemed to have

Page 11

reacquired Philippine citizenship upon


taking the following oath of allegiance to
the Republic:
"I ___________________________, solemnly swear
(or affirm) that I will support and defend the
Constitution of the Republic of the Philippines
and obey the laws and legal orders
promulgated
by
the
duly
constituted
authorities of the Philippines; and I hereby
declare that I recognize and accept the
supreme authority of the Philippines and will
maintain true faith and allegiance thereto; and
that I impose this obligation upon myself
voluntarily without mental reservation or
purpose of evasion."
Natural-born citizens of the Philippines
who, after the effectivity of this Act,
become citizens of a foreign country shall
retain their Philippine citizenship upon
taking the aforesaid oath.
SEC. 4. Derivative Citizenship. - The
unmarried child, whether legitimate,
illegitimate or adopted, below eighteen
(18) years of age, of those who reacquire
Philippine citizenship upon effectivity of
this Act shall be deemed citizens of the
Philippines.
SEC. 5. Civil and Political Rights and Liabilities.
- Those who retain or reacquire Philippine
citizenship under this Act shall enjoy full civil
and political rights and be subject to all
attendant liabilities and responsibilities under
existing laws of the Philippines and the
following conditions:
1. Those intending to exercise their right
of suffrage must meet the requirements
under Section 1, Article V of the
Constitution, Republic Act No. 9189,
otherwise known as "The Overseas
Absentee Voting Act of 2003" and other
existing laws;

2. Those seeking elective public office in


the Philippines shall meet the qualifications
for holding such public office as required
by the Constitution and existing laws and,
at the time of the filing of the certificate of
candidacy, make a personal and sworn
renunciation of any and all foreign
citizenship before any public officer
authorized to administer an oath;

3. Those appointed to any public office


shall subscribe and swear to an oath
of allegiance to the Republic of the
Philippines and its duly constituted
authorities prior to their assumption
of office: Provided, That they renounce
their oath of allegiance to the country
where they took that oath;

Private International Law

4. Those intending to practice their


profession in the Philippines shall apply
with the proper authority for a license or
permit to engage in such practice; and
5. That right to vote or be elected or
appointed to any public office in the
Philippines cannot be exercised by, or
extended to, those who:
a.

are candidates for or are occupying


any public office in the country of
which they are naturalized citizens;
and/or

b.

are
in
the
active
service
as
commissioned or noncommissioned
officers in the armed forces of the
country which they are naturalized
citizens.

SEC. 6. Separability Clause. - If any section or


provision of this Act is held unconstitutional or
invalid, any other section or provision not
affected thereby shall remain valid and
effective.
SEC. 7. Repealing Clause. - All laws, decrees,
orders, rules and regulations inconsistent with
the provisions of this Act are hereby repealed
or modified accordingly.
SEC. 8. Effectivity Clause. - This Act shall take
effect after fifteen (15) days following its
publication in the Official Gazette or two (2)
newspapers of general circulation.
ARTICLE IV
CITIZENSHIP
Section 1.
Philippines:

The following are citizens of the

1.
Those who are
citizens
of
the
Philippines at the time
of the adoption of this
Constitution;
2.
Those
whose
fathers or mothers are
citizens
of
the
Philippines;
3.
Those
born
before
January
17,
1973,
of
Filipino
mothers,
who
elect
Philippine
citizenship
upon reaching the age
of majority; and

Page 12

4.
Those who are
naturalized
in
accordance with law.
Section 2.

Natural-born citizens are those


who are citizens of the
Philippines
from
birth
without having to perform
any act to acquire or
perfect
their
Philippine
citizenship. Those who elect
Philippine
citizenship
in
accordance with paragraph (3),
Section 1 hereof shall be
deemed natural-born citizens.

Section 3.
Philippine citizenship may be
lost or reacquired in the manner provided by
law.
Section 4.

Citizens of the Philippines who


marry aliens shall retain
their citizenship, unless by
their act or omission, they are
deemed, under the law, to
have renounced it.

Section 5.

Dual allegiance of citizens is


inimical to the national
interest and shall be dealt
with by law.

NCC Art. 15.

Laws relating to family rights


and duties, or to the status,
condition and legal capacity of
persons are binding upon
citizens of the Philippines, even
though living abroad. (9a)

Art. 16.

Real property as well as


personal property is subject to
the law of the country where it
is stipulated.
However,
intestate
and
testamentary successions, both
with respect to the order of
succession and to the amount
of successional rights and to
the
intrinsic
validity
of
testamentary provisions, shall
be regulated by the national
law of the person whose
succession
is
under
consideration, whatever may
be the nature of the property
and regardless of the country
wherein said property may be
found. (10a)

Art. 66.

When either or both of the


contracting parties are citizens
or subjects of a foreign
country, it shall be necessary,
before a marriage license can
be
obtained,
to
provide

Private International Law

themselves with a certificate of


legal capacity to contract
marriage, to be issued by their
respective
diplomatic
or
consular officials. (13a)
Art. 1039.

Capacity
to
succeed
is
governed by the law of the
nation of the decedent. (n)

Art. 50.

For the exercise of civil rights


and the fulfillment of civil
obligations, the domicile of
natural persons is the place
of their habitual residence.
(40a)

Art. 51.

When the law creating or


recognizing them, or any other
provision does not fix the
domicile of juridical persons,
the same shall be understood
to be the place where their
legal
representation
is
established or where they
exercise
their
principal
functions. (41a)

Art. 99.

No person shall be entitled to a


legal separation who has not
resided in the Philippines for
one year prior to the filing of
the petition, unless the cause
for the legal separation has
taken place within the territory
of this Republic. (Sec. 2a, Act
No. 2710)

Art. 1251.

Payment shall be made in the


place
designated
in
the
obligation.
There
being
no
express
stipulation
and
if
the
undertaking is to deliver a
determinate
thing,
the
payment
shall
be
made
wherever the thing might be at
the moment the obligation was
constituted.
In any other case the place of
payment shall be the domicile
of the debtor.
If the debtor changes his
domicile in bad faith or after he
has incurred in delay, the
additional expenses shall be
borne by him.
These provisions are without
prejudice to venue under the
Rules of Court. (1171a)

Page 13

Art. 58.

Art. 829.

Save
marriages
of
an
exceptional
character
authorized in Chapter 2 of this
Title, but not those under
Article 75, no marriage shall be
solemnized without a license
first being issued by the local
civil
registrar
of
the
municipality where either
contracting party habitually
resides. (7a)

taking the oath recognizes and accepts

A revocation done outside the


Philippines, by a person who
does not have his domicile
in this country, is valid
when it is done according
to the law of the place
where the will was made, or
according to the law of the
place in which the testator
had his domicile at the time;
and if the revocation takes
place in this country, when it is
in accordance with the
provisions of this Code. (n)

Held:

the supreme authority of the Philippines


is

an

unmistakable

and

categorical

affirmation of his undivided loyalty to the


Republic.
Issue:
Whether R.A. 9225 is unconstitutional and
whether the court

R.A. 9225 is constitutional and that the Court


has no jurisdiction yet to pass upon the issue of
dual allegiance. The court held that that the
intent of the legislature in drafting Rep.
Act No. 9225 is to do away with the
provision in Commonwealth Act No. 635
which takes away Philippine citizenship
from natural-born Filipinos who become
naturalized citizens of other countries.
What Rep. Act No. 9225 does is allow dual
citizenship to natural-born Filipino citizens who

Cases:

have lost Philippine citizenship by reason of


their naturalization as citizens of a foreign

AASJS vs. Datumanong

country. On its face, it does not recognize

G.R. No. 160869, May 11, 2007

dual

allegiance.

By

swearing

to

the

Sec. 3, RA 9225 stayed clear of the problem of


dual allegiance and shifted the burden of
confronting the issue of whether or not there is
dual allegiance to the concerned foreign
country.

supreme authority of the Republic, the

FACTS:

the burden of confronting the issue of

person implicitly renounces his foreign


citizenship. Plainly, from Section 3, Rep.
Act No. 9225 stayed clear out of the
problem of dual allegiance and shifted
whether or not there is dual allegiance to

Petitioner filed the instant petition against

the

concerned

foreign

country.

What

respondent, then Secretary of Justice Simeon

happens to the other citizenship was not made

Datumanong, the official tasked to implement

a concern of Rep. Act No. 9225.

laws governing citizenship in order to prevent


the Justice Secretary from implementing R. A.

For its part, the OSG counters that pursuant to

9225.

is

Section 5, Article IV of the 1987 Constitution,

unconstitutional as it violates Sec. 5, Article VI

dual allegiance shall be dealt with by law.

of the Constitution which states that dual

Thus, until a law on dual allegiance is enacted

allegiance of citizens is inimical to national

by Congress, the Supreme Court is without any

interest and shall be dealt with by law. The

jurisdiction to entertain issues regarding dual

Office of the Solicitor General (OSG) claims that

allegiance.

Petitioner

argues

that

RA

9225

Section 2 merely declares as a state policy that


"Philippine citizens who become citizens of

Moreover, Section 5, Article IV of the

another country shall be deemed not to have

Constitution is a declaration of a policy

lost their Philippine citizenship." The OSG

and it is not a self-executing provision.

further claims that the oath in Section 3 does

The legislature still has to enact the law

not allow dual allegiance since the oath taken

on dual allegiance. In Sections 2 and 3 of

by the former Filipino citizen is an effective

Rep. Act No. 9225, the framers were not

renunciation and repudiation of his foreign

concerned with dual citizenship per se,

citizenship. The fact that the applicant

but with the status of naturalized citizens

Private International Law

Page 14

who maintain their allegiance to their


countries

of

naturalization.

origin

even

Congress

after

was

their

given

mandate to draft a law that would set specific


parameters of what really constitutes dual
allegiance. Until this is done, it would be
premature
including

for
this

the
Court,

judicial
to

rule

department,
on

issues

pertaining to dual allegiance.


ELLIS V. REPUBLIC
7 SCRA 962
Petitioner Marvin G. Ellis, a native of San
Fransisco, California and is married to Gloria G.
Ellis in Banger, Maine, United States. Both are
citizens of the United States. Baby Rose was
born on September 26, 1959. Four or five days
later, the mother of Rose left her with the
Heart of Mary Villa an institution for unwed
mothers and their babies stating that she
(the mother) could not take of Rose without
bringing disgrace upon her (the mother's
family.).
Being without issue, on November 22, 1959,
Mr. and Mrs. Ellis filed a petition with the Court
of First Instance of Pampanga for the adoption
of the aforementioned baby. At the time of the
hearing of the petition on January 14, 1960,
petitioner Marvin G. Ellis and his wife had been
in the Philippines for three (3) years, he being
assigned thereto as staff sergeant in the United
States Air Force Base, in Angeles, Pampanga
where both lived at that time. They had been in
the Philippines before, or, to exact, in 1953.
Issue:
Whether or not being permanent residents in
the Philippines, petitioners are qualified to
adopt Baby Rose.
Held:
Inasmuch as petitioners herein are not
domiciled in the Philippines and, hence, nonresident aliens the Court cannot assume and
exercise jurisdiction over the status, under
either the nationality theory or the domiciliary
theory.
Article 335 of the Civil Code of the Philippines,
provides that:
"The following cannot adopt:
xxx
xxx
xxx
(4) Non-resident aliens;".
xxx
xxx
xxx
This legal provisions is too clear to require
interpretation.
Since adoption is a proceedings in rem,
no court may entertain unless it has
jurisdiction, not only over the subject
matter of the case and over the parties,

Private International Law

but also over the res, which is the


personal status of Baby Rose as well as
that of petitioners herein.
Art. 15 adheres to the theory that
jurisdiction over the status of a natural
person is determined by the latters'
nationality. Pursuant to this theory, the Court
has jurisdiction over the status of Baby Rose,
she being a citizen of the Philippines, but not
over the status of the petitioners, who are
foreigners.
Under the Philippines political law, which
is patterned after the Anglo-American
legal system, the Court have, likewise,
adopted the latter's view to the effect
that personal status, in general, is
determined by and/or subject to the
jurisdiction
of
the
domiciliary
law
(Restatement of the Law of Conflict of Laws, p.
86; The Conflict of Laws by Beale, Vol. I, p. 305,
Vol. II, pp. 713-714).
This, perhaps, is the reason why our Civil
Code does not permit adoption by nonresident aliens, and we have consistently
refused to recognize the validity of
foreign decrees of divorce regardless of
the grounds upon which the same are
based involving citizens of the
Philippines who are not bona fide
residents of the forum, even when our laws
authorized absolute divorce in the Philippines
(Ramirez v. Gmur, 42 Phil. 855; Gonayeb v.
Hashim, 30 Phil. 22; Cousine Hix v. Fleumer, 55
Phil. 851; Barretto Gonzales v. Gonzales, 58
Phil. 67; Recto v. Harden, L-6897, Nov. 29,
1955)".
Republic v. Maddela
27 SCRA 702
This is a petition to have the petitioners
Miguela Tan Suat, and Chan Po Lan, all Chinese
Nationals, to be declared a Filipino citizens.
That sometime in the year 1937 Miguela was
legally married to Sy Ing Seng, a Filipino
citizen; likewise, in the year 1961, Chan Po Lan
was legally married to Cu Bon Piao, a Filipino
citizen; and that the petitioners have all the
qualifications and none of the disqualifications
to become Filipino citizens.
The court inquired from Fiscal Veluz, who
represents the Solicitor General, if he has any
opposition to the petition to which the Fiscal
answered that he has no opposition. The Court
had it announced to the public if there is any
opposition to the petitions of both to be
declared a Filipino citizen and nobody in the
crowded courtroom registered his opposition.

Page 15

As such, the petitioners Miguela Tan Suat and


Chan Po Lan were declared a Filipino citizens
by marriage and the Commissioner of
Immigration is hereby ordered to cancel the
necessary alien certificate of registration and
immigrant certificate of residence of the
petitioner and to issue the corresponding
identification card.

Petitioner seeks reconsideration of the decision


in this case which reversed that of the Court of
First Instance of Leyte declaring her a citizen of
the Philippines, the said court have found her
to be married to a Filipino citizen and to
possess all the qualifications and none of the
disqualifications to become Filipino citizen
enumerated in the Naturalization Law.

The Solicitor General filed the instant petitions


instead, including the Commissioner of
Immigration as co-petitioner in view of the fact
that the dispositive parts of the decisions of the
lower court are addressed to him for
compliance.

Issue:

Issue:

The Court cannot grant petitioner-appellee's


prayer for the affirmance of the trial court's
judgment declaring her a Filipino citizen. It
must be noted that the sole and only
purpose of the petition is to have
petitioner declared a Filipino citizen.
Under Philippine laws there can be no
judicial action or proceeding for the
declaration of the citizenship of an
individual. It is as an incident only of the
adjudication of the rights of the parties to
a controversy, that the courts may pass
upon,
and
make
a
pronouncement
relative to, their status.

Whether or not a person claiming to be a


citizen may get a judicial declaration of
citizenship.
Held:
Under Philippine laws, there can be no
action or proceeding for the judicial
declaration of the citizenship of an
individual. Courts of justice exist for the
settlement of justiciable controversies, which
imply a given right, legally demandable and
enforceable, an act or omission violative of said
right, and a remedy, granted or sanctioned by
law, for said breach of right.
As an incident only of the adjudication of
the right of the parties to a controversy,
the court may pass upon, and make a
pronouncement relative to, their status.
Otherwise, such a pronouncement is beyond
judicial power. Thus, for instance, no action or
proceeding may be instituted for a declaration
to the effect that plaintiff or petitioner is
married, or single, or a legitimate child,
although a finding thereon may be made as a
necessary premise to justify a given relief
available only to one enjoying said status. At
times, the law permits the acquisition of a
given status, such as naturalization by judicial
decree. But there is no similar legislation
authorizing the institution of a judicial
proceeding to declare that a given person is a
Filipino Citizen. (Tan v. Republic, L-14159, April
18, 1960).
Burca v. Republic
51 SCRA 248
1st case: Burca not granted citizenship because
such power is granted under the executive
branch.
Moya Lim Yao: alien wife deemed ipso facto
Filipino citizen as long as no disqualifications,
no need to prove the qualifications;
Burca motion for reconsideration:

Private International Law

Whether or not a court may grant a judicial


declaration of citizenship.
Held:

In Moy Ya Lim Yao, the Court emphasized


the administrative procedure that needs
to be followed in the Bureau Immigration
regarding the steps to be taken by an
alien woman married to a Filipino for the
cancellation of her alien certificate of
registration, and thus secure recognition
of her status Filipino citizen. Such a
procedure could be availed of Petitioner.
Judicial recourse would be avoidable to
Petitioner in case of an adverse action by the
Immigration Commissioner.
However,
if
the
decision
of
an
administrative agency on the question of
citizenship, is affirmed by this Court on
the ground that the same is supported by
substantial evidence on the whole record,
there appears to be no valid reason why
such finding should have no conclusive
effect in other cases, where the same
issue is involved. The same observation
holds true with respect to a decision of a court
on the matter of citizenship as a material
matter in issue in the case before it, which is
affirmed by this Court. For the "effective
operation of courts in the social and economic
scheme requires that their decision have the
respect of and be observed by the parties, the
general public and the courts themselves.
According insufficient weight to prior decisions
encourages disrespect and disregard of courts
and their decisions and invites litigation"
(Clear, Res Judicata Reexamined, 57 Yale Law
Journal, 345).

Page 16

Wherefore, the Court declared it to be a


sound rule, that where citizenship of a
party in a case is definitely resolved by a
court or by an administrative agency, as a
material issue in controversy, after a fullblown hearing, with the act participation
of the Solicitor General or his authority
representative, and this finding on the
Citizenship of the party is affirmed by this
Court, the decision on the matter shows
constitute conclusive proof of such
person's citizenship, in another case or
proceeding. But it is made clear that in
instance will a decision on the question of
citizenship in such cases be considered
conclusive or binding in any other case
proceeding, unless obtained in accordance with
the procedure herein stated.
Moy Ya Lim Yao v. Commissioner
41 SCRA 292
Facts:
On 8 February 1961, Lau Yuen Yeung
applied for a passport visa to enter the
Philippines as a non-immigrant. In the
interrogation made in connection with her
application for a temporary visitor's visa to
enter the Philippines, she stated that she
was a Chinese residing at Kowloon,
Hongkong, and that she desired to take a
pleasure trip to the Philippines to visit
her great-grand-uncle Lau Ching Ping for a
period of one month. She was permitted to
come into the Philippines on 13 March
1961, and was permitted to stay for a
period of one month which would expire on
13 April 1961. On the date of her arrival, Asher
Y, Cheng filed a bond in the amount of
P1,000.00 to undertake, among others, that
said Lau Yuen Yeung would actually depart
from the Philippines on or before the expiration
of her authorized period of stay in this country
or within the period as in his discretion the
Commissioner of Immigration or his authorized
representative might properly allow. After
repeated extensions, Lau Yuen Yeung was
allowed to stay in the Philippines up to 13
February 1962. On 25 January 1962, she
contracted marriage with Moy Ya Lim Yao
alias Edilberto Aguinaldo Lim an alleged
Filipino citizen. Because of the contemplated
action of the Commissioner of Immigration to
confiscate her bond and order her arrest and
immediate deportation, after the expiration of
her authorized stay, she brought an action for
injunction with preliminary injunction. At the
hearing which took place one and a half years
after her arrival, it was admitted that Lau Yuen
Yeung could not write either English or
Tagalog. Except for a few words, she could not
speak either English or Tagalog. She could not
name any Filipino neighbor, with a Filipino

Private International Law

name except one, Rosa. She did not know the


names of her brothers-in-law, or sisters-in-law.
The Court of First Instance of Manila (Civil Case
49705) denied the prayer for preliminary
injunction. Moya Lim Yao and Lau Yuen Yeung
appealed.
Issue:
Whether Lau Yuen Yeung ipso facto became a
Filipino citizen upon her marriage to a Filipino
citizen.
Held:
Lau Yuen Yeung, was declared to have become
a Filipino citizen from and by virtue of her
marriage to Moy Ya Lim Yao al as Edilberto
Aguinaldo Lim, a Filipino citizen of 25 January
1962.
Under Section 15 of Commonwealth Act
473, an alien woman marrying a Filipino,
native born or naturalized, becomes ipso
facto a Filipina provided she is not
disqualified to be a citizen of the
Philippines under Section 4 of the same
law. Likewise, an alien woman married to
an alien who is subsequently naturalized
here follows the Philippine citizenship of
her husband the moment he takes his
oath as Filipino citizen, provided that she
does not suffer from any of the
disqualifications under said Section 4.
Whether the alien woman requires to undergo
the naturalization proceedings, Section 15 is a
parallel provision to Section 16. Thus, if the
widow of an applicant for naturalization as
Filipino, who dies during the proceedings, is not
required to go through a naturalization
proceedings, in order to be considered as a
Filipino citizen hereof, it should follow that the
wife of a living Filipino cannot be denied the
same privilege. This is plain common sense
and there is absolutely no evidence that the
Legislature intended to treat them differently.
As the laws of our country, both
substantive and procedural, stand today,
there is no such procedure (a substitute
for naturalization proceeding to enable
the alien wife of a Philippine citizen to
have the matter of her own citizenship
settled and established so that she may
not have to be called upon to prove it
everytime she has to perform an act or
enter into a transaction or business or
exercise a right reserved only to
Filipinos), but such is no proof that the
citizenship is not vested as of the date of
marriage or the husband's acquisition of
citizenship, as the case may be, for the truth is
that the situation obtains even as to nativeborn Filipinos. Everytime the citizenship of a
person is material or indispensible in a judicial
or
administrative
case,
Whatever
the
corresponding court or administrative authority

Page 17

decides therein as to such citizenship is


generally not considered as res adjudicata,
hence it has to be threshed out again and
again as the occasion may demand.
Nota bene:
There are two laws, which govern the Loss of
Philippine
citizenship.
These
are
Commonwealth
Act
No.
63
and
Commonwealth Act No. 473. The former
applies to both natural-born and naturalized
citizens and the latter applies only to
naturalized citizens.
As stated in Commonwealth Act. No. 63,
A Filipino citizen may lose his citizenship in any
of the following ways and/or events:

1. By naturalization in a foreign country;


2. By

express
citizenship;

renunciation

of

oath of allegiance incident


thereto, states that he does so
only in connection with his
service to said foreign country:
And provided, finally, That any
Filipino citizen who is rendering
service to, or is commissioned
in, the armed forces of a
foreign country under any of
the circumstances mentioned
in paragraph (a) or (b), shall
not be permitted to participate
nor vote in any election of the
Republic of the Philippines
during the period of his service
to, or commission in, the
armed forces of said foreign
country. Upon his discharge
from the service of the said
foreign country, he shall be
automatically entitled to the
full enjoyment of his civil and
political rights as a Filipino
citizen;

3. By subscribing to an oath of
allegiance to support the constitution or
laws of a foreign country upon attaining
twenty-one years of age or more:
Provided, however, That a Filipino may not
divest himself of Philippine citizenship in
any manner while the Republic of the
Philippines is at war with any country;

4. By

rendering services to, or


accepting commission in, the armed
forces of a foreign country: Provided,
That the rendering of service to, or the
acceptance of such commission in, the
armed forces of a foreign country, and the
taking of an oath of allegiance incident
thereto, with the consent of the Republic
of the Philippines, shall not divest a
Filipino of his Philippine citizenship if
either of the following circumstances is
present:

a. The

Republic
of
the
Philippines has a defensive
and/or offensive pact of
alliance with the said foreign
country; or

b. The

said
foreign
country
maintains armed forces on
Philippine territory with the
consent of the Republic of
the Philippines: Provided,
That
the
Filipino
citizen
concerned, at the time of
rendering said service, or
acceptance
of
said
commission, and taking the

Private International Law

5. By cancellation of the of the


certificates of naturalization;

6. By having been declared by competent


authority, a deserter of the Philippine
armed forces in time of war, unless
subsequently, a plenary pardon or
amnesty has been granted; and

7. In the case of a woman, upon her


marriage to a foreigner if, by virtue
of the laws in force in her husband's
country, she acquires his nationality.
Commonwealth Act No. 473, Section 18 of
said law provides that:
A naturalization certificate may be cancelled by
a competent judge on any of the following
grounds:

1. If it is shown that said naturalization


certificate
was
fraudulently or illegally;

obtained

2. If the person naturalized shall, within


five years next following the
issuance of said naturalization
certificated, return to his native
country or to some foreign country
and
establish
his
permanent
residence there: Provided, That the
fact
of
the
person
naturalized
remaining for more than one year in his
native country or the country of his
former nationality, or two years in any
other
foreign
country,
shall
be
considered prima facie evidence of his

Page 18

intention of taking up his permanent


residence in the same;

3. If the petition was made on an


invalid declaration of intention

4. If it is shown that the minor children


of the person naturalized failed to
graduate from a public high school
recognized by the Office of Private
Education of the Philippines, where
Philippine history, government and
civics are taught as part of the school
curriculum, through the fault of their
parents either by neglecting to support
them or by transferring them to
another school or schools. A certified
copy of the decree of naturalization
certificate shall be forwarded by the
Clerk of Court of the Department of
Interior and the Bureau of Justice.

5. If it is shown that the naturalized


citizen has allowed himself to be
used as a dummy in violation of
the
constitutional
provisions
requiring Philippine citizenship as
a requisite for the exercise, use or
enjoyment of a right, franchise or
privilege.
The main reason why a decision in a
naturalization proceeding is not res
judicata is because such is not a judicial
adversarial proceeding. Similarly, estoppel
or laches cannot apply to the government in
action for the cancellation of a certificate of
naturalization, since it is a known principle that
the government is never estopped by the
mistakes on the part of its agents.
However, according to the Constitution,
marriage
to
an
alien
would
not
automatically divest a person of his
citizenship, unless he or she performs
certain acts or omission which would
result to the loss of his or her citizenship.
This provision, however, is not retroactive;
thus, does not repatriate those who lost their
Philippine citizenship by marriage under the
1935 and 1973 Constitutions.
Oh Hek How v. Republic
29 SCRA 94
Petitioner Oh Hek How having been granted
naturalization through his petition filed a
motion alleging that he had complied with the
requirements of Republic Act No. 530 and
praying that he be allowed to take his oath of
allegiance as such citizen and issued the
corresponding certificate of naturalization.
The Court of First Instance of Zamboanga del
Norte issued forthwith an order authorizing the

Private International Law

taking of said oath. On that same date,


petitioner took it and the certificate of
naturalization was issued to him.
The Government seasonably gave notice of its
intention to appeal from said order of February
9, 1966 and filed its record on appeal among
the grounds that the oath was taken prior to
judgment having been final and executor.
Issue:
Is the oath valid?
Whether or not a permission to renounce
citizenship is necessary from the Minister of
the Interior of Nationalist China.
Held:
First issue:
The order of February 9, 1966 (oathtaking) had not and up to the present
has not become final and executory in
view of the appeal duly taken by the
Government.
2nd Issue:
It is argued that the permission is not
required by our laws and that the
naturalization of an alien, as a citizen of
the Philippines, is governed exclusively
by such laws and cannot be controlled by
any foreign law. However, the question of
how a Chinese citizen may strip himself of
that status is necessarily governed
pursuant to Articles 15 and 16 of our Civil
Code by the laws of China, not by those
of the Philippines.
As a consequence, a Chinese national cannot
be naturalized as a citizen of the Philippines,
unless he has complied with the laws of
Nationalist China requiring previous permission
of its Minister of the Interior for the
renunciation of nationality.

Section 12 of Commonwealth Act No.


473 provides, however, that before the
naturalization certificate is issued, the
petitioner shall "solemnly swear," inter
alia, that he renounces "absolutely and
forever all allegiance and fidelity to any
foreign
prince,
potentate"
and
particularly to the state "of which" he is
"a subject or citizen." The obvious
purpose of this requirement is to divest
him of his former nationality, before
acquiring
Philippine
citizenship,
because, otherwise, he would have two
nationalities and owe allegiance to two
(2) distinct sovereignties, which our
laws do not permit, except that,
pursuant to Republic Act No. 2639, "the
acquisition of citizenship by a natural-

Page 19

born Filipino citizen from one of the


Iberian and any friendly democratic
Ibero-American countries shall not
produce loss or forfeiture of his
Philippine citizenship, if the law of that
country grants the same privilege to its
citizens and such had been agreed
upon by treaty between the Philippines
and the foreign country from which
citizenship is acquired."
Tecson v. Comelec
424 SCRA 277
Facts:
On 31 December 2003, Ronald Allan Kelly
Poe, also known as Fernando Poe, Jr.
(FPJ), filed his certificate of candidacy for
the position of President of the Republic
of the Philippines under the Koalisyon ng
Nagkakaisang Pilipino (KNP) Party, in the 2004
national elections. In his certificate of
candidacy, FPJ, representing himself to be a
natural-born citizen of the Philippines,
stated his name to be "Fernando Jr.," or
"Ronald Allan" Poe, his date of birth to be
20 August 1939 and his place of birth to
be Manila. Victorino X. Fornier, initiated, on 9
January 2004, a petition before the Commission
on Elections (COMELEC) to disqualify FPJ and to
deny due course or to cancel his certificate of
candidacy upon the thesis that FPJ made a
material
misrepresentation
in
his
certificate of candidacy by claiming to be
a natural-born Filipino citizen when in
truth, according to Fornier, his parents
were foreigners; his mother, Bessie Kelley
Poe, was an American, and his father,
Allan Poe, was a Spanish national, being
the son of Lorenzo Pou, a Spanish
subject. And even if Allan F. Poe was a Filipino
citizen, he could not have transmitted his
Filipino citizenship to FPJ, the latter being an
illegitimate child of an alien mother. Fornier
based the allegation of the illegitimate birth of
FPJ on two assertions: (1) Allan F. Poe
contracted a prior marriage to a certain Paulita
Gomez before his marriage to Bessie Kelley
and, (2) even if no such prior marriage had
existed, Allan F. Poe, married Bessie Kelly only
a year after the birth of FPJ.
Issue:
Whether FPJ was a natural born citizen, so as to
be allowed to run for the offcie of the President
of the Philippines.
Held:
Any conclusion on the Filipino citizenship of
Lorenzo Pou could only be drawn from the
presumption that having died in 1954 at 84
years old, Lorenzo would have been born

Private International Law

sometime in the year 1870, when the


Philippines was under Spanish rule, and
that San Carlos, Pangasinan, his place of
residence upon his death in 1954, in the
absence of any other evidence, could
have well been his place of residence
before death, such that Lorenzo Pou
would have benefited from the "en masse
Filipinization" that the Philippine Bill had
effected in 1902. That citizenship (of
Lorenzo Pou), if acquired, would thereby
extend to his son, Allan F. Poe, father of
respondent FPJ. The 1935 Constitution,
during which regime respondent FPJ has seen
first light, confers citizenship to all persons
whose fathers are Filipino citizens regardless of
whether such children are legitimate or
illegitimate.
Board of Commissioners v. Dela Rosa
Facts:
On July 12, 1960, Santiago Gatchalian,
grandfather of William Gatchalian, was
recognized by the Bureau of Immigration as a
native born Filipino citizen following the
citizenship
of
natural
mother
Mariana
Gatchalian. On June 27, 1961, Willian, then
twelve years old, arrives in Manila from
Hongkong together with a daughter and a son
of Santiago. They had with them certificate of
registration and identity issued by the
Philippine consulate in Hongkong based on a
cablegram bearing the signature of the
secretary of foreign affairs, Felixberto Serrano,
and sought admission as Filipino citizens.
On August 15, 1990, the Commission on
Immigration and Deportatiion ordered the
arrest of William and was released upon
posting P 200,000 cash bond. Thus on the 29th
of the same month, he filed a petition for
certiorari and prohibition before the RTC of
Manila. A motion to dismiss was filed but
denied.
Petitioners, claim that respondent is an alien.
In support of their position, petitioners point
out that Santiago Gatchalian's marriage with
Chu Gim Tee in China as well as the marriage
of Francisco (father of William) Gatchalian to
Ong Chiu Kiok, likewise in China, were not
supported by any evidence other than their
own self-serving testimony nor was there any
showing what the laws of China were. It is the
postulate advanced by petitioners that for the
said marriages to be valid in this country, it
should have been shown that they were valid
by the laws of China wherein the same were
contracted. There being none, petitioners
conclude that the aforesaid marriages
cannot be considered valid. Hence,
Santiago's children, including Francisco,
followed the citizenship of their mother,
having been born outside of a valid
marriage. Similarly, the validity of the
Francisco's
marriage
not
having
been
demonstrated, William and Johnson followed
the citizenship of their mother, a Chinese
national.

Page 20

Issue:
Whether or not William Gatchalian is to be
declared as a Filipino citizen
Held:
In Miciano vs. Brimo (50 Phil. 867 [1924]; Lim
and Lim vs. Collector of Customs, 36 Phil. 472;
Yam Ka Lim vs. Collector of Customs, 30 Phil.
46 [1915]), this Court held that in the
absence of evidence to the contrary,
foreign laws on a particular subject are
presumed to be the same as those of the
Philippines. In the case at bar, there being no
proof of Chinese law relating to marriage,
there arises the presumption that it is the
same as that of Philippine law.
The lack of proof of Chinese law on the matter
cannot be blamed on Santiago Gatchalian
much more on respondent William Gatchalian
who was then a twelve-year old minor. The fact
is, as records indicate, Santiago was not
pressed by the Citizenship Investigation
Board to prove the laws of China relating
to marriage, having been content with
the testimony of Santiago that the
Marriage Certificate was lost or destroyed
during the Japanese occupation of China.
The testimonies of Santiago Gatchalian and
Francisco Gatchalian before the Philippine
consular and immigration authorities regarding
their marriages, birth and relationship to each
other are not self-serving but are admissible in
evidence as statements or declarations
regarding family reputation or tradition in
matters of pedigree (Sec. 34, Rule 130).
Philippine law, following the lex loci
celebrationis, adheres to the rule that a
marriage formally valid where celebrated
is valid everywhere. Referring to marriages
contracted abroad, Art. 71 of the Civil Code
(now Art. 26 of the Family Code) provides that
"all marriages performed outside of the
Philippines in accordance with the laws in force
in the country where they were performed, and
valid there as such, shall also be valid in this
country . . ."
And any doubt as to the validity of the
matrimonial unity and the extent as to how far
the validity of such marriage may be extended
to the consequences of the coverture is
answered by Art. 220 of the Civil Code in this
manner:
"In
case
of
doubt,
all
presumptions favor the solidarity of the
family. Thus, every intendment of law or
facts leans toward the validity of
marriage, the indissolubility of the
marriage
bonds,
the
legitimacy
of
children, the community of property
during marriage, the authority of parents
over their children, and the validity of
defense for any member of the family in
case of unlawful aggression." (Emphasis
supplied). Bearing in mind the "processual
presumption" enunciated in Miciano and other
cases, he who asserts that the marriage is not
valid under our law bears the burden of proof
to present the foreign law.

Private International Law

Having declared the assailed marriages


as valid, respondent William Gatchalian
follows the citizenship of his father
Francisco, a Filipino, as a legitimate child
of the latter. Francisco, in turn is likewise a
Filipino being the legitimate child of Santiago
Gatchalian who (the latter) is admittedly a
Filipino citizen whose Philippine citizenship was
recognized by the Bureau of Immigration in an
order dated July 12, 1960.
Finally, respondent William Gatchalian belongs
to the class of Filipino citizens who became as
such at the time of the adoption of the
Constitution. . . .
Nota bene: for Gatchalian
In Moy Ya Lim vs. Commissioner of Immigration
(41 SCRA 292 [1971]) and in Lee vs.
Commissioner of Immigration (supra), this
Court declared that:
Everytime the citizenship of a person is
material or indispensable in a judicial or
administrative
case,
whatever
the
corresponding court or administrative authority
decides therein as to such citizenship is
generally
not
considered
as
res
adjudicata, hence it has to be threshed out
again and again as the occasion may demand.
An exception to the above rule was laid by this
Court in Burca vs. Republic (51 SCRA 248
[1973]), viz:
We declare it to be a sound rule that
where the citizenship of a party in a case
is definitely resolved by a court or by an
administrative agency, as a material issue
in the controversy, after a full-blown
hearing with the active participation of the
Solicitor
General
or
his
authorized
representative, and this finding or the
citizenship of the party is affirmed by this
Court, the decision on the matter shall
constitute conclusive proof of such party's
citizenship in any other case or proceeding.
But it is made clear that in no instance will a
decision on the question of citizenship in such
cases be considered conclusive or binding in
any other case or proceeding, unless obtained
in accordance with the procedure herein
stated.
Thus, in order that the doctrine of res judicata
may be applied in cases of citizenship, the
following must be present: 1) a person's
citizenship must be raised as a material
issue in a controversy where said person
is a party; 2) the Solicitor General or his
authorized representative took active
part in the resolution thereof, and 3) the
finding or citizenship is affirmed by this
Court.
NUVAL VS. GURRAY
Facts:

This appeal was taken by the petitioner


Gregorio Nuval from the judgment of
the Court of First Instance of La Union,

Page 21

upholding the defense of res judicata


and dismissing the quo warranto
proceedings instituted by the said
Gregorio Nuval against Norbeto Guray
and others, with costs against the
petitioner.

Gregorio Nuval filed, in his dual


capacity as a voter duly qualified and
registered in the election list of the
municipality of Luna and as a duly
registered candidate for the office of
municipal
president
of
said
municipality, a petition against
Norberto Guray asking for the
exclusion of his name from the
election list of said municipality,
not being a qualified voter of said
municipality and he had not resided
therein for six months as required
by
section
431
of
the
said
Administrative Code.

Norbeto Guray was elected to the


office of municipal president of
Luna by a plurality of votes,
Gregorio Nuval obtaining second place.

Gregorio Nuval filed the present action


of quo warranto asking that Norberto
Guray be declared ineligible had a
legal
residence
of
one
year
previous to the election as required
by
section
2174
of
the
said
Administrative Code in order to be
eligible to an elective municipal office.

Norberto Guray had resided in the


municipality
of
Luna,
his
birthplace, where he had married
and had held the office of
municipal treasurer. On that date he
was appointed municipal treasurer of
Balaoan, Province of La Union. The
rules of the provincial treasurer of La
Union, to which Norberto Guray was
subject as such municipal treasurer,
require
that
municipality
treasurers live continuously in the
municipality where they perform they
official duties, in order to be able to
give an account of their acts as such
treasurers at any time. In order to
qualify and be in a position to vote as
an elector in Balaoan in the general
election of 1925, Norberto Guray asked
for the cancellation of his name in the
election lists of Luna, where he had
voted in the general elections of 1922,
alleging as a ground therefore the
following: "On
the
ground
of
transfer of any residence which
took place on the 28th day of June,
1922. My correct and new address

Private International Law

is Poblacion, Balaoan, La Union;"


and in order to be registered in the
subscribed affidavit Exhibit F-1 before
the board of election inspectors of
precinct No. 1 of Balaoan, by virtue of
which he was registered as an elector
of the said precinct, having made use
of the right of suffrage in said
municipality in the general elections of
1925. In his cedula certificates issued
by himself as municipal treasurer of
Balaoan from the year 1923 to 1928,
included, he made it appear that his
residence was the residential district of
Balaoan. In the year 1926, his wife and
children who, up to that time, had lived
in the municipality of Balaoan, went
back to live in the town of Luna in the
house of his wife's parents, due to the
high cost of living in that municipality.
Norberto Guray used to go home to
Luna in the afternoons after office
hours, and there he passed the nights
with his family. His children studied in
the public school of Luna. In January,
1927,
he
commenced
the
construction of a house of strong
materials in Luna, which has not
yet been completed, and neither
be nor his family has lived in it. On
February 1, 1928, Norberto Guray
applied for and obtained vacation leave
to be spent in Luna, and on the 16th of
the same month he filed his resignation
by telegraph, which was accepted on
the same day, also by telegraph.
Nothwithstanding that he was already
provided with a cedula by himself as
municipal treasurer of Balaoan on
January 31, 1928, declaring him
resident of said town, he obtained
another cedula from the municipality of
Luna on February 20, 1928, which was
dated January 15, 1928, in which it is
presented that he resided in the barrio
of Victoria, municipality of Luna,
Province of La Union. On February 23,
1928, Norberto Guray applied for and
obtained the cancellation of his name
in the election list of the municipality of
Balaoan, and on April 14, 1928, he
applied for registration as a voter in
Luna, alleging that he had been
residing in said municipality for thirty
years. For this purpose he made of the
cedula certificate antedated.

Issue:
Whether or not Norberto Guray had the legal
residence of one year immediately prior to the

Page 22

general elections of June 5, 1928, in order to be


eligible to the office of municipal president of
Luna, Province of La Union?

Ruling:
It is an established rule that "where a voter
abandons his residence in a state and
acquires one in another state, he cannot
again vote in the state of his former
residence until he has qualified by a new
period of residence" (20 Corpus Juris, p. 71,
par. 28). "The term 'residence' as so used
is synonymous with 'domicile,' which
imports not only intention to reside in a
fixed place, but also personal presence in
that place, coupled with conduct
indicative of such intention." (People vs.
Bender, 144 N. Y. S., 145.)
Since Norberto Guray abandoned his first
residence in the municipality of Luna and
acquired another in Balaoan, in order to vote
and be a candidate in the municipality of Luna,
he needed to reacquire residence in the latter
municipality for the length of time prescribed
by the law, and for such purpose, he needed
not only the intention to do so, but his
personal presence in said municipality.
VELILLA VS. POSADA

That Arthur Graydon Moody died in


Calcutta, India, on February 18, 1931.

That Arthur Graydon Moody executed


in the Philippine Islands a will, by
virtue of which will, he bequeathed
all his property to his only sister,
Ida M. Palmer, who then was and still is
a citizen and resident of the State of
New York, United States of America.
That on February 24,1931, a petition
for
appointment
of
special
administrator of the estate of the
deceased Arthur Graydon Moody was
filed by W. Maxwell Thebaut with the
Court of First Instance of Manila.
That subsequently or on April 10, 1931,
a petition to the will of the deceased
Arthur Graydon Moody, and the same
was, after hearing, duly probated by
the court in a decree dated May 5,
1931.

Facts:

That on July 14, 1931, Ida M. Palmer


was declared to be the sole and
only heiress of the deceased Arthur
Graydon Moody
That the property left by the late
Arthur
Graydon
Moody consisted
principally of bonds and shares of stock
of corporations organized under the
laws of the Philippine Islands, bank
deposits and other personal properties.
That on July 22, 1931, the Bureau of
Internal Revenue prepared for the
estate of the late Arthur Graydon
Moody an inheritance tax return.

Private International Law

That on September 9, 1931, an income


tax return for the fractional period from
January 1, 1931 to June 30, 1931, was
also prepared by the Bureau of Internal
Revenue for the estate of the said
deceased Arthur Graydon Moody.
That on December 3, 1931, the
committee on claims and appraisals
filed with the court its report.
That on November 4, 1931, and in
answer to the letter mentioned in the
preceding paragraph, the Bureau of
Internal Revenue addressed to the
attorney for Ida M. Palmer another
letter, copy of which marked Exhibit NN
is hereto attached and made a part
hereof.
That the estate of the late Arthur
Graydon Moody paid under protest the
sum of P50,000 on July 22, 1931, and
the other sum of P40,019.75 on
January 19, 1932, making assessment
for inheritance tax and the sum of
P13,001.41 covers the assessment for
income tax against said estate.
That on January 21, 1932, the Collector
of Internal Revenue overruled the
protest made by Ida M. Palmer through
her attorney.
The parties reserve their right to
introduce additional evidence at the
hearing of the present case.
Manila, August 15, 1933.
In addition to the foregoing agreed
statement of facts, both parties
introduced oral and documentary
evidence from which it appears that
Arthur G. Moody, an American citizen,
came to the Philippine Islands in 1902
or 1903 and engaged actively in
business in these Islands up to the time
of his death in Calcutta, India, on
February 18, 1931. He had no business
elsewhere and at the time of his death
left an estate consisting principally of
bonds and shares of stock of
corporations organized under the laws
of the Philippine Islands, bank deposits
and other intangibles and personal
property valued by the commissioners
of appraisal and claims at P609,767.58
and by the Collector of Internal
Revenue
for
the
purposes
of
inheritance tax at P653,657.47. All of
said property at the time of his
death was located and had its
situs within the Philippine Islands.
So far as this record shows, he left
no property of any kind located
anywhere else. In his will,he made a
statement that: Arthur G. Moody, a
citizen of the United States of America,
residing in the Philippine Islands,
hereby publish and declare the
following as my last Will and Testament
. . ..

Ruling:
To effect the abandonment of one's
domicile, there must be a deliberate and
provable choice of a new domicile,
coupled with actual residence in the place

Page 23

chosen, with a declared or provable


intent that it should be one's fixed and
permanent place of abode, one's home.
There is a complete dearth of evidence in the
record that Moody ever established a new
domicile in a foreign country.
Finding no merit in any of the assignments of
error of the appellant, the court affirm the
judgment of the trial court, first, because the
property in the estate of Arthur G. Moody at
the time of his death was located and had its
situs within the Philippine Islands and, second,
because his legal domicile up to the time
of his death was within the Philippine
Islands.

UJANO VS. REPUBLIC


Facts:

Petitioner seeks to reacquire his


Philippine citizenship in a petition
filed before the Court of First Instance
of Ilocos Sur. Petitioner was born 66
years ago of Filipino parents in Magsingal
Ilocos Sur. He is married to Maxima O.
Ujano with whom he has one son, Prospero,
who is now of legal age. He left the
Philippines for the United States of
America in 1927 where after a
residence of more than 20 years he
acquired American citizenship by
naturalization. He returned to the
Philippines on November 10, 1960 to which
he was admitted merely for a temporary
stay. He owns an agricultural land and
a
residential
house
situated
in
Magsingal, Ilocos Sur. He receives a
monthly pension from the Social Security
Administration of the United States of
America. He has no record of conviction
and it is his intention to renounce his
allegiance to the U.S.A. After hearing, the
court a quo rendered decision denying the
petition on the ground that petitioner did
not have the residence required by
law six months before he filed his petition
for reacquisition of Philippine citizenship.

Ruling:

The court a quo, in denying the petition,


made the following comment: "One of the
qualifications
for
reacquiring
Philippine citizenship is that the
applicant 'shall have resided in the
Philippines at least six months before
he applies for naturalization' [Section
3(1), Commonwealth Act No. 63]. This

Private International Law

'residence' requirement in cases of


naturalization,
has
already
been
interpreted to mean the actual or
constructive
permanent
home
otherwise known as legal residence or
domicile (Wilfredo Uytengsu vs. Republic
of the Philippines, 95 Phil. 890). A place in
a country or state where he lives and stays
permanently, and to which he intends to
return after a temporary absence, no
matter how long, is his domicile. In other
words domicile is characterized by
animus manendi. So an alien who has
been admitted into this country as a
temporary visitor, either for business or
pleasure, or for reasons of health, though
actually present in this country cannot be
said to have established his domicile here
because the period of his stay is only
temporary in nature and must leave when
the purpose of his coming is accomplished.
In the present case, petitioner, who is
presently a citizen of the United
States of America, was admitted into
this country as a temporary visitor, a
status he has maintained at the time
of the filing of the present petition for
reacquisition of Philippine citizenship
and which continues up to the
present. Such being the case, he has not
complied with the specific requirement of
law regarding six months residence before
filing his present petition."
CAASI VS. COURT OF APPEALS
Facts:

These two cases were consolidated


because they have the same objective;
the disqualification under Section 68 of
the Omnibus Election Code of the
private respondent, Merito Miguel for
the position of municipal mayor of
Bolinao, Pangasinan, to which he was
elected in the local elections of January
18, 1988, on the ground that he is a
green card holder, hence, a permanent
resident of the United States of
America, not of Bolinao.

In his answer to both petitions, Miguel


admitted that he holds a green card
issued to him by the US Immigration
Service, but he denied that he is a
permanent resident of the United
States. He allegedly obtained the green
card for convenience in order that he
may freely enter the United States for
his periodic medical examination and
to visit his children there. He alleged
that he is a permanent resident of
Bolinao, Pangasinan, that he voted in
all previous elections, including the
plebiscite on February 2,1987 for the

Page 24

ratification of the 1987 Constitution,


and the congressional elections on May
18,1987.

After hearing the consolidated petitions


before it, the COMELEC dismissed the
petitions on the ground that: The
possession of a green card by the
respondent
(Miguel)
does
not
sufficiently establish that he has
abandoned his residence in the
Philippines. On the contrary, inspite
(sic) of his green card, Respondent has
sufficiently indicated his intention to
continuously reside in Bolinao as
shown by his having voted in
successive
elections
in
said
municipality. As the respondent meets
the basic requirements of citizenship
and residence for candidates to
elective local officials (sic) as provided
for in Section 42 of the Local
Government Code, there is no legal
obstacle to his candidacy for mayor of
Bolinao, Pangasinan. (p. 12, Rollo, G.R.
No. 84508).

Issues: (1) whether or not a green card is proof


that the holder is a permanent resident of the
United States, and
(2) whether respondent Miguel had waived his
status as a permanent resident of or immigrant
to the U.S.A. prior to the local elections on
January 18, 1988.
Ruling: Despite his vigorous disclaimer,
Miguel's immigration to the United States in
1984 constituted an abandonment of his
domicile and residence in the Philippines. For
he did not go to the United States merely to
visit his children or his doctor there; he entered
the limited States with the intention to have
there permanently as evidenced by his
application for an immigrant's (not a visitor's or
tourist's) visa. Based on that application of his,
he was issued by the U.S. Government the
requisite green card or authority to reside
there permanently.
To be "qualified to run for elective office" in the
Philippines, the law requires that the candidate
who is a green card holder must have "waived
his status as a permanent resident or
immigrant of a foreign country." Therefore, his
act of filing a certificate of candidacy for
elective office in the Philippines, did not of
itself constitute a waiver of his status as a
permanent resident or immigrant of the United
States. The waiver of his green card should be
manifested by some act or acts independent of
and done prior to filing his candidacy for
elective office in this country. Without such
prior waiver, he was "disqualified to run for any
elective office" (Sec. 68, Omnibus Election
Code).
MARCOS vs COMELEC

Private International Law

(residence and domicile, for purposes of


election laws are synonymous)
The
mischief
which
this
provision

reproduced
verbatim
from
the
1973
Constitution seeks to prevent is the
possibility of a "stranger or newcomer
unacquainted with the conditions and needs of
a community and not identified with the latter,
from an elective office to serve that
community."
Petitioner Imelda Romualdez-Marcos filed her
Certificate of Candidacy for the position of
Representative of the First District of Leyte
with the Provincial Election Supervisor on
March 8, 1995.
Private respondent Cirilo Roy Montejo, the
incumbent Representative of the First District
of Leyte and a candidate for the same position,
filed
a "Petition
for
Cancellation
and
Disqualification" with the Commission on
Elections alleging that petitioner did not meet
the constitutional requirement for residency. In
his petition, private respondent contended that
Mrs. Marcos lacked the Constitution's one year
residency requirement for candidates for the
House of Representatives.
Held:
So settled is the concept (of domicile) in our
election law that in these and other election
law cases, this Court has stated that the mere
absence of an individual from his
permanent
residence
without
the
intention to abandon it does not result in
a loss or change of domicile.
It stands to reason therefore, that petitioner
merely committed an honest mistake in jotting
the word "seven" in the space provided for the
residency qualification requirement. It would
be plainly ridiculous for a candidate to
deliberately and knowingly make a statement
in a certificate of candidacy which would lead
to his or her disqualification.
Residence in the civil law is a material fact,
referring to the physical presence of a person
in a place. A person can have two or more
residences, such as a country residence and a
city residence. Residence is acquired by living
in place; on the other hand, domicile can exist
without actually living in the place. The
important thing for domicile is that, once
residence has been established in one place,
there be an intention to stay there
permanently, even if residence is also
established in some other place.
For political purposes the concepts of residence
and domicile are dictated by the peculiar
criteria of political laws. As these concepts
have evolved in our election law, what

Page 25

has clearly and unequivocally emerged is


the fact that residence for election
purposes is used synonymously with
domicile.
In Nuval vs. Guray, the Court held that
"the term residence. . . is synonymous
with domicile which imports not only
intention to reside in a fixed place, but
also personal presence in that place,
coupled with conduct indicative of such
intention." Larena vs. Teves reiterated the
same doctrine in a case involving the
qualifications of the respondent therein to the
post of Municipal President of Dumaguete,
Negros Oriental. Faypon vs. Quirino, held that
the absence from residence to pursue studies
or practice a profession or registration as a
voter other than in the place where one is
elected does not constitute loss of residence.
So settled is the concept (of domicile) in our
election law that in these and other election
law cases, this Court has stated that the mere
absence of an individual from his permanent
residence without the intention to abandon it
does not result in a loss or change of domicile.
In Co vs. Electoral Tribunal of the House of
Representatives, this Court concluded that the
framers of the 1987 Constitution obviously
adhered to the definition given to the term
residence in election law, regarding it as
having the same meaning as domicile.
A Person cannot have 2 domiciles. As long
as the Domicile was not lost, it continues
to be the same until replaced by a new
one. Marcos did not overtedly abandon
her domicile since even if living in
Malakanyang, she constantly goes home
to her domicile.
Prior to this:
The civil code provides that the wife follows the
domicile of her husband.
JIMENEZ V. REPUBLIC
109 PHIL 273
FACTS:
1. Plaintiff Joel Jimenez filed a complaint
praying of a decree annulling his marriage
with Remedios Canizares.
2. He claimed that the orifice of her
genitals was too small to allow the
penetration of a male organ or penis for
copulation.
3. He also claimed that the condition of her
genitals existed at the time of marriage
and continues to exist.
4. The wife was summoned and served
with a copy of the complaint but she did
not file an answer.
5. The court entered an order requiring
defendant to submit to a physical

Private International Law

examination by a competent lady physician


to determine her physical capacity for
copulation.
6. Defendant did not submit herself to the
examination and the court entered a
decree annulling the marriage.
7. The City Attorney filed a Motion for
Reconsideration, among the grounds that
the defendants impotency has not been
satisfactorily established as required by
law; that she had not been physically
examined because she refused to be
examined.
ISSUE:
Whether or not the marriage may be annulled
on the strength only of the lone testimony of
the husband who claimed and testified that his
wife is impotent.
HELD:

The law specifically enumerates the legal


grounds that must be proved to exist by
indubitable evidence to annul a marriage. In
the case at bar, the annulment of the marriage
in question was decreed upon the sole
testimony of the husband who was expected to
give testimony tending or aiming at securing
the annulment of his marriage he sought and
seeks. Whether the wife is really impotent
cannot be deemed to have been satisfactorily
established because from the commencement
of the proceedings until the entry of the decree
she had abstained from taking part therein.
Although her refusal to be examined or failure
to appear in court show indifference on her
part, yet from such attitude the presumption
arising out of the suppression of evidence
could not arise or be inferred because women
of this country are by nature coy, bashful and
shy and would not submit to a physical
examination unless compelled to by competent
authority.
A physical examination in this case is not selfincriminating. She is not charged with any
offense . She is not being compelled to be a
witness against herself.
Impotency being an abnormal condition
should not be presumed. The
presumption is in favor of potency. The
lone testimony of the husband that his wife is
physically incapable of sexual intercourse is
insufficient to tear asunder the ties that have
bound them together as husband and wife.
Recto v. Harden
100 Phil 427
Facts:
Recto and Harden entered into a contract
for professional services wherein the latter

Page 26

engaged the services of the former as her


counsel against her husband for a claim in their
conjugal property. Mr. Harden previously filed
for divorce against the Mrs. The Court awarded
Mrs. Harden an amount totaling to almost 4
million pesos plus litis expensae.
Subsequently however, Mrs. Harden ordered
her counsel to vacate all orders and
judgments rendered therein, and abandon
and nullify all her claims to the conjugal
partnership existing between her and Mr.
Harden. Later, she entered into an amicable
settlement with Mr. Harden agreeing to a share
of a lesser amount.
Appellee counsel for Mrs. Harden alleged
that the purpose of the said instruments,
executed by Mr. and Mrs. Harden, was to
defeat the claim of the former for attorneys
fees, for which reason, he prayed that the court
grant him the necessary fees.
Appellants assail the contract for professional
services as void, mainly, upon the grounds
that:

1. that Mrs. Harden cannot bind the


2.
3.

4.

conjugal
partnership
without
her
husbands consent; c
that Article 1491 of the Civil Code of
the Philippines in effect prohibits
contingent fees; c
that the contract in question has for its
purpose to secure a decree of divorce,
allegedly in violation of Articles 1305,
1352 and 1409 of the Civil Code of the
Philippines;
that the terms of said contract are
harsh, inequitable and oppressive.

Held:
The first objection has no foundation in
fact, for the contract in dispute does not
seek to bind the conjugal partnership. By
virtue of said contract, Mrs. Harden merely
bound herself or assumed the personal
obligation to pay, by way of contingent fees,
20% of her share in said partnership. The
contract neither gives, nor purports to give, to
the Appellee any right whatsoever, personal or
real, in and to her aforesaid share. The amount
thereof is simply a basis for the computation of
said fees.
For the same reason, the second objection is,
likewise, untenable. Moreover, it has already
been held that contingent fees are not
prohibited in the Philippines and are
impliedly sanctioned by our Cannons (No.
13) of Professional Ethics. (see, also, Ulanday
vs. Manila Railroad Co., 45 Phil., 540, 554.)
Such is, likewise, the rule in the United States
(Legal Ethics by Henry S. Drinker, p. 176).
in the United States, the great weight of
authority recognizes the validity of contracts
for contingent fees, provided such contracts
are not in contravention of public policy, and it
is only when the attorney has taken an
unfair or unreasonable advantage of his
client that such a claim is condemned.

Private International Law

(See 5 Am. Jur. 359 et seq; Ballentine, Law


Dictionary, 2nd ed., p. 276.)

The third objection is not borne out, either by


the language of the contract between them, or
by the intent of the parties thereto. Its
purpose was not to secure a divorce, or to
facilitate or promote the procurement of
a divorce. It merely sought to protect the
interest of Mrs. Harden in the conjugal
partnership, during the pendency of a divorce
suit she intended to file in the United States.
What is more, inasmuch as Mr. and Mrs.
Harden are admittedly citizens of the
United States, their status and the
dissolution thereof are governed
pursuant to Article 9 of the Civil Code of
Spain (which was in force in the
Philippines at the time of the execution of
the contract in question) and Article 15 of
the Civil Code of the Philippines by the
laws of the United States, which sanction
divorce. In short, the contract of services,
between Mrs. Harden and herein Appellee, is
not contrary to law, morals, good customs,
public order or public policy.
It is a basic principle that status, once
established by the personal law of the
party, is given universal recognition.
Therefore, aliens can sue and be sued in
our
courts
subject
to
Philippine
procedural law even on matters relating
to their status and capacity. However, the
law to be applied by Philippine courts in
determining their capacity and status is
their personal law.
The last objection is based upon principles of
equity, but, pursuant thereto, one who seeks
equity must come with clean hands (Bastida, et
al., vs. Dy Buncio & Co., 93 Phil., 195; 30 C.J. S.
475), and Appellants have not done so, for the
circumstances surrounding the case show, to
our satisfaction, that their aforementioned
agreements, ostensibly for the settlement of
the differences between husband and wife,
were made for the purpose of circumventing or
defeating the rights of herein Appellee, under
his above-quoted contract of services with Mrs.
Harden.
BARNUEVO V. FUSTER
29 PHIL 606
FACTS:
Gabriel and Constanza were married in Spain.
Thereafter, Gabriel went to the Philippines,
settled, and acquired real and personal
properties. Constanza later followed. A few
years however, both parties executed a
contract for their separation wherein the wife
returned to Spain and has agreed to be
supported by the husband to be paid in Madrid,
Spain. Eventually, the wife instituted a petition
for divorce here in the Philippines against the
husband. The husband opposed the petition on
the grounds that:

a. neither the trial court nor any other


court in the Philippine Islands has
jurisdiction over the subject matter of

Page 27

b.

the complaint, because, as to the


allowance for support, since neither the
plaintiff
nor
the
defendant
are
residents of Manila, or of any other
place in the Philippine Islands, the
agreement upon the subject was
neither celebrated, nor was it to
be fulfilled, in the Philippine
Islands;
and as to the divorce, because the
action therefore ought to be tried by
the ecclesiastical courts.

In deciding the case, the Court of First Instance


of the city of Manila held itself to have
jurisdiction, decreed the suspension of life in
common between the plaintiff and defendant;
Issue:
Do Philippine Courts have jurisdiction over the
petition for divorce?
What law should apply in construing the term
pesetas?

That by the express provision of article 80 of


the Civil Code of Spain, "jurisdiction in
actions for divorce and nullification of
canonical
marriages
lies
with
ecclesiastical courts," while that of civil
tribunals is limited to civil marriages; that
this being so, the action for divorce brought by
the plaintiff in the cause does not fall within
the jurisdiction of the civil courts,
according to his own law of persons, because
these courts ought to apply the Spanish law in
accordance with the said article 9 of the Civil
Cod of Spain, and this Spanish law grants the
jurisdiction over the present cause to the
ecclesiastical courts, in the place of which no
tribunal of these Islands con subrogate itself.
However, husband was unable to prove by any
law or legal doctrine whatever that the
personal statute of a foreigner carries with it,
to whether he transfers his domicile, the
authority established by the law of his nation to
decree his divorce, which was what he had to
demonstrate.

Held:
As the Court upheld:
The lower court did not commit this error
attributed to him. The defendant had not
proved that he had elsewhere a legal
domicile other than that which he
manifestly had in the Philippines during
the seventeen years preceding the date
of the complaint. On the contrary, it plainly
appears, without proof to the contrary, that
during
this
not
inconsiderable
period,
extending from the year 1892 until a month
prior to the arrival of his wife in the Philippines
in March, 1909, he had constantly resided in
the said Islands, had kept open house, and had
acquired in the city of Manila quite a little real
property which is now the object of the division
of the conjugal society.
It has been established that defendant is
domiciled in the Philippines.
Defendant, although a Spanish subject, was a
resident of these Islands. Article 26 of the Civil
Code that he cites itself provides that
"Spaniards who change their domicile to a
foreign country, where they may be
considered as natives without other
conditions than that of residents therein,
shall be required, in order to preserve the
Spanish nationality, to state that such is
their wish before the Spanish diplomatic
or consular agent, who must record them
in the registry of Spanish residents, as
well as their spouses, should they be
married, and any children they may
have." From this provision, which is the
exclusive and irrefutable law governing the
defendant, we are to conclude that the
domicile of the defendant and the plaintiff is
fully proven, irrespective of the Treaty of Paris.
Section 377 of the Code of Civil Procedure
leaves to the election of the plaintiff the
bringing of a personal action like the one
at bar either in the place where the
defendant may reside or be found, or in
that where the plaintiff resides.
As held by the Husband:

Private International Law

"The jurisdiction of courts and other questions


relating to procedure are considered to be of a
public nature and consequently are generally
submitted to the territorial principle. . . . All
persons that have to demand justice in a case
in which foreigners intervene, since they can
gain nothing by a simple declaration, should
endeavor to apply to the tribunales of the state
which have coercive means (property situated
in the territory) to enforce any decision they
may render. Otherwise, one would expose
himself in the suit to making useless
expenditures which, although he won his case,
would not contribute to secure his rights
because of the court's lack of means to enforce
them." (Torres Campos, "Elementos de
Derecho International Privado," p. 108.)
The provisions of article 80 of the Civil
Law of Spain is only binding within the
dominions
of
Spain.
It
does
not
accompany the persons of the Spanish
subject wherever he may go. He could not
successfully invoke it if he resided in Japan, in
China, in Hongkong or in any other territory not
subject to the dominion of Spain. Foreign
Catholics domiciled in Spain, subject to the
ecclesiastical courts in actions for divorce
according to the said article 80 of the Civil
Code, could not allege lack of jurisdiction
by invoking, as the law of their personal
statute, a law of their nation which gives
jurisdiction in such a case to territorial
courts, or to a certain court within or
without the territory of their nation.

(Benedicto vs. De la Rama, 3 Phil. Rep., 34, and


Ibaez vs. Ortiz, 5 Phil. Rep., 325).
In the present action for divorce the Court of
First Instance of the city of Manila did not lack
jurisdiction over the persons of the litigants,
for, although Spanish Catholic subjects, they
were residents of this city and had their
domicile herein.

Page 28

The Courts of First Instance of the Philippine


Islands have the power and jurisdiction to try
actions for divorce. That of the city of Manila
did not lack jurisdiction by reason of the
subject matter of the litigation.
With respect to their property regime, the
Foral Law presented by the husband in an
affidavit, with which conjugal partnership
is known to be inexistent, cannot apply
since this affidavit was never presented
in proof, was never received by the trial
judge, and cannot seriously be considered
as an effort to establish the law of a
foreign jurisdiction. Sections 300, 301 and
302 of the Code of Civil Procedure, now in force
in these islands, indicate the method by which
the law of a foreign country may be proved.
The Court maintains that the affidavit of a
person not versed in the law, which was
never submitted as proof, never received
by the trial court, and which has never
been subjected to any cross-examination,
is not a means of proving a foreign law on
which the defendant relies. Thus, since no
proof has been submitted to this effect, all the
property of the marriage, says article 1407 of
the Civil Code, shall be considered as conjugal
property until it is proven that it belongs
exclusively to the husband or to the wife.
ON PLAINTIFFS APPEAL
The court did not commit it in applying the rule
contained in article 1287 of the Civil Code.
"The usages or customs of the country
shall be taken into consideration in
interpreting ambiguity in contracts. . . ." If
in the contract the word " pesetas," not being
specific, was ambiguous, then it was in
harmony with this precept to interpret it as
being the peseta then in use or current when
and where the agreement was made,
Mexican being then the usual and current
money in the Philippines.
QUITA V. PADLAN
GR NO. 124371, DECEMBER 22, 1998

FE D. QUITA and Arturo T. Padlan, both


Filipinos, were married in the Philippines on 18
May 1941. They were not however blessed with
children. Somewhere along the way their
relationship soured. Eventually Fe sued Arturo
for divorce in San Francisco, California, U.S.A.
and obtained a final judgment of divorce. She
married thrice thereafter.
On 1972 Arturo died. He left no will.
Respondent Blandina Padlan claiming to be the
surviving spouse of Arturo Padlan, and Claro,
Alexis, Ricardo, Emmanuel, Zenaida and
Yolanda, all surnamed Padlan, named in the
children of Arturo Padlan opposed the petition.
Ruperto T. Padlan, claiming to be the sole
surviving brother of the deceased Arturo, also
intervened.
The court held that no dispute exists as to the
right of the six (6) Padlan children to inherit
from the decedent because there are proofs
that they have been duly acknowledged by him

Private International Law

and petitioner herself even recognizes them as


heirs of Arturo Padlan;
Issue:
Whether or not petitioner could inherit as the
surviving spouse of Arturo?
Held:
Case remanded subject to determination of
petitioners citizenship. However, Private
respondent's claim to heirship was already
resolved by the trial court. She and Arturo
were married on 22 April 1947 while the
prior marriage of petitioner and Arturo
was subsisting thereby resulting in a
bigamous marriage considered void from
the beginning under Arts. 80 and 83 of the
Civil Code. Consequently, she is not a surviving
spouse that can inherit from him as this status
presupposes a legitimate relationship.
Case remanded to the court a quo for further
proceedings since the trial court was not able
to completely ascertain petitioners citizenship.
The trial court must have overlooked the
materiality of this aspect. Once proved that
she was no longer a Filipino citizen at the
time of their divorce, Van Dorn would
become applicable and petitioner could
very well lose her right to inherit from
Arturo.
Nota bene:
Van dorn: ang bana ga laot sa wife since
under
Philippine
law,
theyre
still
married.
LLORENTE V. COURT OF APPEALS
GR No. 124371, November 23, 2000
FACTS:

Lorenzo Llorente and petitioner Paula


Llorente were married in 1937 in the
Philippines. Lorenzo was an enlisted
serviceman of the US Navy. Soon after, he
left
for
the
US
where
through
naturalization, he became a US Citizen.
Upon his visit to his wife, he discovered that
she was living with his brother and a child was
born. The child was registered as
illegitimate but the name of the father
was left blank. Llorente filed a divorce in
California in which Paula was represented by
counsel, John Riley, and actively participated in
the proceedings, which later on became final.
He married Alicia and they lived together for 25
years bringing 3 children. He made his last will
and testament stating that all his properties
will be given to his second marriage. He filed a
petition of probate that made or appointed
Alicia his special administrator of his estate.
Before the proceeding could be terminated,
Lorenzo died. Paula filed a letter of
administration over Llorentes estate. The trial
granted the letter and denied the motion for
reconsideration. An appeal was made to the

Page 29

Court of Appeals, which affirmed and modified


the judgment of the Trial Court that she be
declared co-owner of whatever properties, she
and the deceased, may have acquired in their
25 years of cohabitation.
ISSUE:

Whether or not national law shall apply?

Nota bene:
The following are the requisites of res
judicata:
1. the former judgment must be final;
2. the court that rendered it had
jurisdiction over the subject matter and
the parties;
3. it is a judgment on the merits; and
4. there is between the first and the
second actions an identity of parties,
subject matter and cause of action.

RULING:
Art. 15. Laws relating to family rights and
duties, or to the status, condition and legal
capacity of persons are binding upon
citizens of the Philippines, even though
living abroad.
Art. 16. Real property as well as personal
property is subject to the law of the country
where it is situated.
First, there is no such thing as one American
law. The "national law" indicated in Article 16
of the Civil Code cannot possibly apply to
general American law. There is no such law
governing the validity of testamentary
provisions in the United States. Each State of
the union has its own law applicable to its
citizens and in force only within the State. It
can therefore refer to no other than the law of
the State of which the decedent was a
resident. Second, there is no showing that
the application of the renvoi doctrine is
called for or required by New York State
law.
However,
intestate
and
testamentary
succession, both with respect to the order of
succession and to the amount of successional
rights and to the intrinsic validity of
testamentary provisions, shall be regulated
by the national law of the person whose
succession
is
under
consideration,
whatever may be the nature of the property
and regardless of the country wherein said
property may be found. (emphasis ours)

Likewise, Lorenzo Llorente was already an


American citizen when he divorced Paula.
Such was also the situation when he married
Alicia and executed his will. As stated in
Article 15 of the civil code, aliens may
obtain divorces abroad, provided that
they are valid in their National Law. Thus
the divorce obtained by Llorente is valid
because the law that governs him is not
Philippine Law but his National Law since the
divorce was contracted after he became an
American citizen. Furthermore, his National
Law allowed divorce.

It has been held that in order that a


judgment in one action can be conclusive
as to a particular matter in another action
between the same parties or their privies,
it is essential that:
1.
2.
3.
4.

5.

the issue be identical;


If a particular point or question is in
issue in the second action,
and the judgment will depend on the
determination of that particular point
or question,
a former judgment between the same
parties or their privies will be final and
conclusive in the second if that same
point or question was in issue and
adjudicated in the first suit (Nabus v.
Court of Appeals, 193 SCRA 732
[1991]).
Identity of cause of action is not
required but merely identity of issue.

Art. 15.

Laws relating to family rights


and duties, or to the status,
condition and legal capacity of
persons are binding upon
citizens of the Philippines, even
though living abroad.
(FRDSCL)

Personal status
legal position of an
individual in a society
Capacity
power to acquire and
exercise rights
Incidental to personal status
Foreign laws regulating the persons status and
capacity are to be disregarded where they are
political or penal in character;
Legislative jurisdiction authority of the state
of his nationality or domicile or where he may
be physically present to promulgate laws
affecting his status;
Judicial jurisdiction the authority of the court
to hear and determine the cause of action
Beginning of personality conception provided
subsequent birth
End death
Other questions of status

1. Absence a special legal status, if one


The case was remanded to the court of origin
for determination of the intrinsic validity of
Lorenzo Llorentes will and determination of
the parties successional rights allowing proof
of foreign law.

Private International Law

is not in his domicile and his


whereabouts is unknown;
When can be declared: under civil code

a. Lapse of 2 years without news or


since the receipt of last news

Page 30

b.

Lapse of 5 years if absentee has


left someone in charge of the
administration of his properties

Who may declare:


a. Spouse
b. Heirs
c. Relatives who will benefit in intestacy
d. Those who have rights over the
properties

Art. XV, Sec. 2, 1987 Constitution


Marriage, as an inviolable social institution, is
the foundation of the family and shall be
protected by the State.
NCC Art. 15.

Laws relating to family rights


and duties, or to the status,
condition and legal capacity of
persons are binding upon
citizens of the Philippines, even
though living abroad. (9a)

Art. 17.

The forms and solemnities of


contracts, wills, and other
public instruments shall be
governed by the laws of the
country in which they are
executed.

2. Conflicts problems respecting an


individuals name and extent of
protection against abuse of his name
Right to use a title of nobility is
determined in accordance with his
national law
3. Minority
CAPACITY
1.

2.

When the acts referred to are


executed before the diplomatic
or consular officials of the
Republic of the Philippines in a
foreign country, the solemnities
established by Philippine laws
shall be observed in their
execution.

Juridical capacity the fitness to be the


subject of legal relations; considered
inherent in every natural person and is
lost only through death;
Capacity to act the power to do acts
with legal effects; it is acquired and
may be lost;

Nota bene:

Prohibitive laws concerning


persons, their acts or property,
and those which have, for their
object, public order, public
policy and good customs shall
not be rendered ineffective by
laws
or
judgments
promulgated,
or
by
determinations or conventions
agreed upon in a foreign
country. (11a)

Under Philippine law, the capacity to dispose of


real property is governed by the lex situs while
the capacity to succeed is governed by the
national law of the deceased.
Question:
A 16-year old Cambodian inherited real
properties from the Philippines; he died and left
a will which disposes of his properties in the
Philippines;
supposing,
Cambodian
law
provides that age of majority is 16, is the will
now if probated in the Philippines valid?

Art. 66.

When either or both of the


contracting parties are citizens
or subjects of a foreign
country, it shall be necessary,
before a marriage license can
be
obtained,
to
provide
themselves with a certificate
of legal capacity to contract
marriage, to be issued by
their respective diplomatic or
consular officials. (13a)

Art. 71.

All
marriages
performed
outside the Philippines in
accordance with the laws in
force in the country where they
were performed, and valid
there as such, shall also be
valid in this country, except
bigamous,
polygamous,
or
incestuous
marriages
as
determined by Philippine law.
(19a)

Art. 75.

Marriages
between
Filipino
citizens
abroad
may
be
solemnized by consuls and
vice-consuls of the Republic of
the Philippines. The duties of

If an 18-year old foreigner, where in his


country, the age of majority is 21 goes to the
Philippines and gets married, is such marriage
valid?
Family rights, duties, status, condition
and legal capacity of persons accompany
a person even when he moves to a
foreign country;
MARRIAGE
Art. II Section 12, 1987 Constitution
The State recognizes the sanctity of family
life and shall protect and strengthen the
family as a basic autonomous social
institution. It shall equally protect the life of
the mother and the life of the unborn from
conception. The natural and primary right and
duty of parents in the rearing of the youth for
civic efficiency and the development of moral
character shall receive the support of the
Government.

Private International Law

Page 31

the local civil registrar and of a


judge or justice of the peace or
mayor with regard to the
celebration of marriage shall be
performed by such consuls and
vice-consuls. (n)
FC Article 1.

Art. 7.
by:

Marriage is a special contract


of permanent union between a
man and a woman entered into
in accordance with law for the
establishment of conjugal and
family life. It is the foundation
of the family and an inviolable
social institution whose nature,
consequences, and incidents
are governed by law and not
subject to stipulation, except
that marriage settlements may
fix the property
relations
during the marriage within the
limits provided by this Code.
(52a)
Marriage may be solemnized

1.
2.

3.

4.

5.

Article. 8.

Any incumbent member of


the judiciary within the
court's jurisdiction;
Any priest, rabbi, imam, or
minister of any church or
religious
sect
duly
authorized by his church or
religious
sect
and
registered with the civil
registrar general, acting
within the limits of the
written authority granted
by his church or religious
sect and provided that at
least one of the contracting
parties belongs to the
solemnizing officer's church
or religious sect;
Any
ship
captain
or
airplane chief only in the
case mentioned in Article
31;
Any military commander of
a unit to which a chaplain
is assigned, in the absence
of the latter, during a
military operation, likewise
only
in
the
cases
mentioned in Article 32;
Any consul-general, consul
or vice-consul in the case
provided in Article 10.
(56a)

The
marriage
shall
be
solemnized publicly in the
chambers of the judge or in
open court, in the church,
chapel or temple, or in the
office
the
consul-general,
consul or vice-consul, as the
case
may
be,
and
not
elsewhere, except in cases of
marriages contracted on the
point of death or in remote
places in accordance with

Private International Law

Article 29 of this Code, or


where both of the parties
request the solemnizing officer
in writing in which case the
marriage may be solemnized at
a house or place designated by
them in a sworn statement to
that effect. (57a)
FC Art. 10.

Marriages
between
Filipino
citizens
abroad
may
be
solemnized
by
a
consulgeneral, consul or vice-consul
of
the
Republic
of
the
Philippines. The issuance of the
marriage license and the duties
of the local civil registrar and of
the solemnizing officer with
regard to the celebration of
marriage shall be performed by
said consular official. (75a)

Art. 26.

All
marriages
solemnized
outside the Philippines, in
accordance with the laws in
force in the country where they
were solemnized, and valid
there as such, shall also be
valid in this country, except
those prohibited under Articles
35 (1), (4), (5) and (6), 3637
and 38. (17a)
Where a marriage between
a Filipino citizen and a
foreigner
is
validly
celebrated and a divorce is
thereafter validly obtained
abroad by the alien spouse
capacitating him or her to
remarry, the Filipino spouse
shall
have
capacity
to
remarry under Philippine
law.
(As
amended
by
Executive Order 227)

FC Art. 35.
The following marriages shall
be void from the beginning:
1. Those contracted by any
party below eighteen years
of age even with the
consent of parents or
guardians;
2. Those solemnized by any
person
not
legally
authorized
to
perform
marriages
unless
such
marriages were contracted
with either or both parties
believing in good faith that
the solemnizing officer had
the legal authority to do so;
3. Those solemnized without
license,
except
those
covered
the
preceding
Chapter;
4. Those
bigamous
or
polygamous marriages not
failing under Article 41;
5. Those contracted through
mistake of one contracting

Page 32

6.

party as to the identity of


the other; and
Those
subsequent
marriages that are void
under Article 53.

Art. 36.

A marriage contracted by any


party who, at the time of the
celebration,
was
psychologically
incapacitated
to comply with the essential
marital obligations of marriage,
shall likewise be void even if
such
incapacity
becomes
manifest
only
after
its
solemnization. (As amended by
Executive Order 227)

Art. 37.

Marriages
between
the
following are incestuous and
void
from
the
beginning,
whether relationship between
the parties be legitimate or
illegitimate:
1.
2.

apply to marriage
and
divorce
wherein
both
parties
are
Muslims,
or
wherein only the
male party is a
Muslim and the
marriage
is
solemnized
in
accordance
with
Muslim law or this
Code in any part of
the Philippines.
(2) In case of marriage
between a Muslim
and a non-Muslim,
solemnized not in
accordance
with
Muslim law or this
Code,
the
Civil
Code
of
the
Philippines
shall
apply. .chan robles
virtual law library
(3) Subject
to
the
provisions of the
preceding
paragraphs,
the
essential requisites
and
legal
impediments
to
marriage, divorce,
paternity
and
filiation,
guardianship
and
custody of minors,
support
and
maintenance,
claims
for
customary
dower
(mahr), betrothal,
breach of contract
to
marry,
solemnization and
registration
of
marriage
and
divorce, rights and
obligations
between husband
and wife parental
authority, and the
properly relations
between husband
and wife shall be
governed by this
Code and other
applicable Muslim
laws.

Between ascendants and


descendants of any degree;
and
Between
brothers
and
sisters, whether of the full
or half blood. (81a)

Art. 38.
The
following
marriages shall be void from the beginning for
reasons of public policy:
1.

2.
3.
4.
5.

6.
7.
8.
9.

Muslim
Code
13. Application.

Between
collateral
blood relatives whether
legitimate
or
illegitimate, up to the
fourth civil degree;
Between step-parents
and step-children;
Between parents-in-law
and children-in-law;
Between the adopting
parent and the adopted
child;
Between the surviving
spouse of the adopting
parent and the adopted
child;
Between the surviving
spouse of the adopted
child and the adopter;
Between an adopted
child and a legitimate
child of the adopter;
Between
adopted
children of the same
adopter; and
Between parties where
one, with the intention
to marry the other,
killed
that
other
person's spouse, or his
or her own spouse. (82)
PD
(1) The
this

Private International Law

1083,

Art. 14.

Nature. Marriage is not only


a civil contract but a social
institution.
Its
nature,
consequences and incidents
are governed by this Code and
the Shari'a and not subject to
stipulation, except that the
marriage settlements may to a
certain extent fix the property
relations of the spouses.

Art. 180.

Law
applicable.

The
provisions of the Revised

Art.

provisions of
Title
shall

Page 33

Penal Code relative to the


crime of bigamy shall not
apply to a person married in
accordance with the provisions
of this Code or, before its
effectivity, under Muslim law.

3.

4.
5.

Essential and Formal Requisites


FC Art. 2.
No marriage shall be valid,
unless these essential requisites are present:
1.

2.

Art. 3.
marriage are:

The

Legal capacity of the


contracting parties who
must be a male and a
female; and
Consent freely given in
the presence of the
solemnizing
officer.
(53a)
formal

1.
2.

3.

requisites

6.

Art. 4. The absence of any of the essential or


formal requisites shall render
the marriage void ab initio,
except as stated in Article 35
(2).
A defect in any of the essential
requisites shall not affect the
validity of the marriage but the
party or parties responsible for
the irregularity shall be civilly,
criminally and administratively
liable. (n)

of

Authority
of
the
solemnizing officer;
A
valid
marriage
license except in the
cases provided for in
Chapter 2 of this Title;
and
A marriage ceremony
which takes place with
the appearance of the
contracting
parties
before the solemnizing
officer
and
their
personal
declaration
that they take each
other as husband and
wife in the presence of
not less than two
witnesses of legal age.
(53a, 55a)

Art. 35 (2) supra


Muslim
Code,
requisites.

Art.

15. Essential

No marriage contract shall be perfected


unless the following essential requisites
are compiled with:

a. Legal
b.
c.

Art. 5. Any male or female of the age of


eighteen years or upwards not
under any of the impediments
mentioned in Articles 37 and
38, may contract marriage.
(54a)

d.

Art. 35.
The
following
marriages shall be void from the beginning:
Those contracted by any
party below eighteen years
of age even with the
consent of parents or
guardians;
2. Those solemnized by
any person not legally
authorized to perform
marriages unless such
marriages
were
contracted with either
or both parties believing
in good faith that the
solemnizing officer had
the legal authority to do
so;

Those solemnized without


license,
except
those
covered
the
preceding
Chapter;
Those
bigamous
or
polygamous marriages not
failing under Article 41;
Those contracted through
mistake of one contracting
party as to the identity of
the other; and
Those
subsequent
marriages that are void
under Article 53.

capacity
of
the
contracting parties;
Mutual consent of the parties
freely given;
Offer (ijab) and acceptance
(qabul) duly witnessed by at
least two competent persons
after the proper guardian in
marriage (wali) has given his
consent; and
Stipulation of customary dower
(mahr) duly witnessed by two
competent persons.

1.

Private International Law

GOITIA VS. CAMPOS RUEDA


Facts:
The parties were legally married in the city of
Manila on January 7, 1915, and immediately
thereafter established their residence at 115
Calle San Marcelino, where they lived together
for about a month, when the plaintiff returned
to the home of her parents.
That the defendant, one month after he had
contracted
marriage
with
the
plaintiff,
demanded of her that she perform unchaste

Page 34

and lascivious acts on his genital organs; that


the plaintiff spurned the obscene demands of
the defendant and refused to perform any act
other than legal and valid cohabitation; that
the defendant, since that date had continually
on other successive dates, made similar lewd
and indecorous demands on his wife, the
plaintiff, who always spurned them, which just
refusals of the plaintiff exasperated the
defendant and induce him to maltreat her by
word and deed and inflict injuries upon her lips,
her face and different parts of her body; and
that, as the plaintiff was unable by any means
to induce the defendant to desist from his
repugnant desires and cease from maltreating
her, she was obliged to leave the conjugal
abode and take refuge in the home of her
parents.
Issue: whether or not that the wife may
claim for support against her husband
outside of their conjugal abode?
Ruling: marriage partakes of the nature of an
ordinary contract. But it is something more
than a mere contract. It is a new relation, the
rights, duties, and obligations of which rest
not upon the agreement of the parties
but upon the general law which defines
and prescribes those rights, duties, and
obligations. Marriage is an institution, in the
maintenance of which in its purity the public is
deeply interested. It is a relation for life and
the parties cannot terminate it at any shorter
period by virtue of any contract they may
make .The reciprocal rights arising from this
relation, so long as it continues, are such as
the law determines from time to time, and
none other. When the legal existence of the
parties is merged into one by marriage, the
new relation is regulated and controlled by the
state or government upon principles of public
policy for the benefit of society as well as the
parties. And when the object of a marriage is
defeated
by
rendering
its
continuance
intolerable to one of the parties and productive
of no possible good to the community, relief in
some way should be obtainable. With these
principles to guide us, we will inquire into the
status of the law touching and governing the
question under consideration.
The mere act of marriage creates an
obligation on the part of the husband to
support his wife. This obligation is
founded not so much on the express or
implied terms of the contract of marriage
as on the natural and legal duty of the
husband; an obligation, the enforcement of
which is of such vital concern to the state itself
that the laws will not permit him to terminate it
by his own wrongful acts in driving his wife to
seek protection in the parental home. A
judgment for separate maintenance is not due
and payable either as damages or as a penalty;
nor is it a debt in the strict legal sense of the
term, but rather a judgment calling for the
performance of a duty made specific by the
mandate of the sovereign. This is done from
necessity and with a view to preserve the
public peace and the purity of the wife; as
where the husband makes so base demands
upon his wife and indulges in the habit of
assaulting her. The pro tanto separation

Private International Law

resulting from a decree for separate support is


not an impeachment of that public policy by
which marriage is regarded as so sacred and
inviolable in its nature; it is merely a stronger
policy overruling a weaker one; and except in
so far only as such separation is tolerated as a
means of preserving the public peace and
morals may be considered, it does not in any
respect whatever impair the marriage contract
or for any purpose place the wife in the
situation of a feme sole.
Where the wife, who is forced to leave the
conjugal abode by her husband without fault
on her part, may maintain an action against
the husband for separate maintenance when
she has no other remedy, notwithstanding
the provisions of article 149 of the Civil
Code giving the person who is obliged to
furnish support the option to satisfy it
either by paying a fixed pension or by
receiving and maintaining in his own
home the one having the right to the
same.

WONG WOO YIU VS. VIVO


Facts:
The Board of Special Inquiry No. 3 rendered a
decision finding petitioner to be legally
married to Perfecto Blas and admitting
her into the country as a non-quota
immigrant. This decision was affirmed by the
Board of Commissioners of which petitioner
was duly informed in a letter sent on the same
date by the Secretary of the Board. However,
the same Board of Commissioners, but
composed entirely of a new set of
members, rendered a new decision
reversing that of the Board of Special
Inquiry No. 3 and ordering petitioner to
be excluded from the country. Petitioner
filed a motion for new trial requesting an
opportunity to clarify certain points taken in
the decision, but the same was denied for lack
of merit. Whereupon, petitioner initiated the
instant petition for mandamus with preliminary
injunction before the Court of First Instance of
Manila which incidentally was considered by it
as a petition for certiorari.
Petitioner declared that she came to the
Philippines in 1961 for the first time to
join her husband Perfecto Blas to whom
she was married in Chingkang, China ;that
they had several children all of whom are not
in the Philippines; that their marriage was
celebrated by one Chua Tio, a village
leader; that on June 28, 1961 the Board of
Special Inquiry No. 3 rendered a decision
finding, among others, that petitioner is legally
married to Perfecto Blas, a Filipino Citizen, and
admitted her into the country as a non-quota
immigrant; that this decision was affirmed by
the Board of Commissioners of which petitioner
was duly notified by the Secretary of said
Board in a letter dated July 12, 1961; that in a
motu proprio decision rendered by the Board of
Commissioners composed of a new set of
members the latter found that petitioner's
claim that she is the lawful wife of Perfecto

Page 35

Blas was without basis in evidence as it was


"bereft of substantial proof of husband-wife
relationship"; that said Board further held that,
it appearing that in the entry proceedings of
Perfecto Blas had on January 23, 1947 he
declared that he first visited China in 1935 and
married petitioner in 1936, it could not possibly
sustain her claim that she married Perfecto
Blas in 1929; that in an affidavit dated August
9, 1962 Perfecto Blas claimed that he went to
China in 1929, 1935 and 1941, although in his
re-entry declaration he admitted that he first
went to China in 1935, then in 1937, then in
1939, and lastly in 1941; and that Perfecto Blas
in the same affidavit likewise claimed that he
first went to China when he was merely four
years old so that computed from his date of
birth in 1908 it must have been in 1912.
In view of the discrepancies found in the
statements made by petitioner and her alleged
husband Perfecto Blas in the several
investigations conducted by the immigration
authorities concerning their alleged marriage
before a village leader in China in 1929,
coupled with the fact that the only basis in
support of petitioner's claim that she is the
lawful wife of Perfecto Blas is "a mass of oral
and
documentary
evidence
bereft
of
substantial proof of husband-wife relationship,"
the Board of Commissioners motu proprio
reviewed the record concerning the admission
of petitioner into the country resulting in its
finding that she was improperly admitted.
Issue: whether or not that the marriage is
valid?
Ruling: there is no documentary evidence to
support the alleged marriage of petitioner to
Perfecto Blas but the record is punctured with
so many inconsistencies which cannot but lead
one to doubt their veracity concerning the
pretended marriage in China in 1929. This
claim cannot also be entertained under our law
on family relations. Thus, Article 15 of our
new Civil Code provides that laws relating
to family rights or to the status of
persons are binding upon citizens of the
Philippines, even though living abroad,
and it is well-known that in 1929 in order
that a marriage celebrated in the
Philippines may be valid it must be
solemnized either by a judge of any court
inferior to the Supreme Court, a justice of
the peace, or a priest or minister of the
gospel
of
any
denomination
duly
registered in the Philippine Library and
Museum (Public Act 3412, Section 2). Even if
we assume, therefore, that the marriage of
petitioner to Perfecto Blas before a village
leader is valid in China, the same is not one of
those authorized in our country.
A marriage contracted outside of the
Philippines which is valid under the law of
the country in which it was celebrated is
also valid in the Philippines. But no
validity can be given to this contention
because no proof was presented relative
to the law of marriage in China. Such being
the case, we should apply the general rule that
in the absence of proof of the law of a foreign

Private International Law

country it should be presumed that it is the


same as our own.
Since our law only recognizes a marriage
celebrated before any of the officers
mentioned therein, and a village leader is not
one of them, it is clear that petitioner's
marriage, even if true, cannot be recognized in
this jurisdiction.
ADONG VS. CHAONG SENG GEE
Facts:
Cheong Boo, a native of China died in
Zamboanga, Philippine Islands on August
5, 1919 and left property worth nearly
P100,000 which is now being claimed by
two parties - (1) Cheong Seng Gee who
alleged that he was a legitimate child by
marriage contracted by Cheong Boo with
Tan Bit in China in 1985, and (2) Mora
Adong who alleged that she had been
lawfully married to Cheong Boo in 1896 in
Basilan, Philippine Islands and had two
daughters with the deceased namely Payang
and Rosalia. The conflicting claims to Cheong
Boos estate were ventilated in the lower
court that ruled that Cheong Seng Gee
failed to sufficiently establish the Chinese
marriage through a mere letter testifying
that Cheong Boo and Tan Bit married
each other but that because Cheong Seng
Gee had been admitted to the Philippine
Islands as the son of the deceased, he
should share in the estate as a natural
child. With reference to the allegations of Mora
Adong and her daughters, the trial court
reached the conclusion that the marriage
between Adong and Cheong Boo had been
adequately proved but that under the laws of
the Philippine Islands it could not be held to be
a lawful marriage and thus the daughter
Payang and Rosalia would inherit as natural
children. The lower court believes that
Mohammedan marriages are not valid under
the Philippine Islands laws this as an Imam as
a solemnizing officer and under Quaranic laws.
ISSUES: Whether or not the Chinese marriage
between Cheong Boo and Tan Dit is valid ?
Whether or not the Mohammedan marriage
between Cheong Boo and Mora Adong is valid?
RULING: The Supreme Court found the (1)
Chinese
marriage
not
proved
and
Chinaman Cheong Seng Gee has only the
rights of a natural child while (2) it found
the Mohammedan marriage to be proved
and to be valid, thus giving to the widow
Mora Adong and the legitimate children Payang
and Rosalia the rights accruing to them under
the law.
(FOR STATCON) The Supreme Court held
that marriage in this jurisdiction is not only a
civil contract but it is a new relation, an
instruction in the maintenance of which the
public is deeply interested. The presumption as
to marriage is that every intendment of the law
leans toward legalizing matrimony. Persons

Page 36

dwelling together in apparent matrimony are


presumed, in the absence of counterpresumption or evidence special to the case, to
be in fact married. The reason is that such is
the common order of society, and if the parties
were not what they thus hold themselves out
as being, they would be living in the constant
violation of decency of the law. As to
retroactive force, marriage laws is in the nature
of a curative provision intended to safeguard
society by legalizing prior marriages. Public
policy should aid acts intended to
validate marriages and should retard acts
intended to invalidate marriages. This as
for public policy, the courts can properly incline
the scales of their decision in favor of that
solution which will most effectively promote
the public policy. That is the true construction
which will best carry legislative intention into
effect.
(FOR PERSONS) Sec. IV of the Marriage law
provides that all marriages contracted outside
the islands, which would be valid by the laws of
the country in which the same were
contracted, are valid in these islands. To
establish
a
valid
foreign
marriage
pursuant to this comity provision, it is
first necessary to prove before the courts
ofthe Islands the existence of the foreign
law as a question of fact, and it is then
necessary to prove the alleged foreign
marriage by convincing evidence. A
Philippine marriage followed by 23 years of
uninterrupted marital life, should not be
impugned and discredited, after the death of
the husband through an alleged prior Chinese
marriage, save upon proof so clear, strong
and unequivocal as to produce a moral
conviction
of
the
existence
of
such
impediment. A marriage alleged to have been
contracted in China and proven mainly by a socalled matrimonial letter held not to be valid in
the Philippines.

CHING HUAT VS. CO HEONG


Facts:
It is alleged in the petition, that the said
minor is his legitimate daughter; that up to
June 21, 1946, said minor had been living with
and had under the custody of petitioner; that
respondent, taking advantage of his
confidential and spiritual relation with
Maria Ching as her godfather, persuaded
and induced her by means of trick, promises
and cajolery, to leave the parental home and to
elope with him in the night of June 21, 1946, to
Plaridel, Bulacan, where they were married on
the following day before the Justice of the
Peace of said municipality, said Maria Ching
being at the time 15 years old; and that ever
since respondent has had the minor Maria
Ching under his custody in Malolos, Bulacan,
and has restrained her at her liberty.
It is further alleged that respondent had
been previously married in China to Gue
Min, said marriage being said to be subsisting
at the time respondent married Maria Ching.
Petitioner further avers that Gue Min has never

Private International Law

been declared an absentee nor generally


considered as dead and believed to be so by
respondent at the time he married Maria Ching.
Respondent, in his answer, among other
things, asserts that on June 21, 1946, he and
Maria Ching alias Avelina Ching were legally
married before the Justice of the Peace of
Plaridel, Bulacan, and alleges that the essential
requisites for such marriage were complied
with.
Issue: whether or not petitioner still retains his
right to the custody of his minor daughter
Maria Ching alias Avelina Ching?
Ruling: the Philippine marriage between
said respondent and Maria Ching before
the Justice of the Peace of Plaridel,
Bulacan, is undisputed. It is also beyond
question that marriage was contracted by a
man much over 16 years old with a girl 15
years old (Act No. 3613, section 2), neither of
whom was included in any of the exceptions
mentioned in section 28 of the same Act; nor in
those stated in section 29 thereof for the
reason that the alleged prior Chinese marriage
has not been established.
If the supposed prior Chinese marriage had
been sufficiently proven, then in order that
the subsequent Philippine marriage could
be valid, it would have been necessary
either (a) that the Chinese marriage
should have been previously annulled or
dissolved: or (b) that the first wife of
respondent should have been absent for 7
consecutive years at the time of the
second marriage without the respondent
having news of the absentee being alive;
or (c) that the absentee should have been
generally
considered
as
dead
and
believed to be so by respondent at the
time of contracting the subsequent
marriage, in either of which last two
cases the subsequent marriage will be
valid until declared null and void by a
competent court, while in the first it will
be valid without this limitation. (Act No.
3613, section 29 [a] and [b]; section 30 [b].)
But as already adverted to, the complete
absence of proof of the supposed former
Chinese marriage makes sections 29 and 30 of
the Marriage Law inapplicable.
Maria Ching having been validly married
on
June
21,
1946,
she
became
emancipated on that same date (arts. 314
[1] and 315, Civil Code). This emancipation
brought about the loss by the father of the
parental authority that he claims. On the other
hand, by article 48 of Chapter V of the Spanish
Marriage Law of 1870, whose articles 44 to 78
were, and are now partly, in force in the
Philippines (Benedicto vs. De la Rama, 3 Phil.,
34), the wife has the duty, among others, of
living in her husband's company and of
following him to wherever he transfer his
domicile or residence. (Yaez de Barbuevo vs.
Fuster, 29 Phil., 606, 612.)

Page 37

PEOPLE V. DUMPO
62 Phil 247
Facts:
Moro Hassan and Mora Dupo have been
legally married according to the rites and
practice of the Mohammedan religion. Without
this marriage being dissolved, it is
alleged that Dumpo contracted another
marriage with Moro Sabdapal after which
the two lived together as husband and wife.
Dumpo was prosecuted for and convicted
of the crime of bigamy.
The accused interposed an appeal. It has been
established by the defense, without the
prosecution having presented any objection
nor evidence to the contrary, that the alleged
second marriage of the accused is null and
void according to Mohammedan rites on the
ground that her father had not given his
consent thereto.

witnesses. Jan Peng, a Chinaman of 52 years of


age, swore that he knew the forms of
ceremonies of marriage in China, Dee Tim also
presented a witness, Ty Cong Ting, a
Chinaman, 32 years of age and a lawyer, who
testified concerning the laws and customs in
China with reference to the forms of marriage
ceremony. He testified that he knew and was
well acquainted with the customs and practices
of Chinamen in China with reference to
marriages and the manner and form in which
they were celebrated, and the form of proof
issued for the purpose of proving that a
marriage ceremony had been performed. Mr.
Ty Cong Ting was, at the time he testified as a
witness, the legal attorney of the Chinese
Consul General in the City of Manila.
Issue:
Who is the legal wife?
Held:

Held:
Marriage among Moslems is a fact of
which no judicial notice may be taken and
must be subject to proof in every
particular case. In the case at bar we have
the uncontradicted testimony of Tahari, an
Imam or Mohammedan priest authorized to
solemnize marriages between Mohammedans,
to the effect that the consent of the bride's
father or in the absence thereof, that of the
chief of the tribe to which she belongs is an
indispensable requisite for the validity of such
contracts.
It is an essential element of the crime of
bigamy that the alleged second marriage,
having all the essential requisites, would
be valid were it not for the subsistence of the
first marriage. However, accuseds subsequent
marriage was void for lack of requisites
necessary under Moslem law, she must be
acquitted.

LAO AND LAO V. DEE TIM


45 Phil 739 (1924)

It is perhaps true that Yap Siong did on various


occasions, depending upon his interest and
convenience at the particular time, state that
Maria Lao was his querida and not his wife. It is
also perhaps true, for the same reason, that he
stated that Dee Tim was not his wife but his
querida. Evidently he was attempting to
keep the information, which he was quite
able to do, until he had passed to that
bourn from which none returns, and until
a distribution of his large accumulated
earnings
among
his
heirs
became
necessary.
Based on a preponderance of the
evidence the Court was convinced that
both Dee Tim and Maria Lao were legally
married to Yap Siong in good faith,
believing that each was his sole and separate
wife, living in absolute ignorance of the fact of
his double marriage. They were each
married in good faith and in ignorance of
the existence of the other marriage. Yap
Siong up to the time of his death seems to
have been successful in keeping each of his
two wives ignorant of the fact that he was
married to the other.

Facts:
Yap Siong died intestate. During the
distribution of his estate, Maria Lao and Jose
Lao appeared claiming to be the legitimate
spouse and son of the deceased. Maria claims
that they had been married in the Philippines
on June 24, 1903. On the other hand, Dee Tim
claims to be the legitimate widow of Yap Siong;
that she and Yap Siong were joined in holy
matrimony on the 14th day of September,
1893, in accordance with the laws of China.

Maria Lao presented marriage certificates as


proof. Dee Tim likewise presented a certificate
of marriage and that it was positive proof of
her marriage and that it complied with the
custom and practice in China with reference to
marriage
ceremonies.
To
support
her
contention she presented a number of

Private International Law

Under the Leyes de Partidas (Law 1, title


13, partida 4), where two women
innocently and in good faith are legally
united in holy matrimony to the same
man, their children born will be regarded
as legitimate children and each family will
be entitled to one-half of the estate of the
husband upon distribution of his estate.
That provision of the Leyes de Partidas is a
very humane and wise law. It justly protects
those who innocently have entered into the
solemn relation of marriage and their
descendants. The good faith of all the
parties will be presumed until the
contrary is positively proved. (Article 69,
Civil Code; Las Leyes de Matrimonio, section
96; Gaines vs. Hennen, 65 U.S., 553.)c
YAO KEE V. GONZALES
167 SCRA 736

Page 38

FACTS:

1.

2.

3.
4.

5.

6.

7.

Sy Kiat, a Chinese national, died in


Calooocan City where he was then
residing leaving behind real and
personal properties here in the
Philippines.
Private respondents (Aida Sy-Gonzales
et al.,) filed a petition for the grant of
letters or administration alleging that
they were the children of the deceased
with Asuncion Gillego.
Petition was opposed by herein
petitioners (Yao Kee et al.,) alleging
that they were the legitimate family.
The probate court found that Sy Kiat
was legally married to Yao Kee and
that their 3 offsprings were the
legitimate children.
The
court
likewise
ruled
that
respondents are the acknowledged
illegitimate offspring of Sy Kiat with
Asuncion Gillego.
On appeal, the lower courts decision
was set aside declaring petitioners as
the acknowledge natural children of Sy
Kiat and Asuncion Gillego.
Oppostiors
were
declared
the
acknowelged natural children of the
deceased since the legality of the
alleged marriage of Sy Kiat and Yao
Kee in China had not been proven to be
valid to the laws of China.

ISSUE:

custom on marriage not only because they


are self serving evidence, but more
importantly, there is no showing that
they are competent to testify on the
subject matter. For failure to prove the
foreign law or custom, and consequently,
the validity of the marriage in accordance
with said law or custom, the marriage
between Yao Kee and Sy Kiat cannot be
recognized in this jurisdiction.
However, as petitioners failed to establish
the marriage of Yao Kee with Sy Kiat
according to the laws of China, they cannot
be accorded the status of legitimate
children but only that of acknowledged
natural children. Petitioners are natural
children, it appearing that at the time of
their conception Yao Kee and Sy Kiat were
not disqualified by any impediment to
marry one another. [See Art. 269, Civil
Code] And they are acknowledged children
of the deceased because of Sy Kiats
recognition of Sze Sook Wah and its
extension to Sze Lai Cho and Sy Chun Yen
who are her sisters of the full blood.
Private respondents on the other hand are
also the deceaseds acknowledged natural
children with Asuncion Gillego , a Filipina
with whom he lived for 25 years without
the benefit of marriage. They have in their
favor their fathers acknowledgment,
evidence by a compromise agreement
entered into by and between their parents
and approved by the CFI wherein Sy Kiat
not only acknowledged them as his
children by Asuncion Gillego but likewise
made provisions for their support and
future inheritance.

Was the fact of marriage of Sy Kiat and Yao


Kee in China proven as a custom?

REPUBLIC V. ORBECIDO
GR NO. 154380, October 5, 2005

HELD:

Custom is defined as a rule of


conduct formed by repetition of acts,
uniformly observed (practiced) as a
social
rule,
legally
binding
and
obligatory. The law requires that a
custom must be proved as a fact,
according to the rules of evidence.
[Article 12, Civil Code] On this score the
Court had occasion to state that a local
custom as a source of right cannot be
considered by a court of justice unless such
custom
is
properly
established
by
competent evidence like any other fact.
The same evidence, if not one of a higher
degree, should be required of a foreign
custom.
Construing this provision of law the Court
has held that to establish a valid foreign
marriage two things must be proven,
namely 1) the existence of the foreign
law as a question of fact; and 2) the
alleged
foreign
marriage
by
convincing evidence.
In the case at bar petitioners did not
present
any
competent
evidence
relative to the law and custom of
China on marriage. The testimonies of
Yao and Gan Ching (brother) cannot be
considered as proof of Chinas law or

Private International Law

Facts:
On May 24, 1981, Cipriano Orbecido III
married Lady Myros M. Villanueva in the
Philippines in Lam-an, Ozamis City. Their
marriage was blessed with a son and a
daughter.
In 1986, Ciprianos wife left for the United
States
bringing
along
their
son
Kristoffer. A few years later, Cipriano
discovered that his wife had been naturalized
as an American citizen.
Sometime in 2000, Cipriano learned from his
son that his wife had obtained a divorce
decree and then married a certain Innocent
Stanley. She, Stanley and her child by him
currently live in San Gabriel, California.
Cipriano thereafter filed with the trial
court a petition for authority to remarry
invoking Paragraph 2 of Article 26 of the
Family Code. No opposition was filed. Finding
merit in the petition, the court granted the
same.
The Republic, herein petitioner,
through the Office of the Solicitor General
(OSG), sought reconsideration but it was
denied.

Page 39

The OSG contends that Paragraph 2 of Article


26 of the Family Code is not applicable to
the instant case because it only applies to
a valid mixed marriage; that is, a marriage
celebrated between a Filipino citizen and an
alien. Furthermore, the OSG argues there is no
law that governs respondents situation. The
OSG posits that this is a matter of legislation
and not of judicial determination.

and compelling reasons for the


exemption.
However,
such
exemption shall not apply if the
same is not compatible with
the solidarity of the family.
(110a)
Art. 70.

The
spouses
are
jointly
responsible for the support of
the family. The expenses for
such
support
and
other
conjugal obligations shall be
paid from the community
property and, in the absence
thereof, from the income or
fruits
of
their
separate
properties.
In
case
of
insufficiency or absence of said
income
or
fruits,
such
obligations shall be satisfied
from the separate properties.
(111a)

Art. 73.

Either spouse may exercise any


legitimate
profession,
occupation, business or activity
without the consent of the
other. The latter may object
only on valid, serious, and
moral grounds.

Held:
Taking into consideration the legislative intent
and applying the rule of reason, we hold that
Paragraph 2 of Article 26 should be
interpreted to include cases involving
parties who, at the time of the
celebration of the marriage were Filipino
citizens, but later on, one of them becomes
naturalized as a foreign citizen and obtains a
divorce decree. The Filipino spouse should
likewise be allowed to remarry as if the
other party were a foreigner at the time
of the solemnization of the marriage. To
rule otherwise would be to sanction absurdity
and injustice. Where the interpretation of a
statute according to its exact and literal import
would lead to mischievous results or
contravene the clear purpose of the legislature,
it should be construed according to its spirit
and reason, disregarding as far as necessary
the letter of the law. A statute may therefore
be extended to cases not within the literal
meaning of its terms, so long as they come
within its spirit or intent.

In case of disagreement, the


court shall decide whether or not:
1.

In view of the foregoing, we state the twin


elements for the application of Paragraph 2 of
Article 26 as follows:

2.

1. There is a valid marriage that has


been celebrated between a Filipino
citizen and a foreigner; and
2. A valid divorce is obtained abroad
by the alien spouse capacitating
him or her to remarry.
The reckoning point is not the citizenship
of the parties at the time of the
celebration of the marriage, but their
citizenship at the time a valid divorce is
obtained abroad by the alien spouse
capacitating the latter to remarry.
VI. CONSEQUENCES OF MARRIAGE
1.

PERSONAL RELATIONS

The foregoing provisions shall


not prejudice the rights of
creditors who acted in good
faith. (117a)
Muslim Code

Art. 34.

Mutual rights and obligations.

1. The husband and the wife

Civil Code Art. 15, supra


Family Code
Art. 69.

The husband and wife shall fix


the family domicile. In case of
disagreement, the court shall
decide.
The court may exempt one
spouse from living with the
other if the latter should live
abroad or there are other valid

Private International Law

The objection is proper;


and
Benefit has occurred to the
family prior to the objection
or thereafter. If the benefit
accrued
prior
to
the
objection,
the
resulting
obligation shall be enforced
against
the
separate
property of the spouse who
has not obtained consent.

2.

are obliged to live together,


observe mutual respect
and fidelity, and render
mutual help and support in
accordance with this Code.
When one of the spouses
neglects his or her duties
to the conjugal union or
brings danger, dishonor or
material injury upon the
other, the injured party
may petition the court for
relief.
The
court
may

Page 40

3.

4.

Art. 35.

counsel the offender to


comply with his or her
duties, and take such
measures
as
may
be
proper.
The husband and the wife
shall inherit from each
other in accordance with
this Code.
The husband and the wife
shall have the right to
divorce in accordance with
this Code.

6. The wife shall be entitled to an


equal and just treatment by the
husband.
2. PROPERTY RELATIONS
Civil Code:
Art. 15, supra
Art. 117.

Rights and obligations of the


husband. The husband shall
fix the residence of the family.
The court may exempt the wife
from living with her husband on
any of the following grounds:

1.

His income is
sufficient for the family,
according to its social
standing, and
2.His
opposition
is
founded on serious and
valid grounds.

a. Her

dower
is
not
satisfied in accordance with
the stipulations; or
b. The conjugal dwelling
is not in keeping with her
social standing or is, for
any reason, not safe for the
members of the family or
her property.

In case of disagreement on this


question, the parents and
grandparents as well as the
family council, if any, shall be
consulted. If no agreement is
still arrived at, the court will
decide
whatever
may
be
proper and in the best interest
of the family. (n)

Art. 36. Rights and obligations of the wife.


1. The wife shall dutifully manage
the affairs of the household.
She may purchase things
necessary for the maintenance
of the family, and the husband
shall be bound to reimburse
the expenses, if he has not
delivered the proper sum.
2. The wife cannot, without the
husband's consent, acquire any
property by gratuitous title,
except from her relatives who
are
within
the
prohibited
degrees in marriage.
3. The wife may, with her
husband's consent, exercise
any profession or occupation or
engage in lawful business
which is in keeping with Islamic
modesty and virtue. However,
if the husband refuses to give
his consent on the ground that
his income is sufficient for the
family according to its social
standing or his opposition is
based on serious and valid
grounds, the matter shall be
referred
to
the
Agama
Arbitration Council.
4. The wife shall have the right to
demand the satisfaction of her
mahr.
5. Unless otherwise stipulated in
the marriage settlements, the
wife retain ownership and
administration of her exclusive
property.

Private International Law

The wife may exercise any


profession or occupation or
engage in business. However,
the husband may object,
provided:

Art. 118.

The property relations between


husband and wife shall be
governed in the following
order:
a. By
contract
executed
before the marriage;
b. By the provisions of this
Code; and
c. By custom. (1315a)

Art. 124.

If the marriage is between a


citizen of the Philippines and a
foreigner, whether celebrated
in the Philippines or abroad,
the following rules shall prevail:

1. If the husband is a citizen


of the Philippines while the
wife is a foreigner, the
provisions of this Code
shall govern their relations;
2. If the husband
is a
foreigner and the wife is a
citizen of the Philippines,
the laws of the husband's
country shall be followed,
without prejudice to the
provisions of this Code with
regard
to
immovable
property. (1325a)
Art. 144.

When a man and a woman live


together as husband and wife,
but they are not married, or
their marriage is void from the
beginning,
the
property

Page 41

acquired by either or both of


them through their work or
industry or their wages and
salaries shall be governed by
the rules on co-ownership. (n)

2.

Family Code:
Art. 74.

Art. 75.

Art. 76.

Art. 77.

3.
A marriage in articulo mortis
may also be solemnized by the
captain of a ship or chief of an
airplane during a voyage, or by
the commanding officer of a
military unit, in the absence of
a chaplain, during war. The
duties mentioned in the two
preceding articles shall be
complied with by the ship
captain, airplane chief or
commanding officer. (n)
Marriages
between
Filipino
citizens
abroad
may
be
solemnized by consuls and
vice-consuls of the Republic of
the Philippines. The duties of
the local civil registrar and of a
judge or justice of the peace or
mayor with regard to the
celebration of marriage shall be
performed by such consuls and
vice-consuls. (n)
No marriage license shall be
necessary when a man and a
woman who have attained the
age of majority and who, being
unmarried, have lived together
as husband and wife for at
least five years, desire to marry
each other. The contracting
parties
shall
state
the
foregoing facts in an affidavit
before any person authorized
by law to administer oaths. The
official, priest or minister who
solemnized the marriage shall
also state in an affidavit that
he took steps to ascertain the
ages and other qualifications of
the contracting parties and that
he found no legal impediment
to the marriage. (n)
In case two persons married in
accordance with law desire to
ratify their union in conformity
with the regulations, rites, or
practices of any church, sect,
or religion it shall no longer be
necessary to comply with the
requirements of Chapter 1 of
this Title and any ratification
made
shall
merely
be
considered as a purely religious
ceremony. (23)

Art. 80.
The
following
marriages shall be void from the beginning:
1. Those contracted under the
ages
of
sixteen
and
fourteen years by the male

Private International Law

4.
5.
6.

7.

Art. 147.

and female respectively,


even with the consent of
the parents;
Those solemnized by any
person
not
legally
authorized
to
perform
marriages;
Those solemnized without
a marriage license, save
marriages of exceptional
character;
Bigamous or polygamous
marriages not falling under
Article 83, Number 2;
Incestuous
marriages
mentioned in Article 81;
Those where one or both
contracting parties have
been found guilty of the
killing of the spouse of
either of them;
Those
between
stepbrothers
and
stepsisters
and
other
marriages
specified
in
Article 82. (n)

When a man and a woman who


are capacitated to marry each
other, live exclusively with
each other as husband and
wife without the benefit of
marriage or under a void
marriage, their wages and
salaries shall be owned by
them in equal shares and the
property acquired by both of
them through their work or
industry shall be governed by
the rules on co-ownership.
In the absence of proof to the
contrary, properties acquired
while they lived together shall
be presumed to have been
obtained by their joint efforts,
work or industry, and shall be
owned by them in equal
shares. For purposes of this
Article, a party who did not
participate in the acquisition by
the other party of any property
shall be deemed to have
contributed
jointly
in
the
acquisition
thereof
if
the
former's efforts consisted in
the care and maintenance of
the
family
and
of
the
household.
Neither party can encumber or
dispose by acts inter vivos of
his or her share in the property
acquired during cohabitation
and owned in common, without
the consent of the other, until
after the termination of their
cohabitation.
When only one of the parties to
a void marriage is in good faith,
the share of the party in bad

Page 42

faith in the co-ownership shall


be forfeited in favor of their
common children. In case of
default of or waiver by any or
all of the common children or
their descendants, each vacant
share shall belong to the
respective
surviving
descendants. In the absence of
descendants, such share shall
belong to the innocent party. In
all cases, the forfeiture shall
take place upon termination of
the cohabitation. (144a)

contract, shall be governed by


the
regime
of
complete
separation
of
property
in
accordance with this Code and,
in a suppletory manner, by the
general principles of Islamic
law and the Civil Code of the
Philippines.
VII. Dissolution of Marriage
1.

Annulment

Civil Code:
Art. 148.

In cases of cohabitation not


falling under the preceding
Article, only the properties
acquired by both of the parties
through their actual joint
contribution
of
money,
property, or industry shall be
owned by them in common in
proportion to their respective
contributions. In the absence of
proof to the contrary, their
contributions
and
corresponding
shares
are
presumed to be equal. The
same rule and presumption
shall apply to joint deposits of
money and evidences of credit.
If one of the parties is validly
married to another, his or her
share in the co-ownership shall
accrue
to
the
absolute
community
or
conjugal
partnership existing in such
valid marriage. If the party who
acted in bad faith is not validly
married to another, his or her
shall be forfeited in the manner
provided in the last paragraph
of the preceding Article.
The
foregoing
rules
on
forfeiture shall likewise apply
even if both parties are in bad
faith. (144a)

Art. 66.

When either or both of the


contracting parties are citizens
or subjects of a foreign
country, it shall be necessary,
before a marriage license can
be
obtained,
to
provide
themselves with a certificate of
legal capacity to contract
marriage, to be issued by their
respective
diplomatic
or
consular officials. (13a)

Art. 71.

All
marriages
performed
outside the Philippines in
accordance with the laws in
force in the country where they
were performed, and valid
there as such, shall also be
valid in this country, except
bigamous,
polygamous,
or
incestuous
marriages
as
determined by Philippine law.
(19a)

Family Code
Art. 45.

A marriage may be annulled for


any of the following causes,
existing at the time of the
marriage:
1.

Muslim Code:
Art. 37.

How governed. The property


relations between husband and
wife shall be governed in the
following order:

a. By contract before or at the


b.
c.

time of the celebration of


marriage;
By the provisions of this
Code; and
By custom.
2.

Art. 38.

Regime of property relations.

The property
relations
between the spouses, in the
absence of any stipulation to
the contrary in the marriage
settlements
or
any
other

Private International Law

3.

That the party in whose


behalf it is sought to have
the marriage annulled was
eighteen years of age or
over but below twenty-one,
and the marriage was
solemnized without the
consent of the parents,
guardian or person having
substitute
parental
authority over the party, in
that order, unless after
attaining
the
age
of
twenty-one,
such
party
freely cohabited with the
other
and
both
lived
together as husband and
wife;
That either party was of
unsound mind, unless such
party after coming to
reason, freely cohabited
with the other as husband
and wife;
That the consent of either
party was obtained by

Page 43

4.

5.

6.

Art. 46.

fraud, unless such party


afterwards,
with
full
knowledge of the facts
constituting
the
fraud,
freely cohabited with the
other as husband and wife;
That the consent of either
party was obtained by
force, intimidation or undue
influence, unless the same
having
disappeared
or
ceased,
such
party
thereafter freely cohabited
with the other as husband
and wife;
That either party was
physically
incapable
of
consummating
the
marriage with the other,
and
such
incapacity
continues and appears to
be incurable; or
That either party was
afflicted with a sexuallytransmissible disease found
to be serious and appears
to be incurable. (85a)

3.

4.

Any
of
the
following
circumstances shall constitute
fraud referred to in Number 3
of the preceding Article:
1.

2.

3.

4.

Non-disclosure
of
a
previous conviction by final
judgment of the other party
of a crime involving moral
turpitude;
Concealment by the wife of
the fact that at the time of
the marriage, she was
pregnant by a man other
than her husband;
Concealment of sexually
transmissible
disease,
regardless of its nature,
existing at the time of the
marriage; or
Concealment
of
drug
addiction,
habitual
alcoholism
or
homosexuality
or
lesbianism existing at the
time of the marriage.

No other misrepresentation or
deceit as to character, health,
rank, fortune or chastity shall
constitute such fraud as will
give grounds for action for the
annulment of marriage. (86a)
Art. 47.

2.

The action for annulment of


marriage must be filed by the
following persons and within
the periods indicated herein:
1.

For causes mentioned in


number 1 of Article 45 by
the party whose parent or
guardian did not give his or
her consent, within five
years after attaining the

Private International Law

5.

Art. 48.

age of twenty-one, or by
the parent or guardian or
person having legal charge
of the minor, at any time
before such party has
reached the age of twentyone;
For causes mentioned in
number 2 of Article 45, by
the same spouse, who had
no
knowledge
of
the
other's insanity; or by any
relative or guardian or
person having legal charge
of the insane, at any time
before the death of either
party, or by the insane
spouse during a lucid
interval or after regaining
sanity;
For causes mentioned in
number 3 of Article 45, by
the injured party, within
five
years
after
the
discovery of the fraud;
For causes mentioned in
number 4 of Article 45, by
the injured party, within
five years from the time
the force, intimidation or
undue
influence
disappeared or ceased;
For causes mentioned in
number 5 and 6 of Article
45, by the injured party,
within five years after the
marriage. (87a)

In all cases of annulment or


declaration of absolute nullity
of marriage, the Court shall
order the prosecuting attorney
or fiscal assigned to it to
appear on behalf of the State
to take steps to prevent
collusion between the parties
and to take care that evidence
is not fabricated or suppressed.
In the cases referred to in the
preceding
paragraph,
no
judgment shall be based upon
a stipulation of facts or
confession of judgment. (88a)

Art. 49.

During the pendency of the


action and in the absence of
adequate
provisions
in
a
written agreement between the
spouses,
the
Court
shall
provide for the support of the
spouses and the custody and
support
of
their
common
children. The Court shall give
paramount consideration to the
moral and material welfare of
said children and their choice
of the parent with whom they
wish to remain as provided to
in Title IX. It shall also provide
for appropriate visitation rights
of the other parent. (n)

Page 44

Art. 50.

The effects provided for by


paragraphs (2), (3), (4) and (5)
of Article 43 and by Article 44
shall also apply in the proper
cases to marriages which are
declared ab initio or annulled
by
final
judgment
under
Articles 40 and 45.
The final judgment in such
cases shall provide for the
liquidation,
partition
and
distribution of the properties of
the spouses, the custody and
support
of
the
common
children, and the delivery of
third presumptive legitimes,
unless such matters had been
adjudicated in previous judicial
proceedings.
All creditors of the spouses as
well as of the absolute
community or the conjugal
partnership shall be notified of
the proceedings for liquidation.
In the partition, the conjugal
dwelling and the lot on which it
is situated, shall be adjudicated
in
accordance
with
the
provisions of Articles 102 and
129.

Art. 51.

In said partition, the value of


the presumptive legitimes of all
common children, computed as
of the date of the final
judgment of the trial court,
shall be delivered in cash,
property or sound securities,
unless the parties, by mutual
agreement judicially approved,
had already provided for such
matters.

the appropriate civil registry


and registries of property;
otherwise, the same shall not
affect third persons. (n)
Art. 53.

Either of the former spouses


may
marry
again
after
compliance
with
the
requirements
of
the
immediately preceding Article;
otherwise,
the
subsequent
marriage shall be null and
void.chan robles virtual law
library

Art. 54.

Children conceived or born


before
the
judgment
of
annulment or absolute nullity
of the marriage under Article
36 has become final and
executory shall be considered
legitimate. Children conceived
or born of the subsequent
marriage under Article 53 shall
likewise be legitimate.

Art. 55.
A petition for legal
separation may be filed on any of the following
grounds:
1.

2.

3.

The children or their guardian


or the trustee of their property
may ask for the enforcement of
the judgment.
4.
The
delivery
of
the
presumptive legitimes herein
prescribed shall in no way
prejudice
the
ultimate
successional rights of the
children accruing upon the
death of either of both of the
parents; but the value of the
properties already received
under the decree of annulment
or absolute nullity shall be
considered as advances on
their legitime. (n)
Art. 52.

The judgment of annulment or


of absolute nullity of the
marriage, the partition and
distribution of the properties of
the spouses and the delivery of
the
children's
presumptive
legitimes shall be recorded in

Private International Law

5.
6.
7.

8.

Repeated
physical
violence
or
grossly
abusive
conduct
directed against the
petitioner, a common
child, or a child of the
petitioner;
Physical violence or
moral
pressure
to
compel the petitioner
to change religious or
political affiliation;
Attempt of respondent
to corrupt or induce the
petitioner, a common
child, or a child of the
petitioner, to engage in
prostitution,
or
connivance
in
such
corruption
or
inducement;
Final
judgment
sentencing
the
respondent
to
imprisonment of more
than six years, even if
pardoned;
Drug
addiction
or
habitual alcoholism of
the respondent;
Lesbianism
or
homosexuality of the
respondent;
Contracting
by
the
respondent
of
a
subsequent bigamous
marriage, whether in
the
Philippines
or
abroad;
Sexual
infidelity
or
perversion;

Page 45

9.

Attempt
by
the
respondent against the
life of the petitioner; or
10. Abandonment
of
petitioner
by
respondent
without
justifiable cause for
more than one year.
For purposes of this Article, the
term "child" shall include a
child by nature or by adoption.
(9a)
2.

Absolute Divorce

ROEHR VS. RODRIGUEZ


Facts:
Petitioner Wolfgang Roehr, a German citizen,
married a Filipina, Carmen Rodriguez in
Germany. The marriage was ratified in
Tayasan, Negros Oriental. Private respondent
filed a petition for the declaration of nullity of
marriage before the RTC of Makati. Petitioner
filed a motion to dismiss but was denied by the
trial court. The petitioner obtained a decree of
divorce from the Court of First Instance of
Hamburg Blankenese and granting the
custody of the children to the father.

Civil Code:

Issue:

Art. 15, supra


Art. 17, Supra

Whether or not the legal effects of a divorce


obtained from a foreign country such as
support and custody of the children can be
determined in our courts?

Muslim Code:

Held:

45-55 supra

Yes. In order to take effect, a foreign


judgement must clearly show that the
opposing party has been given ample
opportunity to do so under the Rules of
Civil Procedure. Accordingly, the respondent
was not given the opportunity to challenge the
judgement of the German Court, therefore,
legal effects of divorce must be determined in
our courts. The court held that the trial court
has jurisdiction over the issue between the
parties as to who has the parental custody.

Art. 27. By a husband. Notwithstanding the


rule of Islamic law permitting a
Muslim to have more than one
wife but one wife unless he can
deal with them with equal
companionship
and
just
treatment as enjoined by
Islamic law and only in
exceptional cases.
Art. 34.

Mutual rights and obligations.

RECTO VS. HARDEN (supra)

1. The husband and the wife

2.

3.

4.

are obliged to live together,


observe mutual respect
and fidelity, and render
mutual help and support in
accordance with this Code.
When one of the spouses
neglects his or her duties
to the conjugal union or
brings danger, dishonor or
material injury upon the
other, the injured party
may petition the court for
relief.
The
court
may
counsel the offender to
comply with his or her
duties, and take such
measures
as
may
be
proper. .chan robles virtual
law library
The husband and the wife
shall inherit from each
other in accordance with
this Code.
The husband and the
wife shall have the right
to divorce in accordance
with this Code.

Private International Law

GONZALES VS. GONZALES

Facts:
Plaintiff and defendant are citizens of the
Philippine Islands and at present residents of
the City of Manila. They were married in the
City of Manila on January 19, 1919, and lived
together as man and wife in the Philippine
Islands until the spring of 1926. They
voluntarily separated and since that time have
not lived together as man and wife. Of this
union four children were born. Negotiations
between the parties, both being represented
by attorneys, whereupon it was mutually
agreed to allow the plaintiff for her support and
that of her children, five hundred pesos (P500)
monthly; this amount to be increased in case of
illness or necessity, and the title of certain
properties to be put in her name. Shortly after
this agreement the husband left the Islands,
betook himself to Reno, Nevada, and secured
in that jurisdiction an absolute divorce on the
ground of desertion. Shortly thereafter the
defendant moved to California and returned to
these Islands in August 1928, where he has
since remained. On the same date that he
secured a divorce in Nevada he went through
the forms of marriage with another citizen of

Page 46

these Islands and now has three children as a


result of that marriage. Defendant, after his
departure from these Islands, reduced the
amount he had agreed to pay monthly for the
support of his wife and four minor children and
has not made the payments fixed in the Reno
divorce as alimony.
Issue:
Whether or not that the divorced acquired in
Nevada is valid here in the Philippines?
Held
While the parties in this action are in dispute
over financial matters they are in unity in
trying to secure the courts of this jurisdiction to
recognize and approve of the Reno divorce. On
the record here presented this can not be
done. The public policy in this jurisdiction on
the question of divorce is clearly set forth in
Act No. 2710, and the decisions of this court.

jurisdiction, except it be for a cause, and under


conditions for which the courts of Philippine
Islands would grant a divorce. The lower court
in granting relief as prayed for frankly stated
that the securing of the divorce, the
contracting of another marriage and the
bringing into the world of innocent children
brings about such a condition that the court
must grant relief. The hardships of the existing
divorce laws of the Philippine Islands are well
known to the members of the Legislature. It is
of no moment in this litigation what he
personal views of the writer on the subject of
divorce may be. It is the duty of the courts to
enforce the laws of divorce as written by the
Legislature if they are constitutional. Courts
have no right to say that such laws are too
strict or too liberal.
TENCHAVEZ V. ESCANO 15 SCRA 355
Facts:

The entire conduct of the parties from the time


of their separation until the case was
submitted to this court, in which they all
prayed that the Reno divorce be ratified and
confirmed, clearly indicates a purpose to
circumvent the laws of the Philippine Islands
regarding divorce and to secure for themselves
a change of status for reasons and under
conditions not authorized by our law. At all
times the matrimonial domicile of this couple
has been within the Philippine Islands and the
residence acquired in the State of Nevada by
the husband of the purpose of securing a
divorce was not a bona fide residence and did
not confer jurisdiction upon the Court of that
State to dissolve the bonds if matrimony in
which he had entered in 1919. While the
decisions of this court heretofore in refusing to
recognize the validity of foreign divorce has
usually been expressed in the negative and
have been based upon lack of matrimonial
domicile or fraud or collusion, we have not
overlooked the provisions of the Civil Code now
in force in these Islands. Article 9 thereof reads
as follows:
The laws relating to family rights
and duties, or to the status,
condition and legal capacity or
persons,
are
binding
upon
Spaniards even though they reside
in a foreign country.
And article 11, the last part of which reads:
. . . the prohibitive laws concerning
persons, their acts and their
property, and those intended to
promote public order and good
morals, shall nor be rendered
without effect by any foreign laws
or judgments or by anything done
or any agreements entered into a
foreign country.
It is therefore a serious question whether any
foreign divorce relating to citizens of the
Philippine Islands, will be recognized in this

Private International Law

Pastor Tenchavez married Vicenta Escano


on Feb. 24, 1948 in Cebu City. As of June
1948, the newly-weds were already
estranged. Vicenta left for the US and filed
a verified complaint for divorce against the
Pastor in the State of Nevada on the
ground of "extreme cruelty, entirely mental
in character."
A decree of divorce was issued by the
Nevada Court. Later on, Vicenta married an
American, Russell Leo Moran in Nevada.
She now lives with him in California and
has begotten children. She acquired
American citizenship on August 8, 1958. On
July 30, 1955, Pastor filed a complaint for
legal separation and damages against
Vicenta and her parents in the CFI-Cebu.

HELD:

At the time the divorce decree was


issued, Vicenta, like her husband, was
still a Filipino citizen. She was then
subject to Philippine laws under Art.
15 of the New Civil Code. Philippine law,
under the NCC then now in force, does not
admit absolute divorce but only provides
for legal separation.

For Philippine courts to recognize


foreign
divorce
decrees
between
Filipino citizens would be a patent
violation of the declared policy of the
State, especially in view of the 3rd
par. of Art. 17, of the New Civil Code
which
reads:
Prohibitive
laws
concerning persons, their acts or
property, and those which have, for
their object, public order, public policy
and good customs shall not be
rendered ineffective by laws or
judgments
promulgated,
or
by
determinations or conventions agreed
upon in a foreign country. Moreover,
recognition would give rise to scandalous
discrimination in favor of wealthy citizens
to the detriment of those members of our
society whose means do not permit them
to sojourn abroad and obtain absolute
divorce outside the Philippines.

Page 47

Therefore, a foreign divorce between


Filipino citizens, sought and decreed after
the effectivity of the NCC, is not entitled to
recognition as valid in this jurisdiction.

SIKAT V. CANSON
67 PHIL 207

Hilaria Sikat and John Canson contracted


marriage and lived together as husband
and wife until 1911 when they separated.
During the same year the wife commenced
divorce proceedings against her spouses,
but the case was dismissed without the
court passing upon the merits thereof.
At the time of their marriage in 1904, John
Canson was an Italian citizen but on
February 27, 1922, he became a
naturalized Filipino citizen.
In 1929, he went to Reno, Nevada, United
States of America, and on October 8, of
that year, he obtained an absolute decree
of divorce on the ground of desertion.
Hilaria, in 1933, filed another action, civil
case No. 5398 of the Court of First Instance
of Rizal, wherein she sought to compel the
defendant to pay her a monthly pension of
P500 as alimony or support.
Canson interposed three defenses: (1)
adultery on the part of the plaintiff: (2)
absolute divorce obtained by the defendant
as decreed by the court in Reno, Nevada,
United States of America; and (3) that the
defendant did not have the means to pay
the allowance sought. The lower court
dismissed the complaint and declined to
accord validity to the divorce obtained in
Reno but found that Hilaria Sikat had
forfeited her right to support because she
had committed adultery.
This judgment was not appealed and it
became final.
On June 1, 1934, the present action was
instituted by the plaintiff-appellant to
obtain the liquidation of the conjugal
partnership. The action is predicated on
the existence of a final decree of absolute
divorce rendered by the court of Reno,
Nevada, since 1929.

Held:

It is not, however, the citizenship of


the plaintiff for divorce which confers
jurisdiction upon a court, but his legal
residence within the State (Cousins Hix
vs. Fluemer, 55 Phil. 851). And assuming
that John Canson acquired legal residence
in the State of Nevada through the
approval of his citizenship papers, this did
not confer jurisdiction on the Nevada court
to grant a divorce that would be valid in
this jurisdiction nor jurisdiction that could
determine
their
matrimonial
status,
because the wife was still domiciled in the
Philippines. The Nevada court never
acquired jurisdiction over her person.
(Gorayeb vs. Hashim, 50 Phil. 26, and
Cousins Hix vs. Fluemer, supra.) This was
not a proceeding in rem to justify a
court in entering a decree as to the
res or marriage relation entitled to be

Private International Law

enforced outside of the territorial


jurisdiction of the court.
Plaintiff-appellant had made her choice of
two inconsistent remedies afforded her by
law: (1) to impugn the divorce and file
an action for support, or (2) uphold
the validity of the divorce and sue for
a liquidation of conjugal partnership.
She chose the first remedy when she
filed her action for support. She lost the
case and should take the consequences.
The courts in the Philippines can grant
a divorce only on the ground of
"adultery on the part of the wife or
concubinage on the part of the
husband" as provided for under
section 1 of Act No. 2710. The divorce
decree in question was granted on the
ground of desertion, clearly not a cause for
divorce under our laws.
That our divorce law, Act No. 2710, is too
strict or too liberal is not for this court to
decide. (Barretto Gonzalez vs. Gonzalez,
supra.)
The allotment of powers between the
different
governmental
agencies
restricts the judiciary within the
confines of interpretation, not of
legislation. The legislative policy on the
matter of divorce in this jurisdiction is
clearly set forth in Act No. 2710 and has
been upheld by this court;
ARCA V. JAVIER
95 PHIL 579

Dissatisfied with the decision of the


Court of First Instance of Cavite ordering
him to give a monthly allowance of P60 to
plaintiffs beginning March 31, 1953, and to
pay them attorney's fees in the amount of
P150 defendant took the case directly to
this Court attributing five errors to the
court below. The facts are not disputed.

Javier and Arca got married in Manila.


Javier, an enlisted US Navy personnel left
for the States 7 years after the birth of
their first born. At such time, Arca lived
with Javiers parents. However, due to
strained relations with the latter, she left
and
transferred
to
her
hometown.
Thereafter, Javier filed a case for divorce in
Alabama
against
Arca
alleging
abandonment. Arca made her reply
claiming among other things that she
never abandoned her husband and that
their separation was due to a physical
impossibility which justifies her separation
if the husband moves to ultra-marine
colonies. The divorce however was
granted.
Javier got married the 2nd time but was
subsequently divorced. After a few years,
he went back to the Philippines, and
believing that the first two divorces were
valid, remarried the 3rd time.

Issue:
Did the Circuit Court of Mobile County acquire
jurisdiction of both spouses and effectively

Page 48

rendered a judgment in rem when it granted


divorce to Javier?
Held:

No, one of the essential conditions for


the validity of a decree of divorce is
that the court must have jurisdiction over
the subject matter and in order that this
may be acquired, plaintiff must be
domiciled in good faith in the State in
which it is granted (Cousins Hix vs.
Fluemer, 55 Phil., 851, 856).
While it is true that Salud R. Arca filed an
answer in the divorce case instituted at the
Mobile County in view of the summons
served upon her in this jurisdiction, but this
action cannot be interpreted as placing her
under the jurisdiction of the court because
its only purpose was to impugn the
claim of appellant that his domicile or
legal residence at that time was
Mobile County, and to show that the
ground of desertion imputed to her was
baseless and false. Such answer should
be considered as a special appearance
the purpose of which is to impugn the
jurisdiction of the court over the case.
It is established by the great weight
of authority that the court of a
country in which neither of the
spouses is domiciled and to which one
or both of them may resort merely for
the purpose of obtaining a divorce has
no jurisdiction to determine their
matrimonial status; and a divorce
granted by such a court is not entitled to
recognition elsewhere. (See Note to
Succession of Benton, 59 L. R. A., 143) The
voluntary appearance of the defendant
before such a tribunal does not invest the
court with jurisdiction. (Andrews vs.
Andrews, 188 U. S., 14; 47 L. ed.,
366.)
It follows that, to give a court
jurisdiction on the ground of the
plaintiff's residence in the State or
country of the judicial forum, his
residence must be bona fide. If a
spouse leaves the family domicile and goes
to another State for the sole purpose of
obtaining a divorce, and with no intention
of remaining, his residence there is not
sufficient to confer jurisdiction on the
courts of the State. (Ramirez vs. Gmur, 82
Phil., 855.)
But even if his residence had been taken
up is good faith, and the court had
acquired jurisdiction to take cognizance of
the divorce suit, the decree issued in his
favor is not binding upon the appellant; for
the matrimonial domicile of the spouses
being the City of Manila;
Ramirez v. Gmur
42 Phil 855

Leona Castro was the natural daughter of


decedent Samuel Bischoff. Whereas Ana
Ramirez was the latter's widow to which they
had no children. Leona was married to
Kauffman. They had 3 children, Elena,

Private International Law

Federico, and Ernesto. Later, Kauffman brought


Leona to Switzerland to recuperate her health.
A few years later, she fell for a Dr. Mory to
whom she had a child, Leontina. She informed
Kauffman that she no longer wished to stay
with him to which the latter obtained a divorce
in France where Leona was in default. Leona,
after the divorce got married in London, and
after which bore two children from which the
last childbirth caused Leona her life. The heirs
of Leona from the first marriage and the
second now claims the Estate of Samuel to
which Ana Ramirez opposed since Leona is not
a recognized natural child.
Held:
The status of Leona Castro as recognized
natural daughter of Samuel Bischoff is fully and
satisfactorily shown.
With reference to the right of the von
Kauffman children, it is enough to say
that they are legitimate children, born to
their parents in lawful wedlock; and they
are therefore entitled to participate in the
inheritance which would have devolved upon
their mother, if she had survived the testator.
The Court is of the opinion that the
decree of divorce upon which reliance is
placed by the representation of the Mory
children cannot be recognized as valid in
the courts of the Philippine Islands. The
French tribunal has no jurisdiction to entertain
an action for the dissolution of a marriage
contracted in these Islands by person
domiciled
here,
such
marriage
being
indissoluble under the laws then prevailing in
this country.
The evidence shows that the decree was
entered against the defendant in default, for
failure to answer, and there is nothing to show
that she had acquired, or had attempted to
acquire, a permanent domicile in the City of
Paris. It is evident of course that the presence
of both the spouses in that city was due merely
to the mutual desire to procure a divorce from
each other.
It is established by the great weight of
authority that the court of a country in
which neither of the spouses is domiciled
and to which one or both of them may
resort merely for the purpose of obtaining
a divorce has no jurisdiction to determine
their matrimonial status; and a divorce
granted by such a court is not entitled to
recognition elsewhere.
It follows that, to give a court jurisdiction
on the ground of the plaintiff's residence
in the State or country of the judicial
forum, his residence must be bona fide. If
a spouse leaves the family domicile and goes
to another State for the sole purpose of
obtaining a divorce, and with no intention of
remaining, his residence there is not sufficient
to confer jurisdiction on the courts of that
State. This is especially true where the cause
of divorce is one not recognized by the laws of
the State of his own domicile.

Page 49

reach of execution to satisfy the judgment debt


of Corominas.
VAN DORN V. ROMILLO
139 SCRA 139

MANILA SURETY & FIDELITY VS. TEODORO


FACTS:

Facts:

Jose Corominas, Jr. and Sonia Lizares were


married in Iloilo on January 5, 1935. On
November 29,1954, a decree of divorce was
granted by the Court of the State of Nevada
dissolving the bonds of matrimony between
Sonia Lizares and Jose Corominas, Jr. . . .

Trinidad Teodoro met Jose Corominas, Jr. in


Hongkong on October 30, 1955. . . . On March
26, 1956, they went through a Buddhist
wedding ceremony in Hongkong. Upon their
return to the Philippines they took up residence
in a rented house at No. 2305 Agno Street . . .
Manila. On September 5, 1961, plaintiff and
Jose Corominas, Jr. were married for a second
time on Washoe County, Nevada. U.S.A.
Additional Pertinent facts, also mentioned in
the decision under review and controverted by
the parties, are that Sonia Lizares is still living
and that the conjugal partnership formed by
her marriage to Corominas was dissolved by
the Juvenile and Domestic Relations Court of
Manila upon their joint petition, the decree of
dissolution having been issued on October 21,
1957. Trinidad questioned the levy on the
property since the property in question was her
paraphernal property.

Held:

ISSUE:
Whether or not the properties in question are
conjugal?

RULING:
There is no doubt that the decree of
divorce granted by the Court of Nevada in
1954 is not valid under Philippine law,
which has outlawed divorce altogether;
that the matrimonial bonds between Jose
Corominas, Jr. and Sonia Lizares have not been
dissolved, although their conjugal partnership
was terminated in 1957; and that the former's
subsequent marriage in Hongkong to Trinidad
Teodoro is bigamous and void.
In the present case, however, we find no need
to pass on this question. The particular
properties involved here which were admittedly
acquired by respondent Teodoro, cannot be
deemed to belong to such co-ownership
because, as found by the trial court and
confirmed by the Court of Appeals, the funds
used in acquiring said properties were
fruits
of
respondent's
paraphernal
investments which accrued before her
"marriage" to Corominas. In other words
they were not acquired by either or both of the
partners in the void marriage through their
work or industry or their wages and salaries,
and hence cannot be the subject of coownership under Article 144. They remain
respondent's exclusive properties, beyond the

Private International Law

Petitioner Alice Reyes Van Dorn is a citizen


of the Philippines while private respondent
Richard Upton is a US citizen; they were
married in Hong Kong in 1972; after the
marriage, they established their residence
in the Philippines and begot 2 children;
Alicia filed for divorce in Nevada; the
parties were divorced in Nevada, US, in
1982; and petitioner has remarried also in
Nevada, this time to Theodore Van Dorn.
On June 18, 1983 Upton filed a suit against
petitioner in the RTC-Pasay, stating that
petitioner's business in Ermita, Manila (the
Galleon Shop), is conjugal property and
asking that petitioner be ordered to render
an accounting of that business, and that
Upton be declared as having the right to
manage the conjugal property.

Owing to the nationality principle


embodied in Art. 13, NCC, only
Philippine nationals are covered by
the policy against absolute divorce
the same being considered contrary to
our concept of public policy and
morality. However, aliens may obtain
divorce abroad, which may be recognized
in the Philippines provided they are valid
according to their national law.
In this case, the divorce in Nevada
released private respondents from the
marriage from the standards of
American law, under which divorce
dissolves the marriage.

Court said that Ours is not only a court


of law but also a court of equity. The
Court could not turn its back on its citizen
when the foreign national itself benefited
from such divorce decree;

Thus, pursuant to his national law,


Upton is no longer the husband of
petitioner. He would have no standing to
sue in the case below as petitioner's
husband who is entitled to exercise control
over conjugal assets.
To maintain, as Upton does, that under our
laws, petitioner has to be considered still
married to him and still subject to a wife's
obligations under the NCC cannot be just.
Petitioner should not be obliged to live
together with, observe respect and fidelity,
and render support to private respondent.
The latter should not continue to be one of
her heirs w/ possible rights to conjugal
properties. She should not be discriminated
against in her own country if the ends of
justice are to be observed.

PILAPIL V. IBAY-SOMERA

Page 50

174 SCRA 653

Petitioner Imelda Manalaysay Pilapil, a


Filipino citizen, and private respondent
Erich Ekkehard Geiling, a German national,
were married in the Federal Republic of
Germany.
The
marriage
started
auspiciously enough, and the couple lived
together for some time in Malate, Manila
where their only child, Isabella Pilapil
Geiling, was born on April 20, 1980.
Thereafter, marital discord set in, with
mutual
recriminations
between
the
spouses, followed by a separation de facto
between them.
After about three and a half years of
marriage, private respondent initiated a
divorce proceeding against petitioner in
Germany. He claimed that there was failure
of their marriage and that they had been
living apart since April, 1982. Petitioner, on
the other hand, filed an action for legal
separation, support and separation of
property before the Regional Trial Court of
Manila.
Thereafter a decree of divorce was
promulgated. The records show that under
German law said court was locally and
internationally competent for the divorce
proceeding and that the dissolution of said
marriage was legally founded on and
authorized by the applicable law of that
foreign jurisdiction.
More than five months after the
issuance of the divorce decree, private
respondent filed two complaints for
adultery before the City Fiscal of Manila
alleging that, while still married to said
respondent, petitioner "had an affair with a
certain William Chua as early as 1982 and
with yet another man named Jesus Chua
sometime in 1983".

Issue:
WON the adultery case be sustained even
though there has already been a finality of a
divorce decree.
Held:

The law specifically provides that in


prosecutions for adultery and concubinage
the person who can legally file the
complaint should be the offended spouse,
and nobody else.

Corollary to such exclusive grant of


power to the offended spouse to
institute the action, it necessarily
follows that such initiator must have
the
status,
capacity
or
legal
representation to do so at the time of
the filing of the criminal action. This is
a familiar and express rule in civil actions;
in fact, lack of legal capacity to sue, as a
ground for a motion to dismiss in civil
cases, is determined as of the filing of the
complaint or petition. In these cases,
therefore, it is indispensable that the status
and capacity of the complainant to
commence the action be definitely

Private International Law

established and, as already demonstrated,


such status or capacity must indubitably
exist as of the time he initiates the action.

In the present case, the fact that


private respondent obtained a valid
divorce in his country, the Federal
Republic of Germany, it is deemed
admitted. Said divorce and its legal
effects may be recognized in the
Philippines insofar as private respondent is
concerned in view of the nationality
principle in our civil law on the matter of
status of persons.
Therefore, private respondent, being
no longer the husband of petitioner,
had no legal standing to commence
the adultery case under the imposture
that he was the offended spouse at
the time he filed suit.

LLORENTE V. COURT OF APPEALS


GR No. 124371, November 23, 2000
FACTS:

Lorenzo Llorente and petitioner Paula


Llorente were married in 1937 in the
Philippines. Lorenzo was an enlisted
serviceman of the US Navy. Soon after, he
left
for
the
US
where
through
naturalization, he became a US Citizen.
Upon his visit to his wife, he discovered that
she was living with his brother and a child was
born. The child was registered as
illegitimate but the name of the father
was left blank. Llorente filed a divorce in
California in which Paula was represented by
counsel, John Riley, and actively participated in
the proceedings, which later on became final.
He married Alicia and they lived together for 25
years bringing 3 children. He made his last will
and testament stating that all his properties
will be given to his second marriage. He filed a
petition of probate that made or appointed
Alicia his special administrator of his estate.
Before the proceeding could be terminated,
Lorenzo died. Paula filed a letter of
administration over Llorentes estate. The trial
granted the letter and denied the motion for
reconsideration. An appeal was made to the
Court of Appeals, which affirmed and modified
the judgment of the Trial Court that she be
declared co-owner of whatever properties, she
and the deceased, may have acquired in their
25 years of cohabitation.
ISSUE:

Whether or not national law shall apply?


RULING:
Art. 15. Laws relating to family rights and
duties, or to the status, condition and legal
capacity of persons are binding upon
citizens of the Philippines, even though
living abroad.

Page 51

Art. 16. Real property as well as personal


property is subject to the law of the country
where it is situated.
First, there is no such thing as one American
law. The "national law" indicated in Article 16
of the Civil Code cannot possibly apply to
general American law. There is no such law
governing the validity of testamentary
provisions in the United States. Each State of
the union has its own law applicable to its
citizens and in force only within the State. It
can therefore refer to no other than the law of
the State of which the decedent was a
resident. Second, there is no showing that
the application of the renvoi doctrine is
called for or required by New York State
law.
However,
intestate
and
testamentary
succession, both with respect to the order of
succession and to the amount of successional
rights and to the intrinsic validity of
testamentary provisions, shall be regulated
by the national law of the person whose
succession
is
under
consideration,
whatever may be the nature of the property
and regardless of the country wherein said
property may be found. (emphasis ours)

Likewise, Lorenzo Llorente was already an


American citizen when he divorced Paula.
Such was also the situation when he married
Alicia and executed his will. As stated in
Article 15 of the civil code, aliens may
obtain divorces abroad, provided that
they are valid in their National Law. Thus
the divorce obtained by Llorente is valid
because the law that governs him is not
Philippine Law but his National Law since the
divorce was contracted after he became an
American citizen. Furthermore, his National
Law allowed divorce.

The case was remanded to the court of origin


for determination of the intrinsic validity of
Lorenzo Llorentes will and determination of
the parties successional rights allowing proof
of foreign law.
GARCIA V. RECIO
October 2, 2001

Rederick Recio, a Filipino, married


Editha Samson, an Australian in Malabon
Rizal. However, on 1989, they got divorced
in an Australian family court.

On
1992,
Rederick
became
an
Australian Citizen. He later married
Petitioner in 1994 in Cabanatuan City.

Thereafter, the two separated and


petitioner filed a complaint for Declaration
of Nullity of Marriage on the ground of
bigamy.

While the suit was pending, Rederick


was able to obtain a divorce decree in
Australia. Trial Court declared the marriage
dissolved based on the subsequent divorce
decree obtained by the respondent.

Whether the divorce between respondent and


Editha Samson was proven;
Whether respondent was proven to be legally
capacitated to marry petitioner;
Held:
A divorce obtained abroad by an alien may be
recognized in our jurisdiction, provided such
decree is valid according to the national law of
the foreigner. However, the divorce decree
and the governing personal law of the
alien spouse who obtained the divorce
must be proven. Our courts do not take
judicial notice of foreign laws and judgment;
hence, like any other facts, both the divorce
decree and the national law of the alien must
be alleged and proven according to our law on
evidence.
Was the first divorce validly obtained and
binding?
At the outset, the Court lays the following basic
legal principles; Philippine law does not provide
for absolute divorce; hence, Philippine courts
cannot grant it. A marriage between two
Filipinos cannot be dissolved even by a divorce
obtained abroad, because of Articles 1522 and
1723 of the Civil Code. In mixed marriages
involving a Filipino and a foreigner,
Article 26 of the Family Code allows the
former to contract a subsequent marriage
in case the divorce is "validly obtained
abroad by the alien spouse capacitating
him or her to remarry." A divorce obtained
abroad by a couple, who are both aliens, may
be recognized in the Philippines, provided it is
consistent with their respective national laws.
Before a foreign divorce decree can be
recognized by our courts, the party
pleading it must prove the divorce as a
fact and demonstrate its conformity to
the foreign law allowing it. Presentation
solely of the divorce decree is insufficient. In
the case at bar, Respondent only presented the
divorce decree;
Likewise, before a foreign judgment is
given presumptive evidentiary value, the
document must first be presented and
admitted in evidence. A divorce obtained
abroad is proven by the divorce decree itself.
Indeed the best evidence of a judgment is the
judgment itself. The decree purports to be a
written act or record of an act of an officially
body or tribunal of a foreign country.
However, under Sections 24 and 25 of Rule
132, a writing or document may be proven as a
public or official record of a foreign country by
either (1) an official publication or (2) a
copy thereof attested by the officer
having legal custody of the document. If
the record is not kept in the Philippines,
such copy must be (a) accompanied by a
certificate
issued
by
the
proper
diplomatic or consular officer in the
Philippine Foreign Service stationed in
the foreign country in which the record is

Issues:

Private International Law

Page 52

kept and (b) authenticated by the seal of


his office.
The divorce decree between respondent and
Editha Samson appears to be an authentic one
issued by an Australian family court. However,
appearance is not sufficient; compliance with
the aforementioned rules on evidence must be
demonstrated.
Fortunately for respondent, this matter
was not objected to by the petitioner,
thus by virtue of such waiver, is deemed
admitted as evidence.
Who has the burden of proving a foreign
law?
Respondent has the burden of proof; The
burden of proof lies with "the party who
alleges the existence of a fact or thing
necessary in the prosecution or defense
of an action." It is a well-settled that courts
cannot take judicial notice of foreign laws. Like
any other facts, they must be alleged and
proved. Australian marital laws are not among
those matters that judges are supposed to
know by reason of their judicial function. The
power of judicial notice must be exercised with
caution, and every reasonable doubt upon the
subject should be resolved in the negative.
2nd
Issue:
Is
Respondent
capacitated to remarry?

legally

Divorce means the legal dissolution of a lawful


union for a cause arising after marriage. But
divorces are of different types. The two basic
ones are (1) absolute divorce or a vinculo
matrimonii and (2) limited divorce or a
mensa et thoro. The first kind terminates the
marriage, while the second suspends it and
leaves the bond in full force. There is no
showing in the case at bar which type of
divorce was procured by respondent.
Respondent presented a decree nisi or an
interlocutory decree a conditional or
provisional judgment of divorce. It is in effect
the same as a separation from bed and
board, although an absolute divorce may
follow after the lapse of the prescribed
period during which no reconciliation is
effected.
On its face, the herein Australian divorce
decree contains a restriction that reads:
"1. A party to a marriage who marries
again
before
this
decree
becomes
absolute (unless the other party has died)
commits the offence of bigamy."
This quotation bolsters the Courts contention
that the divorce obtained by respondent may
have been restricted. It did not absolutely
establish his legal capacity to remarry
according to his national law.
Significance of the Certificate of Legal
Capacity
Legal capacity to contract marriage is
determined by the national law of the party

Private International Law

concerned. The certificate mentioned in Article


21 of the Family Code would have been
sufficient to establish the legal capacity of
respondent, had he duly presented it in court.
A duly authenticated and admitted
certificate is prima facie evidence of legal
capacity to marry on the part of the alien
applicant for a marriage license.
In the case at bar, there is absolutely no
evidence that proves respondent's legal
capacity to marry petitioner.
Based on the above records, the Court cannot
conclude that respondent, who was then a
naturalized Australian citizen, was legally
capacitated to marry petitioner on January 12,
1994. The court a quo erred in finding that
the divorce decree ipso facto clothed
respondent with the legal capacity to
remarry without requiring him to adduce
sufficient evidence to show the Australian
personal law governing his status; or at the
very least, to prove his legal capacity to
contract the second marriage.
Neither can the Court grant petitioner's
prayer to declare her marriage to
respondent null and void on the ground of
bigamy. After all, it may turn out that under
Australian law, he was really capacitated to
marry petitioner as a direct result of the
divorce decree.
Hence, case was remanded to the court a quo
for further determination of legal capacity and
to receive evidence to determine if bigamy has
been committed;
REPUBLIC V. ORBECIDO
GR NO. 154380, October 5, 2005
Facts:
On May 24, 1981, Cipriano Orbecido III
married Lady Myros M. Villanueva in the
Philippines in Lam-an, Ozamis City. Their
marriage was blessed with a son and a
daughter.
In 1986, Ciprianos wife left for the United
States
bringing
along
their
son
Kristoffer. A few years later, Cipriano
discovered that his wife had been naturalized
as an American citizen.
Sometime in 2000, Cipriano learned from his
son that his wife had obtained a divorce
decree and then married a certain Innocent
Stanley. She, Stanley and her child by him
currently live in San Gabriel, California.
Cipriano thereafter filed with the trial
court a petition for authority to remarry
invoking Paragraph 2 of Article 26 of the
Family Code. No opposition was filed. Finding
merit in the petition, the court granted the
same.
The Republic, herein petitioner,
through the Office of the Solicitor General
(OSG), sought reconsideration but it was
denied.

Page 53

The OSG contends that Paragraph 2 of Article


26 of the Family Code is not applicable to
the instant case because it only applies to
a valid mixed marriage; that is, a marriage
celebrated between a Filipino citizen and an
alien. Furthermore, the OSG argues there is no
law that governs respondents situation. The
OSG posits that this is a matter of legislation
and not of judicial determination.

Art. 99.

No person shall be entitled to a


legal separation who has not
resided in the Philippines for
one year prior to the filing of
the petition, unless the cause
for the legal separation has
taken place within the territory
of this Republic. (Sec. 2a, Act
No. 2710)
TITLE II

Held:
Taking into consideration the legislative intent
and applying the rule of reason, we hold that
Paragraph 2 of Article 26 should be
interpreted to include cases involving
parties who, at the time of the
celebration of the marriage were Filipino
citizens, but later on, one of them becomes
naturalized as a foreign citizen and obtains a
divorce decree. The Filipino spouse should
likewise be allowed to remarry as if the
other party were a foreigner at the time
of the solemnization of the marriage. To
rule otherwise would be to sanction absurdity
and injustice. Where the interpretation of a
statute according to its exact and literal import
would lead to mischievous results or
contravene the clear purpose of the legislature,
it should be construed according to its spirit
and reason, disregarding as far as necessary
the letter of the law. A statute may therefore
be extended to cases not within the literal
meaning of its terms, so long as they come
within its spirit or intent.
In view of the foregoing, we state the twin
elements for the application of Paragraph 2 of
Article 26 as follows:
3. There is a valid marriage that has
been celebrated between a Filipino
citizen and a foreigner; and
4. A valid divorce is obtained abroad
by the alien spouse capacitating
him or her to remarry.

LEGAL SEPARATION
Art. 55. A petition for legal separation may be
filed on any of the following grounds:
(1) Repeated physical violence or
grossly abusive conduct directed
against the petitioner, a common child,
or a child of the petitioner;
(2) Physical violence or moral pressure
to compel the petitioner to change
religious or political affiliation;
(3) Attempt of respondent to corrupt or
induce the petitioner, a common child,
or a child of the petitioner, to engage in
prostitution, or connivance in such
corruption or inducement;
(4) Final judgment sentencing the
respondent to imprisonment of more
than six years, even if pardoned;
(5)
Drug
addiction
or
habitual
alcoholism of the respondent;
(6) Lesbianism or homosexuality of the
respondent;
(7) Contracting by the respondent of a
subsequent
bigamous
marriage,
whether in the Philippines or abroad;
(8) Sexual infidelity or perversion;

The reckoning point is not the citizenship


of the parties at the time of the
celebration of the marriage, but their
citizenship at the time a valid divorce is
obtained abroad by the alien spouse
capacitating the latter to remarry.
5.

(9) Attempt by the respondent against


the life of the petitioner; or
(10) Abandonment of petitioner by
respondent without justifiable cause for
more than one year.

Legal Separation

Civil Code:
Art. 97.
A
separation may be filed:
1.

2.

petition

for

legal

For adultery on the part of


the
wife
and
for
concubinage on the part of
the husband as defined in
the Penal Code; or
An attempt by one spouse
against the life of the
other. (n)

Private International Law

For purposes of this Article, the term "child"


shall include a child by nature or by adoption.
(9a)
Art. 56. The petition for legal separation shall
be denied on any of the following grounds:
(1) Where the aggrieved party has
condoned
the
offense
or
act
complained of;
(2) Where the aggrieved party has
consented to the commission of the
offense or act complained of;

Page 54

(3) Where there is connivance between


the parties in the commission of the
offense or act constituting the ground
for legal separation;

absolute community or the conjugal


partnership, which shall be forfeited in
accordance with the provisions of
Article 43(2);

(4) Where both parties have given


ground for legal separation;

(3) The custody of the minor children


shall be awarded to the innocent
spouse, subject to the provisions of
Article 213 of this Code; and

(5) Where there is collusion between


the parties to obtain decree of legal
separation; or
(6) Where the action is barred by
prescription. (100a)
Art. 57. An action for legal separation shall be
filed within five years from the time of the
occurrence of the cause. (102)
Art. 58. An action for legal separation shall in
no case be tried before six months shall have
elapsed since the filing of the petition. (103)
Art. 59. No legal separation may be decreed
unless the Court has taken steps toward the
reconciliation of the spouses and is fully
satisfied,
despite
such
efforts,
that
reconciliation is highly improbable. (n)
Art. 60. No decree of legal separation shall be
based upon a stipulation of facts or a
confession of judgment.
In any case, the Court shall order the
prosecuting attorney or fiscal assigned to it to
take steps to prevent collusion between the
parties and to take care that the evidence is
not fabricated or suppressed. (101a)
Art. 61. After the filing of the petition for legal
separation, the spouses shall be entitled to live
separately from each other.
The court, in the absence of a written
agreement between the spouses, shall
designate either of them or a third person to
administer the absolute community or conjugal
partnership
property.
The
administrator
appointed by the court shall have the same
powers and duties as those of a guardian under
the Rules of Court. (104a)
Art. 62. During the pendency of the action for
legal separation, the provisions of Article 49
shall likewise apply to the support of the
spouses and the custody and support of the
common children. (105a)

(4) The offending spouse shall be


disqualified from inheriting from the
innocent
spouse
by
intestate
succession. Moreover, provisions in
favor of the offending spouse made in
the will of the innocent spouse shall be
revoked by operation of law. (106a)
Art. 64. After the finality of the decree of legal
separation, the innocent spouse may revoke
the donations made by him or by her in favor
of the offending spouse, as well as the
designation of the latter as beneficiary in any
insurance policy, even if such designation be
stipulated as irrevocable. The revocation of the
donations shall be recorded in the registries of
property in the places where the properties are
located. Alienations, liens and encumbrances
registered in good faith before the recording of
the complaint for revocation in the registries of
property shall be respected. The revocation of
or change in the designation of the insurance
beneficiary shall take effect upon written
notification thereof to the insured.
The action to revoke the donation under this
Article must be brought within five years from
the time the decree of legal separation become
final. (107a)
Art. 65. If the spouses should reconcile, a
corresponding joint manifestation under oath
duly signed by them shall be filed with the
court in the same proceeding for legal
separation. (n)
Art. 66. The reconciliation referred to in the
preceding Articles shall have the following
consequences:
(1) The legal separation proceedings, if
still
pending,
shall
thereby
be
terminated at whatever stage; and

Art. 63. The decree of legal separation shall


have the following effects:

(2) The final decree of legal separation


shall be set aside, but the separation of
property and any forfeiture of the
share of the guilty spouse already
effected shall subsist, unless the
spouses agree to revive their former
property regime.

(1) The spouses shall be entitled to live


separately from each other, but the
marriage bonds shall not be severed;

The court's order containing the foregoing shall


be recorded in the proper civil registries.
(108a)

(2) The absolute community or the


conjugal partnership shall be dissolved
and liquidated but the offending
spouse shall have no right to any share
of the net profits earned by the

Art. 67. The agreement to revive the former


property regime referred to in the preceding
Article shall be executed under oath and shall
specify:

Private International Law

Page 55

(1) The properties to be contributed


anew to the restored regime;
(2) Those to be retained as separated
properties of each spouse; and

Civil Code:
Art. 15, supra
Art. 335.

The following cannot adopt:

(3) The names of all their known


creditors, their addresses and the
amounts owing to each.

1. Those
who
have
legitimate,
legitimated,
acknowledged
natural
children, or natural children
by legal fiction;
2. The
guardian,
with
respect to the ward, before
the final approval of his
accounts;
3. A
married
person,
without the consent of the
other spouse;
4. Non-resident aliens;
5. Resident aliens with
whose
government
the
Republic of the Philippines
has
broken
diplomatic
relations;
6. Any person who has
been convicted of a crime
involving moral turpitude,
when the penalty imposed
was
six
months'
imprisonment
or
more.
(174a)

The agreement of revival and the motion for its


approval shall be filed with the court in the
same proceeding for legal separation, with
copies of both furnished to the creditors named
therein. After due hearing, the court shall, in its
order, take measure to protect the interest of
creditors and such order shall be recorded in
the proper registries of properties.
The recording of the ordering in the registries
of property shall not prejudice any creditor not
listed or not notified, unless the debtor-spouse
has sufficient separate properties to satisfy the
creditor's claim. (195a, 108a)
Capacity to Remarry
Art. 26.

All
marriages
solemnized
outside the Philippines, in
accordance with the laws in
force in the country where they
were solemnized, and valid
there as such, shall also be
valid in this country, except
those prohibited under Articles
35 (1), (4), (5) and (6), 3637
and 38. (17a)
Where a marriage between
a Filipino citizen and a
foreigner
is
validly
celebrated and a divorce is
thereafter validly obtained
abroad by the alien spouse
capacitating him or her to
remarry, the Filipino spouse
shall
have
capacity
to
remarry under Philippine
law.
(As
amended
by
Executive Order 227)

Art. 339.
adopted:

of

1. Personal law: status/legal capacity


National law of the parties

2. lex loci celebrationis:

exceptions:

Family Code Art. 26, paragraph


1

Consular Marriages consul


granted by the accepting
country where such consular
office was located to solemnize
marriage

VIII. Parents
Relationship)

and

Children

Private International Law

(Parental

following

cannot

be

1. A
married
person,
without the written consent
of the other spouse;
2. An alien with whose
government the Republic of
the Philippines has broken
diplomatic relations;
3. A person who has
already been adopted. (n)
Muslim Code:
TITLE V
Parental Authority
CHAPTER I
Nature and Effects

Nota bene:
Formal validity law of the place
celebration; lex loci contractus rule
Substantial validity governed by:

The

Art. 71.

Who exercises.
(1) The father and the
mother
shall
jointly
exercise
just
and
reasonable
parental
authority and fulfill their
responsibility over their
legitimate
and
acknowledged children. In
case of disagreement, the
father's
decision
shall
prevail unless there is a
judicial
order
to
the
contrary.
(2) The
mother
shall
exercise parental authority
over her children born out
of wedlock, but the court

Page 56

may,
when
the
best
interests of the children so
require, appoint a general
guardian.
.chan
robles
virtual law library
Art. 72.

renounced
nor
transferred
except as otherwise provided
in this Code and the general
principles of Islamic law.
Art. 77.
authority.

Duty to parents.
(1) Children shall respect,
revere, and obey their
parents always unless the
latter
cast
them
into
disbelief.

Duty to children. Every


parent
and
every
person
exercising parental authority
shall see to it that the rights of
the children are respected, and
their duties complied with, and
shall particularly by precept
and example, imbue them with
religious and civic attachment
to the ideal of permanent world
peace.

Art. 74.

Effects upon person of children.


The parents have, with
respect to their unemancipated
children:

Art. 78.

property

of

(1) The father, or in his


absence the mother, shall
be the legal administrator
of the property of the child
under parental authority. If
the property is worth more
than five thousand pesos,
the father or the mother
shall give a bond to be
approved by the court.
(2) The court may appoint
a guardian (wasi) in the
absence of one who is
natural or testamentary.
Art. 76.

Parental
authority
nontransferable.

Parental
authority
can
neither
be

Private International Law

Care and custody.


(1) The care and custody
of children below seven
years of age whose parents
are divorced shall belong to
the mother or, in her
absence, to the maternal
grandmother, the paternal
grandmother, the sister
and aunts. In their default,
it shall devolve upon the
father and the nearest
paternal
relatives.
The
minor above seven years of
age but below the age of
puberty may choose the
parent with whom he wants
to stay.

(b) The power to correct,


discipline, and punish them
moderately.
Effects
upon
children.

parental

CHAPTER II
Custody and Guardianship

(a) The duty to support


them, have them in their
company,
educate
and
instruct them in keeping
with their means and
represent them in all
actions which shall redound
to their benefits; and

Art. 75.

of

(1) Parental
authority
terminates upon the death
of the parents or the child,
or upon emancipation.
(2) Subject to Article 78,
the widowed mother who
contracts a subsequent
marriage
shall
lose
parental
authority
and
custody over all children by
the deceased husband,
unless the second husband
is related to them within
the prohibited degrees of
consanguinity.
(3) The court may deprive
a
person
of
parental
authority or suspend the
exercise thereof if he treats
his children with excessive
harshness,
gives
then
corrupting
or
immoral
orders and counsel, or
abandons them.

(2) Grandparents
are
likewise entitled to respect
and reverence, and shall be
consulted
whenever
practicable by all members
of the family on all
important questions.
Art. 73.

Extinguishment

(2) The
unmarried
daughter who has reached
the age of puberty shall
stay with the father; the
son,
under
the
same
circumstances, shall stay
with the mother.
Art. 79.

Guardian for marriage (wali).


The following persons shall
have authority to act as
guardian for marriage (wali) in
the order of precedence:
(a) Father
(b) Paternal grandfather;
(c) Brother
and
other
paternal relatives;

Page 57

(d) Paternal grandfather's


executor or nominee; or
.chan robles virtual law
library
(e) The court.
Art. 80.

Guardian of minor's property.


The following persons shall
exercise guardianship over the
property of minors in the order
of precedence:
(a) Father;
(b) Father's executor or
nominee;
(c) Paternal grandfather;
(d) Paternal grandfather's
nominee; or
(e) The court.

Art. 163. The filiation of children may be by


nature or by adoption. Natural filiation may be
legitimate or illegitimate. (n)
Art. 164. Children conceived or born during the
marriage of the parents are legitimate.
Children conceived as a result of artificial
insemination of the wife with the sperm of the
husband or that of a donor or both are likewise
legitimate children of the husband and his wife,
provided, that both of them authorized or
ratified such insemination in a written
instrument executed and signed by them
before the birth of the child. The instrument
shall be recorded in the civil registry together
with the birth certificate of the child. (55a,
258a)
Art. 165. Children conceived and born outside a
valid
marriage
are
illegitimate,
unless
otherwise provided in this Code. (n)
Art. 166. Legitimacy of a child may
impugned only on the following grounds:

(3) That in case of children conceived


through artificial insemination, the
written authorization or ratification of
either parent was obtained through
mistake, fraud, violence, intimidation,
or undue influence. (255a)
Art. 167. The child shall be considered
legitimate although the mother may have
declared against its legitimacy or may have
been sentenced as an adulteress. (256a)
Art. 168. If the marriage is terminated and the
mother contracted another marriage within
three hundred days after such termination of
the former marriage, these rules shall govern
in the absence of proof to the contrary:
(1) A child born before one hundred
eighty days after the solemnization of
the subsequent marriage is considered
to have been conceived during the
former marriage, provided it be born
within three hundred days after the
termination of the former marriage;
(2) A child born after one hundred
eighty days following the celebration of
the subsequent marriage is considered
to have been conceived during such
marriage, even though it be born
within the three hundred days after the
termination of the former marriage.
(259a)
Art. 169. The legitimacy or illegitimacy of a
child born after three hundred days following
the termination of the marriage shall be proved
by whoever alleges such legitimacy or
illegitimacy. (261a)

be

(1) That it was physically impossible for


the husband to have sexual intercourse
with his wife within the first 120 days
of the 300 days which immediately
preceded the birth of the child because
of:
(a) the physical incapacity of
the husband to have sexual
intercourse with his wife;
(b) the fact that the husband
and wife were living separately
in such a way that sexual
intercourse was not possible; or
(c) serious illness of the
husband,
which
absolutely
prevented sexual intercourse;
(2) That it is proved that for biological
or other scientific reasons, the child
could not have been that of the
husband, except in the instance

Private International Law

provided in the second paragraph of


Article 164; or

Art. 170. The action to impugn the legitimacy


of the child shall be brought within one year
from the knowledge of the birth or its recording
in the civil register, if the husband or, in a
proper case, any of his heirs, should reside in
the city or municipality where the birth took
place or was recorded.
If the husband or, in his default, all of his heirs
do not reside at the place of birth as defined in
the first paragraph or where it was recorded,
the period shall be two years if they should
reside in the Philippines; and three years if
abroad. If the birth of the child has been
concealed from or was unknown to the
husband or his heirs, the period shall be
counted from the discovery or knowledge of
the birth of the child or of the fact of
registration of said birth, whichever is earlier.
(263a)
Art. 171. The heirs of the husband may impugn
the filiation of the child within the period
prescribed in the preceding article only in the
following cases:

Page 58

(1) If the husband should died before


the expiration of the period fixed for
bringing his action;
(2) If he should die after the filing of
the complaint without having desisted
therefrom; or
(3) If the child was born after the death
of the husband. (262a)
Chapter 2. Proof of Filiation
Art. 172. The filiation of legitimate children is
established by any of the following:
(1) The record of birth appearing in the
civil register or a final judgment; or
(2) An admission of legitimate filiation
in a public document or a private
handwritten instrument and signed by
the parent concerned.
In the absence of the foregoing evidence, the
legitimate filiation shall be
proved by:
(1)
The
open
and
continuous
possession of the status of a legitimate
child; or
(2) Any other means allowed by the
Rules of Court and special laws. (265a,
266a, 267a)
Art. 173. The action to claim legitimacy may be
brought by the child during his or her lifetime
and shall be transmitted to the heirs should the
child die during minority or in a state of
insanity. In these cases, the heirs shall have a
period of five years within which to institute
the action.
Art. 174. Legitimate children shall have the
right:
(1) To bear the surnames of the father
and the mother, in conformity with the
provisions of the Civil Code on
Surnames;
(2) To receive support from their
parents, their ascendants, and in
proper cases, their brothers and
sisters,
in
conformity
with
the
provisions of this Code on Support; and
(3) To be entitled to the legitimate and
other successional rights granted to
them by the Civil Code. (264a)
Chapter 3. Illegitimate Children
Art. 175. Illegitimate children may establish
their illegitimate filiation in the same way and
on the same evidence as legitimate children.

Private International Law

The action must be brought within the same


period specified in Article 173, except when the
action is based on the second paragraph of
Article 172, in which case the action may be
brought during the lifetime of the alleged
parent. (289a)
Art. 176. Illegitimate children shall use the
surname and shall be under the parental
authority of their mother, and shall be entitled
to support in conformity with this Code. The
legitime of each illegitimate child shall consist
of one-half of the legitime of a legitimate child.
Except for this modification, all other provisions
in the Civil Code governing successional rights
shall
remain
in
force.
(287a)

Chapter 4. Legitimated Children


Art. 177. Only children conceived and born
outside of wedlock of parents who, at the time
of the conception of the former, were not
disqualified by any impediment to marry each
other may be legitimated. (269a)
Art. 178. Legitimation shall take place by a
subsequent valid marriage between parents.
The annulment of a voidable marriage shall not
affect the legitimation. (270a)chan robles
virtual law library
Art. 179. Legitimated children shall enjoy the
same rights as legitimate children. (272a)
Art. 180. The effects of legitimation shall
retroact to the time of the child's birth. (273a)
Art. 181. The legitimation of children who died
before the celebration of the marriage shall
benefit their descendants. (274)
Art. 182. Legitimation may be impugned only
by those who are prejudiced in their rights,
within five years from the time their cause of
action accrues. (275a)
Art. 209. Pursuant to the natural right and duty
of parents over the person and property of
their
unemancipated
children,
parental
authority and responsibility shall include the
caring for and rearing them for civic
consciousness
and
efficiency
and
the
development of their moral, mental and
physical character and well-being. (n)
Art. 210. Parental authority and responsibility
may not be renounced or transferred except in
the cases authorized by law. (313a)
Art. 211. The father and the mother shall jointly
exercise parental authority over the persons of
their
common
children.
In
case
of
disagreement, the father's decision shall
prevail, unless there is a judicial order to the
contrary.
Children shall always observe respect and
reverence towards their parents and are
obliged to obey them as long as the children

Page 59

are under parental authority. (311a)


robles virtual law library

chan

Art. 212. In case of absence or death of either


parent, the parent present shall continue
exercising parental authority. The remarriage
of the surviving parent shall not affect the
parental authority over the children, unless the
court appoints another person to be the
guardian of the person or property of the
children. (n)
Art. 213. In case of separation of the parents,
parental authority shall be exercised by the
parent designated by the Court. The Court shall
take into account all relevant considerations,
especially the choice of the child over seven
years of age, unless the parent chosen is unfit.
(n)
Art. 214. In case of death, absence or
unsuitability of the parents, substitute parental
authority shall be exercised by the surviving
grandparent. In case several survive, the one
designated by the court, taking into account
the same consideration mentioned in the
preceding article, shall exercise the authority.
(355a)
Art. 215. No descendant shall be compelled, in
a criminal case, to testify against his parents
and grandparents, except when such testimony
is indispensable in a crime against the
descendant or by one parent against the other.
(315a)

Chapter 2. Substitute and Special Parental


Authority
Art. 216. In default of parents or a judicially
appointed guardian, the following person shall
exercise substitute parental authority over the
child in the order indicated:
(1) The surviving grandparent, as
provided in Art. 214;
(2) The oldest brother or sister, over
twenty-one years of age, unless unfit or
disqualified; and
(3) The child's actual custodian, over
twenty-one years of age, unless unfit or
disqualified.
Whenever the appointment or a judicial
guardian over the property of the child
becomes necessary, the same order of
preference shall be observed. (349a, 351a,
354a)
Art. 217. In case of foundlings, abandoned
neglected or abused children and other
children similarly situated, parental authority
shall be entrusted in summary judicial
proceedings to heads of children's homes,
orphanages and similar institutions duly
accredited by the proper government agency.
(314a)

Private International Law

Art. 218. The school, its administrators and


teachers, or the individual, entity or institution
engaged in child are shall have special parental
authority and responsibility over the minor
child while under their supervision, instruction
or custody.
Authority and responsibility shall apply to all
authorized activities whether inside or outside
the premises of the school, entity or institution.
(349a)
Art. 129. Those given the authority and
responsibility under the preceding Article shall
be principally and solidarily liable for damages
caused by the acts or omissions of the
unemancipated minor. The parents, judicial
guardians or the persons exercising substitute
parental authority over said minor shall be
subsidiarily liable.
The respective liabilities of those referred to in
the preceding paragraph shall not apply if it is
proved that they exercised the proper diligence
required under the particular circumstances.
All other cases not covered by this and the
preceding articles shall be governed by the
provisions of the Civil Code on quasi-delicts. (n)

Chapter 3. Effect of Parental Authority


Upon the Persons of the Children
Art. 220. The parents and those exercising
parental authority shall have with the respect
to their unemancipated children on wards the
following rights and duties:
(1) To keep them in their company, to
support, educate and instruct them by
right precept and good example, and
to provide for their upbringing in
keeping with their means;
(2) To give them love and affection,
advice and counsel, companionship
and understanding;
(3) To provide them with moral and
spiritual guidance, inculcate in them
honesty, integrity, self-discipline, selfreliance, industry and thrift, stimulate
their interest in civic affairs, and inspire
in them compliance with the duties of
citizenship;
(4) To furnish them with good and
wholesome
educational
materials,
supervise their activities, recreation
and association with others, protect
them from bad company, and prevent
them from acquiring habits detrimental
to their health, studies and morals;
(5) To represent them in all matters
affecting their interests;
(6) To demand from them respect and
obedience;

Page 60

(7) To impose discipline on them as


may
be
required
under
the
circumstances; and
(8) To perform such other duties as are
imposed by law upon parents and
guardians. (316a)
Art. 221. Parents and other persons exercising
parental authority shall be civilly liable for the
injuries and damages caused by the acts or
omissions of their unemancipated children
living in their company and under their
parental authority subject to the appropriate
defenses provided by law. (2180(2)a and (4)a )
Art. 222. The courts may appoint a guardian of
the child's property or a guardian ad litem
when the best interests of the child so requires.
(317)
Art. 223. The parents or, in their absence or
incapacity, the individual, entity or institution
exercising parental authority, may petition the
proper court of the place where the child
resides, for an order providing for disciplinary
measures over the child. The child shall be
entitled to the assistance of counsel, either of
his choice or appointed by the court, and a
summary hearing shall be conducted wherein
the petitioner and the child shall be heard.
However, if in the same proceeding the court
finds the petitioner at fault, irrespective of the
merits of the petition, or when the
circumstances so warrant, the court may also
order the deprivation or suspension of parental
authority or adopt such other measures as it
may deem just and proper. (318a)
Art. 224. The measures referred to in the
preceding article may include the commitment
of the child for not more than thirty days in
entities or institutions engaged in child care or
in children's homes duly accredited by the
proper government agency.
The parent exercising parental authority shall
not interfere with the care of the child
whenever committed but shall provide for his
support. Upon proper petition or at its own
instance, the court may terminate the
commitment of the child whenever just and
proper.
(391a)

may determine, but not less than ten per


centum (10%) of the value of the property or
annual income, to guarantee the performance
of the obligations prescribed for general
guardians.
A verified petition for approval of the bond
shall be filed in the proper court of the place
where the child resides, or, if the child resides
in a foreign country, in the proper court of the
place where the property or any part thereof is
situated.
The petition shall be docketed as a summary
special proceeding in which all incidents and
issues regarding the performance of the
obligations referred to in the second paragraph
of this Article shall be heard and resolved.
The ordinary rules on guardianship shall be
merely suppletory except when the child is
under substitute parental authority, or the
guardian is a stranger, or a parent has
remarried, in which case the ordinary rules on
guardianship shall apply. (320a)
Art. 226. The property of the unemancipated
child earned or acquired with his work or
industry or by onerous or gratuitous title shall
belong to the child in ownership and shall be
devoted exclusively to the latter's support and
education, unless the title or transfer provides
otherwise.
The right of the parents over the fruits and
income of the child's property shall be limited
primarily to the child's support and secondarily
to the collective daily needs of the family.
(321a, 323a)
Art. 227. If the parents entrust the
management or administration of any of their
properties to an unemancipated child, the net
proceeds of such property shall belong to the
owner. The child shall be given a reasonable
monthly allowance in an amount not less than
that which the owner would have paid if the
administrator were a stranger, unless the
owner, grants the entire proceeds to the child.
In any case, the proceeds thus give in whole or
in part shall not be charged to the child's
legitime.
(322a)

Chapter 5. Suspension or Termination of


Parental Authority
Chapter 4. Effect of Parental Authority Upon
the Property of the Children
Art. 225. The father and the mother shall jointly
exercise legal guardianship over the property
of the unemancipated common child without
the necessity of a court appointment. In case of
disagreement, the father's decision shall
prevail, unless there is a judicial order to the
contrary.
Where the market value of the property or the
annual income of the child exceeds P50,000,
the parent concerned shall be required to
furnish a bond in such amount as the court

Private International Law

Art. 228. Parental authority terminates


permanently:
(1) Upon the death of the parents;
(2) Upon the death of the child; or
(3) Upon emancipation of the child.
(327a)
Art. 229. Unless subsequently revived by a final
judgment, parental authority also terminates:
(1) Upon adoption of the child;

Page 61

(2) Upon appointment of a general


guardian;

exercising special parental authority inflict


corporal punishment upon the child. (n)

(3) Upon judicial declaration of


abandonment of the child in a case
filed for the purpose;

Ching Leng v. Galang


57 OG 2312

(4) Upon final judgment of a competent


court divesting the party concerned of
parental authority; or

In Ching Leng the provision in the 1935


Constitution stating "those whose fathers are
citizens of the Philippines" refers only to
legitimate children. When the 1973 and 1987
Constitutions were drafted, the framers did not
attempt to change the intent of this provision,
even as they were presumably aware of the
Ching Leng doctrine.

(5) Upon judicial declaration of absence


or incapacity of the person exercising
parental authority. (327a)
Art. 230. Parental authority is suspended upon
conviction of the parent or the person
exercising the same of a crime which carries
with it the penalty of civil interdiction. The
authority is automatically reinstated upon
service of the penalty or upon pardon or
amnesty of the offender. (330a)
Art. 231. The court in an action filed for the
purpose in a related case may also suspend
parental authority if the parent or the person
exercising the same:
(1) Treats the child with excessive
harshness or cruelty;
(2) Gives the child corrupting orders,
counsel or example;
(3) Compels the child to beg; or
(4) Subjects the child or allows him to
be subjected to acts of lasciviousness.
The grounds enumerated above are deemed to
include cases which have resulted from
culpable negligence of the parent or the person
exercising parental authority.
If the degree of seriousness so warrants, or the
welfare of the child so demands, the court shall
deprive the guilty party of parental authority or
adopt such other measures as may be proper
under the circumstances.
The suspension or deprivation may be revoked
and the parental authority revived in a case
filed for the purpose or in the same proceeding
if the court finds that the cause therefor has
ceased and will not be repeated. (33a)
Art. 232. If the person exercising parental
authority has subjected the child or allowed
him to be subjected to sexual abuse, such
person shall be permanently deprived by the
court of such authority. (n)
Art. 233. The person exercising substitute
parental authority shall have the same
authority over the person of the child as the
parents.
In no case shall the school administrator,
teacher of individual engaged in child care

Private International Law

Nevertheless, I believe that it is now time to


abandon the Ching Leng doctrine. (Tecson V.
Comelec)
The prevailing doctrine today is that an
illegitimate child of a Filipino father and an
alien mother follows the citizenship of the alien
mother as the only legally known parent. The
illegitimate child, even if acknowledged and
legally adopted by the Filipino father, cannot
acquire the citizenship of the father. The Court
made this definitive doctrinal ruling in Ching
Leng v. Galang which involved the illegitimate
minor children of a naturalized Filipino of
Chinese descent with a Chinese woman, Sy An.
The illegitimate children were later on jointly
adopted by the naturalized Filipino and his
legal wife, So Buan Ty.
The facts in Ching Leng as quoted by the Court
from the trial courts decision are as follows:
After the petitioner Ching Leng Alias
Ching Ban Lee obtained judgment in this
Court dated May 2, 1950 granting his
petition for naturalization, he together with
his wife So Buan Ty filed another petition also
in this Court in Special Proc. No. 1216 for the
adoption of Ching Tiong Seng, Ching Liong
Ding, Victoria Ching Liong Yam, Sydney Ching
and Ching Tiong An, all minors and
admittedly the illegitimate children of
petitioner Ching Leng with one Sy An, a
Chinese citizen. Finding the petition for
adoption proper, this Court granted the same
in a decision dated September 12, 1950,
declaring the said minors free from all legal
obligations of obedience and maintenance with
respect to their mother Sy An and to all legal
intents and purposes the children of the
adopter Ching Leng alias Ching Ban Lee
and So Buan Ty with all the legal rights
and obligations provided by law.
On September 29, 1955, Ching Leng took his
oath of allegiance and became therefore
a full pledge (sic) Filipino citizen. Believing
now that his adopted illegitimate children
became Filipino citizens by virtue of his
naturalization, petitioner Ching Leng addressed
a
communication
to
the
respondent
Commissioner of Immigration requesting that
the alien certificate of registration of the said
minors be cancelled. (Bold underscoring
supplied)

Page 62

In Ching Leng, the Court made a definitive


ruling on the meaning of "minor child or
children" in Section 15 of the Naturalization
Law, as well as the meaning of children "whose
parents are citizens of the Philippines" under
the Constitution. The Court categorically
ruled that these children refer to
legitimate children only, and not to
illegitimate children. Thus, the Court held:
It is claimed that the phrases "minor children"
and "minor child", used in these provisions,
include adopted children. The argument is
predicated upon the theory that an
adopted child is, for all intents and
purposes, a legitimate child. Whenever,
the word "children" or "child" is used in
statutes, it is generally understood,
however, to refer to legitimate children,
unless the context of the law and its spirit
indicate clearly the contrary. Thus, for
instance, when the Constitution provides that
"those whose parents are citizens of the
Philippines, "and "those whose mothers are
citizens of the Philippines," who shall elect
Philippine citizenship "upon reaching the age of
majority", are citizens of the Philippines (Article
IV, Section 1, subdivisions 3 and 4), our
fundamental law clearly refers to legitimate
children (Chiong Bian vs. De Leon, 46 Off. Gaz.,
3652-3654; Serra v. Republic, L-4223, May 12,
1952).
Similarly, the children alluded to in said
section 15 are those begotten in lawful
wedlock, when the adopter, at least is the
father. In fact, illegitimate children are under
the parental authority of the mother and follow
her nationality, not that of the illegitimate
father (U.S. vs. Ong Tianse, 29 Phil. 332, 335336; Santos Co vs. Govt of the Philippines, 52
Phil. 543, 544; Serra v. Republic, supra;
Gallofin v. Ordoez, 70 Phil. 287; Quimsuan vs.
Republic, L-4693, Feb. 16, 1953). Although,
adoption gives "to the adopted person the
same rights and duties as if he were a
legitimate child of the adopter", pursuant to
said Article 341 of our Civil Code, we have
already seen that the rights therein
alluded to are merely those enumerated
in Article 264, and do not include the
acquisition of the nationality of the
adopter.

On 30th of October, 1915 the steamship


Tian arrived at the port of Manila.
A woman, Marcosa S. Dy Jiongco, together
with two children, Ng Tio a female of the
age of 9 years, and Ng Hian a boy of 16
years of age (the petitioner herein) were on
board.
Marcosa S. Dy Jiongco, a Filipina born of a
Filipina mother and a Chinese father was
married to a Chinaman by the name of
(Filipino name) Juan Uy Tue, (Chinese
name) Ng Chion Tue.
That Juan Uy Tue (Ng Chion Tue), before
his marriage with Marcosa S. Dy Jiongco,
had been married to a Chinese woman with
whom he had some children, the petitioner
herein and also one called Ng Guan.
It appears that Ng Guan was residing in the
Philippine Islands at the time of the
presentation of the present petition;
The Chinese wife of Juan Uy Tue died while
the petitioner herein, Ng Hian, was a very
small child;
Juan Uy Tue, after the death of his Chinese
wife, married Marcosa S. Dy Jiongco.
Ng Tio was the daughter of the brother of
the said Juan Uy Tue, born of a Chinese
father and mother; that the father of the
little girl had given her to Marcosa S. Dy
Jiongco;
Marcosa S. Dy Jiongco, being the
stepmother of the said Ng Hian, adopted
him and was bringing him to the Philippine
Islands to study.
The Board of Special Inquiry refused the
right of each child to enter the Philippine
Islands;
However, after a rehearing, Ng Tio was
admitted but Ng Hian was not.
Petitioner petitioned for habeas corpus in
the CFI, CFI granted entry to petitioner,
Collector of Customs appealed; Thus the
present petition;

Issue:
Whether or not the minor children of a
deceased resident Chinese merchant have a
right to enter the territory of the Philippine
Islands;
Held:

Moreover, as used in said section 15 of


the
Naturalization
Law,
the
term
"children" could not possibly refer to
those whose relation to the naturalized
person is one created by legal fiction, as,
for instance, by adoption, for, otherwise,
the place and time of birth of the child
would be immaterial. The fact that the
adopted persons involved in the case at bar
are illegitimate children of appellant Ching
Leng does not affect substantially the legal
situation before us, for, by legal fiction, they
are now being sought to be given the status of
legitimate children of said appellant, despite
the circumstance that the Civil Code of the
Philippine does not permit their legitimation.

It is true that the petitioner, Ng Hian, had never


been in the Philippine Islands before. It is also
true that the said Marcosa S. Dy Jingco
was his stepmother. She swore positively
that she had adopted him. That fact is not
denied of record. Until the fact is denied we
must accept it. There is nothing in the
record which shows or tends to show that
she had not adopted him in good faith.
The question whether or not Marcosa S. Dy
Jiongco could bring Ng Hian into the territory of
the Philippine Islands as her adopted son has
been discussed by the Federal Courts of the
United States. In the case of Ex parte Fong
Yim (134 Fed. Rep., 938), the court held
that:

Ng Hian v. Collector
34 Phil 248

A Chinese merchant domiciled in the


United States has the right to bring into

Private International Law

Page 63

this country with his wife minor children


legally adopted by him in China, where it
is shown that the adoption was bona fide,
and that the children have lived as
members of his family and have been
supported by him for several years.
The court further said:
Of course, the question whether the adoption is
a genuine one is a question of fact, open to
investigation . . . . The evidence shows that the
practice of adopting children in China is very
common, that it takes place substantially
without legal formalities, but that the rights
and obligations of children adopted and
recognized as such are similar to those of
natural
children.
Under
these
circumstances I can see no difference
between the legal status of adopted
children and of natural children. The
Supreme Court (of the United States) having
decided that a Chinese merchant domiciled
in this country has the right to bring into
it his natural children, I think that the
same decision is authority for the
proposition that he has the right to
introduce his adopted children;
ROEHR VS. RODRIGUEZ
Facts:
Petitioner Wolfgang Roehr, a German citizen,
married a Filipina, Respondent Carmen
Rodriguez in Germany. The marriage was
ratified in Tayasan, Negros Oriental. They had
two children. Private respondent filed a petition
for the declaration of nullity of marriage before
the RTC of Makati. Petitioner filed a motion to
dismiss but was denied by the trial court.
However, The petitioner obtained a decree of
divorce from the Court of First Instance of
Hamburg - Blankenese with the custody of the
children granted to the father.

such decree is valid according to the


national law of the foreigner. Relevant to
the present case is Pilapil v. Ibay-Somera,
where the Court specifically recognized the
validity of a divorce obtained by a German
citizen in his country, the Federal Republic of
Germany. The Court held in Pilapil that a
foreign divorce and its legal effects may be
recognized in the Philippines insofar as
respondent is concerned in view of the
nationality principle in our civil law on the
status of persons.
In this case, the divorce decree issued by
the German court dated December 16,
1997 has not been challenged by either of
the parties. In fact, save for the issue of
parental custody, even the trial court
recognized said decree to be valid and binding,
thereby endowing private respondent the
capacity to remarry. Thus, the present
controversy mainly relates to the award of the
custody of their two children, Carolynne and
Alexandra Kristine, to petitioner.
As a general rule, divorce decrees
obtained by foreigners in other countries
are recognizable in our jurisdiction, but
the legal effects thereof, e.g. on custody,
care and support of the children, must
still be determined by our courts. Before
our courts can give the effect of res judicata to
a foreign judgment, such as the award of
custody to petitioner by the German court, it
must be shown that the parties opposed
to the judgment had been given ample
opportunity to do so on grounds allowed
under Rule 39, Section 50 of the Rules of
Court (now Rule 39, Section 48, 1997
Rules of Civil Procedure), to wit:
SEC.
50.
Effect
of
foreign
judgments. - The effect of a judgment
of a tribunal of a foreign country,
having jurisdiction to pronounce the
judgment is as follows:

Issue:
Whether or not the legal effects of a divorce
obtained from a foreign country such as
support and custody of the children can be
determined in our courts?
Held:
Yes. In order to take effect, a foreign
judgement must clearly show that the
opposing party has been given ample
opportunity to do so under the Rules of
Civil Procedure. Accordingly, the respondent
was not given the opportunity to challenge the
judgment of the German Court, therefore, legal
effects of divorce must be determined in our
courts. The court held that the trial court has
jurisdiction over the issue between the parties
as to who has the parental custody.

In Garcia v. Recio,19 Van Dorn v. Romillo,


Jr.,20 and Llorente v. Court of Appeals, the
Court consistently held that a divorce
obtained abroad by an alien may be
recognized in our jurisdiction, provided

Private International Law

(a) In case of a judgment upon a


specific
thing,
the
judgment
is
conclusive upon the title to the thing;
(b) In case of a judgment against a
person,
the
judgment
is
presumptive evidence of a right as
between the parties and their
successors
in
interest
by
a
subsequent title; but the judgment
may be repelled by evidence of a
want of jurisdiction, want of notice
to the party, collusion, fraud, or
clear mistake of law or fact.
It is essential that there should be an
opportunity to challenge the foreign judgment,
in order for the court in this jurisdiction to
properly determine its efficacy. In this
jurisdiction, our Rules of Court clearly
provide that with respect to actions in
personam, as distinguished from actions
in rem, a foreign judgment merely
constitutes prima facie evidence of the

Page 64

justness of the claim of a party and, as


such, is subject to proof to the contrary.
In the present case, it cannot be said that
private respondent was given the opportunity
to challenge the judgment of the German court
so that there is basis for declaring that
judgment as res judicata with regard to the
rights of petitioner to have parental custody of
their two children. The proceedings in the
German court were summary.
Exam is on Wednesday
630 to 830
Coverage is from start to parents and
children

Private International Law

Page 65

You might also like